X



トップページ数学
1002コメント575KB
スレタイ 箱入り無数目を語る部屋17
レス数が1000を超えています。これ以上書き込みはできません。
0001132人目の素数さん
垢版 |
2024/03/05(火) 08:04:40.23ID:FscjMFDQ
前スレが1000近く又は1000超えになったので、新スレを立てる

https://rio2016.5ch.net/test/read.cgi/math/1708680610/
前スレ スレタイ 箱入り無数目を語る部屋16

(参考)時枝記事
https://imgur.com/a/8bqlb08
数学セミナー201511月号「箱入り無数目」
https://rio2016.5ch.net/test/read.cgi/math/1620904362/401-406
純粋・応用数学(含むガロア理論)8 より
1.時枝問題(数学セミナー201511月号の記事)の最初の設定はこうだった。
「箱がたくさん,可算無限個ある.箱それぞれに,私が実数を入れる.
どんな実数を入れるかはまったく自由,例えばn番目の箱にe^nを入れてもよいし,すべての箱にπを入れてもよい.
もちろんでたらめだって構わない.そして箱をみな閉じる.
今度はあなたの番である.片端から箱を開けてゆき中の実数を覗いてよいが,一つの箱は開けずに閉じたまま残さねばならぬとしよう.
どの箱を閉じたまま残すかはあなたが決めうる.
勝負のルールはこうだ. もし閉じた箱の中の実数をピタリと言い当てたら,あなたの勝ち. さもなくば負け.
勝つ戦略はあるでしょうか?」

2.続けて時枝はいう
 私たちのやろうとすることはQのコーシー列の集合を同値関係で類別してRを構成するやりかた(の冒頭)に似ている.
但しもっときびしい同値関係を使う.
実数列の集合 R^Nを考える.
s = (s1,s2,s3 ,・・・),s'=(s'1, s'2, s'3,・・・ )∈R^Nは,ある番号から先のしっぽが一致する∃n0:n >= n0 → sn= s'n とき同値s 〜 s'と定義しよう(いわばコーシーのべったり版).
念のため推移律をチェックすると,sとs'が1962番目から先一致し,s'とs"が2015番目から先一致するなら,sとs"は2015番目から先一致する.
〜は R^N を類別するが,各類から代表を選び,代表系を袋に蓄えておく.
幾何的には商射影 R^N→ R^N/〜の切断を選んだことになる.
任意の実数列s に対し,袋をごそごそさぐってそいつと同値な(同じファイパーの)代表r= r(s)をちょうど一つ取り出せる訳だ.
sとrとがそこから先ずっと一致する番号をsの決定番号と呼び,d = d(s)と記す.
つまりsd,sd+1,sd+2,・・・を知ればsの類の代表r は決められる.
更に,何らかの事情によりdが知らされていなくても,あるD>=d についてsD+1, sD+2,sD+3,・・・
が知らされたとするならば,それだけの情報で既に r = r(s)は取り出せ, したがってd= d(s)も決まり,
結局sd (実はsd,sd+1,・・・,sD ごっそり)が決められることに注意しよう.
(補足)
sD+1, sD+2,sD+3,・・・:ここでD+1などは下付添え字

つづく
0002132人目の素数さん
垢版 |
2024/03/05(火) 08:05:02.26ID:FscjMFDQ
つづき

3.
問題に戻り,閉じた箱を100列に並べる.
箱の中身は私たちに知らされていないが, とにかく第l列の箱たち,第2列の箱たち第100 列の箱たちは100本の実数列s^1,s^2,・・・,s^100を成す(肩に乗せたのは指数ではなく添字).
これらの列はおのおの決定番号をもつ.
さて, 1〜100 のいずれかをランダムに選ぶ.
例えばkが選ばれたとせよ.
s^kの決定番号が他の列の決定番号どれよりも大きい確率は1/100に過ぎない.
 第1列〜第(k-1) 列,第(k+1)列〜第100列の箱を全部開ける.
第k列の箱たちはまだ閉じたままにしておく.
開けた箱に入った実数を見て,代表の袋をさぐり, s^1〜s^(k-l),s^(k+l)〜s^100の決定番号のうちの最大値Dを書き下す.
 いよいよ第k列 の(D+1) 番目から先の箱だけを開ける:s^k(D+l), s^k(D+2),s^k(D+3),・・・.いま
 D >= d(s^k)
を仮定しよう.この仮定が正しい確率は99/100,そして仮定が正しいばあい,上の注意によってs^k(d)が決められるのであった.
おさらいすると,仮定のもと, s^k(D+1),s^k(D+2),s^k(D+3),・・・を見て代表r=r(s^k) が取り出せるので
(代表)列r のD番目の実数rDを見て, 「第k列のD番目の箱に入った実数はs^k(D)=rDと賭ければ,めでたく確率99/100で勝てる.
確率1-ε で勝てることも明らかであろう.
(補足)
s^k(D+l), s^k(D+2),s^k(D+3),・・・, rD:ここで^kは上付き添え字、(D+l), Dなどは下付添え字

さらに、数学セミナー201511月号P37 時枝記事に、次の一文がある
「R^N/〜 の代表系を選んだ箇所で選択公理を使っている.
その結果R^N →R^N/〜 の切断は非可測になる.
ここは有名なヴィタリのルベーグ非可測集合の例(Q/Zを「差が有理数」で類別した代表系, 1905年)にそっくりである.」

さらに、過去スレでは引用しなかったが、続いて下記も引用する
「逆に非可測な集合をこさえるには選択公理が要る(ソロヴェイ, 1970年)から,この戦略はふしぎどころか標準的とさえいえるかもしれない.
しかし,選択公理や非可測集合を経由したからお手つき, と片付けるのは,面白くないように思う.
現代数学の形式内では確率は測度論によって解釈されるゆえ,測度論は確率の基礎, と数学者は信じがちだ.
だが,測度論的解釈がカノニカル, という証拠はないのだし,そもそも形式すなわち基礎, というのも早計だろう.
確率は数学を越えて広がる生き物なのである(数学に飼いならされた部分が最も御しやすいけれど).」

つづく
0003132人目の素数さん
垢版 |
2024/03/05(火) 08:05:23.15ID:FscjMFDQ
つづき

「もうちょっと面白いのは,独立性に関する反省だと思う.
確率の中心的対象は,独立な確率変数の無限族
X1,X2,X3,…である.
いったい無限を扱うには,
(1)無限を直接扱う,
(2)有限の極限として間接に扱う,
二つの方針が可能である.
確率変数の無限族は,任意の有限部分族が独立のとき,独立,と定義されるから,(2)の扱いだ.
(独立とは限らない状況におけるコルモゴロフの拡張定理なども有限性を介する.)
しかし,素朴に,無限族を直接扱えないのか?
扱えるとすると私たちの戦略は頓挫してしまう.
n番目の箱にXnのランダムな値を入れられて,ある箱の中身を当てようとしたって,
その箱のX と他のX1,X2,X3,・・・がまるまる無限族として独立なら,
当てられっこないではないか−−他の箱から情報は一切もらえないのだから.
勝つ戦略なんかある筈ない,と感じた私たちの直観は,無意識に(1)に根ざしていた,といえる.
ふしぎな戦略は,確率変数の無限族の独立性の微妙さをものがたる, といってもよい.」

数学セミナー201511月号の記事で、引用していなかった部分を、以下に引用する(^^;

”ばかばかしい,当てられる筈があるものか,と感じられるだろう.
何か条件が抜け落ちているのではないか,と疑う読者もあろう.問題を読み直していただきたい.
条件はほんとうに上記のとおり.無限個の実数が与えられ,一個を除いてそれらを見た上で,除いた一個を当てよ,というのだ.
ところがところが--本記事の目的は,確率99%で勝てそうな戦略を供することにある.
この問題はPeter Winkler氏との茶のみ話がてら耳にした.氏は原型をルーマニアあたりから仕入れたらしい.”
(引用終り)

この部分を掘り下げておくと
1.時枝氏は、この記事を、数学の定理の紹介とはしていないことに気付く
2.”Peter Winkler氏との茶のみ話がてら耳にした.氏は原型をルーマニアあたりから仕入れたらしい.”と
3.まあ、お気楽な、おとぎ話とまでは言ってないとしても、その類いの話として紹介しているのだった

ついでに”コルモゴロフの拡張定理”について、時枝記事は上記に引用の通りだが
1.”確率変数の無限族は,任意の有限部分族が独立のとき,独立,と定義されるから,(2)の扱いだ.(独立とは限らない状況におけるコルモゴロフの拡張定理なども有限性を介する.)”と
  そして、”しかし,素朴に,無限族を直接扱えないのか? 扱えるとすると私たちの戦略は頓挫してしまう.”とも
  記事の結論として、”勝つ戦略なんかある筈ない,と感じた私たちの直観は,無意識に(1)に根ざしていた,といえる.
ふしぎな戦略は,確率変数の無限族の独立性の微妙さをものがたる, といってもよい”と締めくくっているのだった
2.言いたいことは、”コルモゴロフの拡張定理”を使えば、この時枝解法が成り立つという主張にはなってないってこと
3.そして、”コルモゴロフの拡張定理”を使ってブラウン運動を記述できるなら、ブラウン運動こそ、”他から情報は一切もらえない”を実現しているように思えるのだが
(引用終り)

つづく
0004132人目の素数さん
垢版 |
2024/03/05(火) 08:07:52.95ID:FscjMFDQ
つづき

https://mathoverflow.net/questions/151286/probabilities-in-a-riddle-involving-axiom-of-choice
Probabilities in a riddle involving axiom of choice
asked Dec 9 '13 at 16:16 Denis
(Denis質問)
I think it is ok, because the only probability measure we need is uniform probability on {0,1,…,N?1}, but other people argue it's not ok, because we would need to define a measure on sequences, and moreover axiom of choice messes everything up.
(Pruss氏)
The probabilistic reasoning depends on a conglomerability assumption, ・・・and we have no reason to think that the conglomerability assumption is appropriate.
(Huynh氏)
If it were somehow possible to put a 'uniform' measure on the space of all outcomes, then indeed one could guess correctly with arbitrarily high precision, but such a measure doesn't exist.

mathoverflowは時枝類似で
・Denis質問でも、もともと”but other people argue it's not ok, because we would need to define a measure on sequences, and moreover axiom of choice messes everything up.”
 となっています。Denisの経歴を見ると、彼は欧州の研究所勤務で、other peopleは研究所の確率に詳しい人でしょう
・Pruss氏とHuynh氏とは、経歴を見ると、数学DRです。両者とも、このパズル(=riddle)は、可測性が保証されていないと回答しています

http://www.ma.huji.ac.il/hart/
Sergiu Hart
http://www.ma.huji.ac.il/hart/#puzzle
Some nice puzzles:
http://www.ma.huji.ac.il/hart/puzzle/choice.pdf?
Choice Games November 4, 2013
P2
Remark. When the number of boxes is finite Player 1 can guarantee a win
with probability 1 in game1, and with probability 9/10 in game2, by choosing
the xi independently and uniformly on [0, 1] and {0, 1,..., 9}, respectively.

Sergiu Hart氏は、ちゃんと”シャレ”が分かっている(関西人かもw)
Some nice puzzles Choice Games と、”おちゃらけ”であることを示している
かつ、”P2 Remark.”で当てられないと暗示している
また、”A similar result, but now without using the Axiom of Choice.GAME2”
で、選択公理なしで同じことが成り立つから、”選択公理”は、単なる目くらましってことも暗示している

つづく
0005132人目の素数さん
垢版 |
2024/03/05(火) 08:08:22.38ID:FscjMFDQ
つづき

だめなのは、時枝記事だ。まあ、題名はおちゃらけだが、もっとはっきり、数学パズルとした方がよかったろう
非可測で、ヴィタリに言及しているのが、ミスリードだ
Hart氏の”A similar result, but now without using the Axiom of Choice.GAME2”のように、選択公理不使用のGAME2があるから、
ソロヴェイの定理(下記 wikipedia ご参照)から、ヴィタリのような非可測は否定される
conglomerabilityか、あるいは総和ないし積分が発散する非正規な分布により、可測性が保証されないと考えるべき
時枝氏は、確率変数の無限族の独立性が理解できていないのも痛いね

https://ja.wikipedia.org/wiki/%E3%83%B4%E3%82%A3%E3%82%BF%E3%83%AA%E9%9B%86%E5%90%88
ヴィタリ集合
ヴィタリ集合が存在し、それらの存在は選択公理の仮定の下で示される。1970年にロバート・ソロヴェイ(英語版)は、到達不能基数の存在を仮定することにより、全ての実数の集合がルベーグ可測となるような(選択公理を除いた)ツェルメロ・フレンケル集合論のモデルを構築した[2]。

https://ja.wikipedia.org/wiki/%E3%82%BD%E3%83%AD%E3%83%B4%E3%82%A7%E3%82%A4%E3%83%A2%E3%83%87%E3%83%AB
ソロヴェイモデル
ソロヴェイモデルはロバート M. ソロヴェイ (1970)によって構成されたモデルでツェルメロ=フレンケル集合論 (ZF) の全ての公理が成り立ち、選択公理を除去し、実数の集合が全てルベーグ可測であるようにしたものである。この構成は到達不能基数の存在に依拠している。
これによってソロヴェイはルベーグ不可測集合の存在をZFC (ZF+選択公理) から証明するには、少なくとも到達不能基数の存在がZFCと矛盾しない限り、選択公理が本質的に必要であることを示した。
ステートメント
DC は従属選択公理の略記とする。
ソロヴェイの定理は次のことである。 到達不能基数の存在を仮定する。このとき、適切な強制拡大 V[G] の ZF+DC の内部モデルであって、実数のいかなる集合も全て、ルベーグ可測であって perfect set property を満たしベールの性質を満たすというモデルがある。
構成
ソロヴェイはそのモデルを二つのステップによって構成した。まず初めに、到達不能基数 κ を含む ZFC のモデル M から始める。
最初のステップでは M のレヴィ崩壊 M[G] を取る。

(引用終り)

つづく
0006132人目の素数さん
垢版 |
2024/03/05(火) 08:08:42.55ID:FscjMFDQ
つづき

(完全勝利宣言!w)(^^
https://rio2016.5ch.net/test/read.cgi/math/1666352731/767 (775の修正を追加済み)
>>701-702 補足説明
 >>760にも書いたが、
” a)確率上、開けた箱と開けてない箱とは、扱いが違う”>>701
をベースに、時枝記事>>1のトリックを、うまく説明できると思う

1)いま、時枝記事のように
 問題の列を100列に並べる
 1~100列 のいずれか、k列を選ぶ(1<=k<=100)
 k以外の列を開け、99列の決定番号の最大値をdmax99 とする
 k列は未開封なので、確率変数のままだ
 なので、k列の決定番号をXdkと書く
2)もし、Xdk<=dmax99 となれば、dmax99+1以降の箱を開けて
 k列の属する同値類を知り、代表列を知り、dmax99番目の箱の数を参照して
 その値を問題のk列の箱の数とすれば、勝てる
(∵決定番号の定義より、dmax99番目の箱は、問題のk列とその代表とで一致しているから)
3)しかし、決定番号は、
 自然数N同様に非正則分布>>13だから、これは言えない
 つまり、確率はP(Xdk<=dmax99)=0 とすべきだ
(非正則分布なので、上限なく発散しているので、dmax99<=Xdk となる場合が殆ど)
4)もし、決定番号が、[0,M](Mは有限の正整数)の一様分布ならば
 dmax99が分かれば、例えば、
 0<=dmax99<=M/2 ならば、勝つ確率は1/2以下
 M/2<=dmax99<=M ならば、勝つ確率は1/2以上
 と推察できて
 それを繰り返せば、大数の法則で、P(Xdk<=dmax99)=99/100が言えるだろう
(注:dmax99は、100列中の99列の最大値なので、P(Xdk<=dmax99)=99/100が正しいだろう)
 しかし、非正則分布では、このような大数の法則は適用できない
5)人は無意識に、決定番号も正則分布のように錯覚して、トリックに嵌まるのです
 しかし、非正則分布では、大数の法則も使えない
 結局、時枝記事の99/100は、だましのトリックってことです

つづく
0007132人目の素数さん
垢版 |
2024/03/05(火) 08:09:09.65ID:FscjMFDQ
つづき

さて、上記を補足します

1)いま、加算無限の箱が、iid 独立同分布 とします
 箱を、加算無限個の確立変数の族 X1,X2,・・Xi・・ として扱うのが
 現代の確率論の常套手段です
2)いま、サイコロ1〜6の数字を入れるならば、任意Xiの的中確率は1/6
 コイントス 0,1の数字を入れるならば、的中確率は1/2
 もし、区間[0,1]の実数を入れるならば、的中確率は0
 もちろん、時枝記事の通り任意実数r∈Rならば やはり、的中確率は0
 です
3)ところが、時枝記事では、確立変数の族 X1,X2,・・Xi・・ を100列に並べ替え
 数列のしっぽ同値類の類別と、類別の代表を使って、決定番号を決めて
 決定番号の大小比較から、ある箱Xjについて、的中確率99/100に改善できる
 と主張します
4)「そんなバカな!」というのが、上記の主張です

マジ基地は無視してさらに補足します

1)時枝記事の決定番号をdとすると、dは1から無限大(∞)までを渡ります
 このような場合、しばしば非正則分布(正則でない)を成します(下記)
2)非正則分布の場合、全体が無限大に発散して、平均値も無限大になり
 分散や標準偏差σなども、無限大に発散します
3)具体例として、テスト回数無限回の合計点で成績評価をする場合を考えます
 テスト回数が、1回、2回、・・n回、・・
 もし、テスト回数が有限なら 例えば100回で1回の満点100点として、総計10,000(1万)点ですが
 テスト回数が無限回ならば、毎回1点の人の総計も無限大(∞)に発散し
 毎回100点満点の人の総計も無限大に発散しまず
 試験の点の合計では、毎回1点の人も毎回100点も区別ができなくなります
 この合計については、平均は無限大、分散や標準偏差σなども無限大に発散します
4)ところで、時枝氏の数学セミナー201511月号の記事では
 このような非正則分布を成す決定番号を、あたかも平均値や分散・標準偏差σが有限である
 正則分布のように扱い、確率 99/100とします

これは、全くのデタラメでゴマカシです

(参考)
https://ai-trend.jp/basic-study/bayes/improper_prior/
AVILEN Inc. 2020
2020/04/14
非正則事前分布とは?〜完全なる無情報事前分布〜
ライター:古澤嘉啓
目次
1 非正則な分布とは?一様分布との比較
2 非正則分布は確率分布ではない!?
3 非正則事前分布は完全なる無情報事前分布
4 まとめ

つづく
0008132人目の素数さん
垢版 |
2024/03/05(火) 08:09:31.03ID:FscjMFDQ
つづき

なお、
おサル=サイコパス*のピエロ(不遇な「一石」https://textream.yahoo.co.jp/personal/history/comment?user=_SrJKWB8rTGHnA91umexH77XaNbpRq00WqwI62dl 表示名:ムダグチ博士 Yahoo! ID/ニックネーム:hyperboloid_of_two_sheets**) (Yahoo!でのあだ名が、「一石」)
<*)サイコパスの特徴>
(参考)https://keiji-pro.com/magazine/10/ 刑事事件マガジン 更新日:2023.10.13
サイコパス(精神病質者)の10の特徴と診断基準|実はあなたの周りに・・・?
サイコパスとは、「反社会性パーソナリティ障害」という精神病者のこと。
サイコパスの10の特徴 表面上は口達者利己的・自己中心的 平然と嘘をつく
(**)注;https://en.wikipedia.org/wiki/Hyperboloid Hyperboloid
Hyperboloid of two sheets :https://upload.wikimedia.org/wikipedia/commons/thumb/f/f2/Hyperboloid2.png/150px-Hyperboloid2.png
https://ja.wikipedia.org/wiki/%E5%8F%8C%E6%9B%B2%E9%9D%A2 双曲面
二葉双曲面 :https://upload.wikimedia.org/wikipedia/commons/thumb/b/b5/HyperboloidOfTwoSheets.svg/180px-HyperboloidOfTwoSheets.svg.png

おサルさんの正体判明!(^^)
スレ12 https://rio2016.5ch.net/test/read.cgi/math/1671460269/923 より
”「ガロア理論 昭和で分からず 令和でわかる
 #平成どうしたw」
昭和の末期に、どこかの大学の数学科
多分、代数学の講義もあったんだ
でも、さっぱりで、落ちこぼれ卒業して
平成の間だけでも30年、前後を加えて35年か”
”(修士の)ボクの専攻は情報科学ですね”とも

可哀想に、数学科のオチコボレで、鳥無き里のコウモリ***)そのもので、威張り散らし、誰彼無く噛みつくアホ
本来お断り対象だが、他のスレでの迷惑が減るように、このスレで放し飼いとするw(^^

注***)鳥無き里のコウモリ:自分より優れた数学DRやプロ数学者が居ないところで、たかが数学科のオチコボレが、威張り散らす姿は、哀れなり〜!(^^;

なお
低脳幼稚園児のAAお絵かき
小学レベルとバカプロ固定
は、お断りです

小学生がいますので、18金(禁)よろしくね!(^^

つづく
0009132人目の素数さん
垢版 |
2024/03/05(火) 08:10:02.69ID:FscjMFDQ
つづき

なお、スレ14から引用追加
https://rio2016.5ch.net/test/read.cgi/math/1696677610/
834132人目の素数さん
2024/02/05 ID:WZ3A8eO8
>>833
あなたのいう病的な空間とは具体的になんですか?
箱入り無数目の確率空間は有限集合{1,・・・,100}であって
まったく病的でもなんでもありませんが、理解できてますか?

922132人目の素数さん
2024/02/09 ID:saO8wFId
まずここから間違ってるのが笑える
>箱入り無数目の確率空間は有限集合{1,・・・,100}であって
>まったく病的でもなんでもありませんが、理解できてますか?

923132人目の素数さん
2024/02/09 ID:nxQ27BqK
>>922 自分が間違ってることに全然気づかない馬鹿っぷりが超笑える ギャハハハハハハ!!!

925132人目の素数さん
2024/02/0 ID:saO8wFId
>>923
こいつ確率論なんもわかってねーんだな
(引用終り)

テンプレは以上です
0010132人目の素数さん
垢版 |
2024/03/05(火) 10:06:43.99ID:GpIsjTrm
>>9
>箱入り無数目の確率空間は有限集合{1,・・・,100}
のどこがどう間違っているのか答えてください
0011132人目の素数さん
垢版 |
2024/03/05(火) 10:12:27.19ID:M7m/0Oso
「箱入り無数目」戦争は2024年2月21日に1の無条件降伏で終了いたしました

https://rio2016.5ch.net/test/read.cgi/math/1707524330/778-779
0778 2024/02/21(水) 14:24:07.05 ID:UxzUPPp/
1.出題者が箱の中に数を入れて閉じた瞬間、箱の中は出題者が入れた数以外の結果はあり得ない
2.そして、尻尾同値類の代表を決めた瞬間、決定番号も決まり
各々の箱について中身と代表の対応する項が一致するしないも決まってしまう
3.起こりえる結果が複数あり得る(つまり確率事象となる)のは回答者がどの列(したがってどの箱)を選択するかだけである
4.そして「箱入り無数目」の方法によれば、選択肢がいくつあろうが、箱の中身と代表の対応する項が相違するのはたかだか1つ
5.だから予測をはずす確率は1−1/n=(n−1)/nである
(完)

0779 2024/02/21(水) 16:55:31.71 ID:ACY+AqAt
箱入り無数目の総括を有難うございます
完全決着ですね
0012132人目の素数さん
垢版 |
2024/03/05(火) 10:41:36.54ID:4qb1Yhq7
>>6
>k列は未開封なので、確率変数のままだ
誤解1:正解は定数です

>しかし、決定番号は、自然数N同様に非正則分布だから、
誤解2:k列は定数なので、その決定番号にも分布はありません

>つまり、確率はP(Xdk<=dmax99)=0 とすべきだ
誤解3:任意の自然数nについて
R^Nにおける決定番号dmax99以下の列の全体の測度は
0ではなく非可測です、したがって、P(Xdk<=dmax99)も
0ではなく算定不能です

>大数の法則で、P(Xdk<=dmax99)=99/100が言えるだろう
誤解4:正則分布の場合には、積分によって99/100がいえますが
それは大数の法則とは無関係です

>人は無意識に、決定番号も正則分布のように錯覚して、トリックに嵌まるのです
誤解5:決定番号が非正則分布だと主張する根拠を
無限列が無情報事前分布としてR^N上一様分布していることに
求めていると思われますが、そもそもその前提が必要ありません

おだいじに
0013132人目の素数さん
垢版 |
2024/03/05(火) 10:47:58.17ID:p19KPbL2
>>7
>いま、加算無限の箱が、iid 独立同分布 とします
誤解6:加算ではなく可算です またiidという前提は無用です

>箱を、加算無限個の確立変数の族 X1,X2,・・Xi・・ として扱うのが現代の確率論の常套手段です
誤解7:現代の確率論ではそんなことは全く云ってません

>時枝記事では、・・・ある箱Xjについて、的中確率99/100に改善できると主張します
誤解8:時枝記事では、ある箱を固定しておりません
箱100個のうち99個について代表の対応する項と中身が一致し
そのような箱を選ぶ確率が99/100だといってるだけです

>「そんなバカな!」というのが、上記の主張です
1は、そもそも問題を取り違えている上に、何が確率変数かも取り違えてます

頭を冷やしましょう
0014基礎論婆
垢版 |
2024/03/05(火) 10:51:55.35ID:RyzmvcBl
選択公理で答えを選べばすべての目が100%当ります
0015132人目の素数さん
垢版 |
2024/03/05(火) 10:54:30.20ID:2o6SqnJo
>>7
>マジ**は無視して
書かれていることを読み取れず
書かれてないことを読み取る
マジ**は、1さん、あなたです

>時枝氏の数学セミナー201511月号の記事では
>非正則分布を成す決定番号を、
>あたかも平均値や分散・標準偏差σが有限である正則分布のように扱い、
>確率 99/100とします

>>12の誤解5で述べたように
決定番号が非正則分布だと主張する根拠を
無限列が無情報事前分布としてR^N上一様分布していることに
求めていると思われますが、そもそもその前提がおかしいです

また>>13の誤解8で述べたように
確率99/100を導けるのは
「箱100個のうち99個について代表の対応する項と中身が一致している」
からです
さらに>>12の誤解4で述べたように
決定番号の分布が正則になるようにR^Nの測度を定めるならば
積分により99/100が求まります
0016132人目の素数さん
垢版 |
2024/03/05(火) 11:11:19.68ID:CteCvkjF
ところで「仮に」無限列が正則分布を為すと前提した場合
勿論、100列は、iid 独立同分布です
0017基礎論婆
垢版 |
2024/03/05(火) 11:24:54.30ID:RyzmvcBl
悲報、サイコロ投げの確率論がわかりませんでした
0018132人目の素数さん
垢版 |
2024/03/05(火) 11:26:33.40ID:GpIsjTrm
>>16
大間違い
100列は固定されているので分布は意味を持たない
0019基礎論婆
垢版 |
2024/03/05(火) 11:28:45.90ID:RyzmvcBl
ウマシカ野郎は時枝記事の前半だけが正しい
数学板公安委員会婆は後半だけが正しい
二つ合わせると矛盾
0020132人目の素数さん
垢版 |
2024/03/05(火) 11:29:50.29ID:jgr2P7XK
以下のような分布は考えられる
第一項以降全部0 
 確率1/2
第一項に任意の実数 第二項以降全部0
 確率1/4
第二項にも任意の実数 第三項以降全部0
 確率1/8
・・・
第n項にも任意の実数 第n+1項以降全部0
 確率1/(2^(n+1))

この場合、確率1で、「全部の項が0の列」と尻尾同値
0021基礎論婆
垢版 |
2024/03/05(火) 11:32:40.32ID:RyzmvcBl
なりすまし野郎が参戦

以上、前スレのまとめ
0022132人目の素数さん
垢版 |
2024/03/05(火) 11:47:14.05ID:GpIsjTrm
出題列、出題列を並べ替えた100列、100列の決定番号
いずれも固定されているので分布を考えてもナンセンス

出題列が定まる前に数当てするゲームなら出題列は確率変数
なぜなら試行毎に出題列が変化するから
しかし箱入り無数目はそのようなゲームではない 問題を取り違えている
0023132人目の素数さん
垢版 |
2024/03/05(火) 11:52:29.95ID:GpIsjTrm
「見えないもの=確率変数」は誤り。
よって「出題列は固定されているが未開封なので確率変数である。よって分布に意味がある。」も誤り。
0024132人目の素数さん
垢版 |
2024/03/05(火) 13:55:19.10ID:p19KPbL2
より一般に以下のような分布は考えられる
mを2以上の任意の自然数とする
第一項以降全部0 
 確率1/m
第一項に任意の実数 第二項以降全部0
 確率1/m*(m−1)/m
第二項にも任意の実数 第三項以降全部0
 確率1/m*((m−1)/m)^2
・・・
第n項にも任意の実数 第n+1項以降全部0
 確率1/m*((m−1)/m)^n

この場合、確率1で、「全部の項が0の列」と尻尾同値
0025132人目の素数さん
垢版 |
2024/03/05(火) 13:57:54.69ID:p19KPbL2
>>24で示したような分布で数列を設定した場合
箱入り無数目で当てられる箱の中身は0に限る
このことからも明らかなように
箱入り無数目はランダムに入れた箱の中身をあてる戦略ではない
0026弥勒菩薩
垢版 |
2024/03/05(火) 15:46:24.94ID:RyzmvcBl
定義
ポーランド空間Xの同値関係Eとポーランド空間Yの同値関係Fがボレル同値とは
ボレル写像f:X->Yがあってx1Ex2<->f(x1)Ff(x2)、かつ逆向きのボレル写像g:Y->Xがあってy1Fy2<->g(y1)Eg(y2)

C0は2^ωの尻尾同値関係
C0(ω)はω^ωの尻尾同値関係
EvはR上のビタリ同値関係:xEvy<->x-y∈Q

定理
(1)C0とC0(ω)はボレル同値
(2)C0とEvはボレル同値
0027132人目の素数さん
垢版 |
2024/03/05(火) 18:23:42.07ID:V2E6ZA4q
ご苦労様です
スレ主です

1)まず、前振りの小話
 ・小学6年生がいる。親に耳で聞いた方程式の話で考えた
  普通は、ax+by=c、 x,y が未知数で、a,b,cが係数
  と書くところを
  xa+yb=z と書いて、a,b が未知数で、x,y,zが係数だという
  それを見ていたある数学者は「ぼくえらい、自分で考えたんだね!」とほめた
 ・私大文学部1年生がいる。耳で聞いた方程式の話で考えた
 (以下同じ)
  それを見ていた数学者は「私大文学部は入試で数学の試験がないんだ
  自分で考えたんだろうが、それ我流だぞ」と言った
 ・社会人1年生(私大文系)がいる。耳で聞いた方程式の話で考えた
 (以下同じ)
  それを見ていた数学者は「入試で数学の試験がなかったんだろうが
  我流だぞ。社会に出たら 常識がないと言われるぞ。もっと勉強しないと」と言った

2)前スレの最後部分の論争 上記のごとし
 ・初心者ならほほえましいだろうが
 ・いい年した大人が、確率論の無知を自慢している(「石が流れて木の葉が沈む」)
 ・数学科卒だぁ? 大学名言わない方がいいぞ、同窓生の恥だろう

3)確かに、「xa+yb=z と書いて、a,b が未知数で、x,y,zが係数だ!」は
 定義だから、定義さえすれば 数学的には可ではある
 しかし、いかにも素人くさいなw
 確率論も同様だ。屁理屈こね回すバカがいる

https://kotobank.jp/word/%E7%9F%B3%E3%81%8C%E6%B5%81%E3%82%8C%E3%81%A6%E6%9C%A8%E3%81%AE%E8%91%89%E3%81%8C%E6%B2%88%E3%82%80-431892
コトバンク
石が流れて木の葉が沈む(読み)いしがながれてこのはがしずむ
精選版 日本国語大辞典
0028132人目の素数さん
垢版 |
2024/03/05(火) 18:48:41.80ID:GpIsjTrm
>>27
> 確率論も同様だ。屁理屈こね回すバカがいる
それがおまえ
0029132人目の素数さん
垢版 |
2024/03/05(火) 18:51:04.75ID:GpIsjTrm
>>27
屁理屈はいいから
「未知のものは確率変数」
と書かれた書籍を早く示せ
0032132人目の素数さん
垢版 |
2024/03/05(火) 19:05:15.54ID:GpIsjTrm
>>31
定数
0033132人目の素数さん
垢版 |
2024/03/05(火) 19:07:22.20ID:GpIsjTrm
>>31
確率変数か定数かは未知か否かでは決まらない
試行毎に変化するか否かで決まる
違うと言うなら黙って>>29を実行せよ
0035132人目の素数さん
垢版 |
2024/03/05(火) 19:26:07.46ID:GpIsjTrm
>>34
屁理屈はいいと言ったばかりでこれだ
辞書で屁理屈の意味を調べろ
0036132人目の素数さん
垢版 |
2024/03/05(火) 19:27:48.39ID:GpIsjTrm
>>34
ちなみに感染確率の定義は?

あと早く>>29やってね 逃げないよう頼みますよ?
0038132人目の素数さん
垢版 |
2024/03/05(火) 19:36:33.54ID:GpIsjTrm
>>37
定義になってない
0039132人目の素数さん
垢版 |
2024/03/05(火) 19:40:42.17ID:GpIsjTrm
>>37
>>29は未だ?
0041132人目の素数さん
垢版 |
2024/03/05(火) 19:45:06.56ID:GpIsjTrm
>>40
いいから>>29に答えろ
答えたらこちらも答えてやる
0042132人目の素数さん
垢版 |
2024/03/05(火) 19:45:31.56ID:sME4PsKC
>>39
個々の問題をどう定式化してるか色んな場合を本で読んで勝手に確認すりゃいいだろ
医療の分野とかにたぶんいっぱいあるだろ
0043132人目の素数さん
垢版 |
2024/03/05(火) 19:58:14.71ID:sME4PsKC
PCR検査の確率モデルなんて、色んなところで記事になってる定番のネタなんだから自分で探して確認しろよ
感染してるかどうかは検査しようかしまいが、あらかじめ決まってることだから定数としてモデル化してる馬鹿なんてこの世にいねーだろ
0044132人目の素数さん
垢版 |
2024/03/05(火) 20:12:22.99ID:GpIsjTrm
>>43
じゃあ>34をなんで投稿した?
>自分で探して確認しろよ
そっくりお返しします
0045132人目の素数さん
垢版 |
2024/03/05(火) 20:13:01.26ID:GpIsjTrm
>>42
おまえは日本語読めんの?
なら小学校の国語からやり直し
0046132人目の素数さん
垢版 |
2024/03/05(火) 20:20:54.20ID:sv6mPLu1
さて、一般に以下のような分布が考えられる

1>ε>0とする

第一の箱が空いて任意の数が入れられる確率 1-ε (*)
第二の箱も空いて任意の数が入れられる確率 (1-ε)^2 (*)
・・・
第nの箱も空いて任意の数が入れられる確率 (1-ε)^n (*)
・・・

(*)開かない場合中身は予め決められた同値類の代表列の項

この場合、確率1で、決められた同値類の代表列と尻尾同値
さらにd1>d2の確率は 1/2-ε/(4-2ε)

ここでε→0とすると、
どの箱についても空いて任意の数が入れられる確率は1
d1>d2の確率も1/2

そしてここで勘のいい人は感づいたはずだが・・・

箱入り無数目で当てられるのは
実は出題者が入れた数ではなく
入れられなかった箱の中の同値類の代表列の項

最初から答えが分かっているんだから当たって当たり前
箱入り無数目はあらかじめ分かってる答えに対して
回答者がどれだけ箱に数を入れて邪魔できるか?というゲームだった!
(完)
0048132人目の素数さん
垢版 |
2024/03/05(火) 20:28:05.11ID:sv6mPLu1
>>46
>ここでε→0とすると
ε=0にはできない
(1-ε)<1 だから(1-ε)^n→0になる
1だったら1^n→0にならない
つまり、箱入り無数目はあくまでε>0の場合の>>46のモデルで成立するので
ε→0の極限とε=0の場合は、もちろん違う
0049132人目の素数さん
垢版 |
2024/03/05(火) 21:00:56.77ID:GpIsjTrm
>>47
命題の真偽は1例で真なら真と?
君って呆れるほど馬鹿なんだね
1例で偽なら偽だよ
0051132人目の素数さん
垢版 |
2024/03/05(火) 21:12:12.76ID:GpIsjTrm
>>50
反例も知らないの?
なら黙ってれば? 恥かくだけだから
0052132人目の素数さん
垢版 |
2024/03/05(火) 21:16:40.84ID:IYPmJEac
>>51
モデリングなんて職人芸なんだから、実戦で色んなパターンを覚える以外にないだろ
コロナ感染確率は感染してるかどうかわからないから確率変数でモデル化するんだろ
そこに試行がどうとか定数だとかそんなの全く関係ねーよ
0053132人目の素数さん
垢版 |
2024/03/05(火) 21:19:34.97ID:GpIsjTrm
>>52
おまえ日本語読めないの?
なら小学校の国語からやり直し
0055132人目の素数さん
垢版 |
2024/03/05(火) 21:26:43.89ID:GpIsjTrm
>>54
おまえ日本語読めないの?
なら小学校の国語からやり直し
0056132人目の素数さん
垢版 |
2024/03/05(火) 21:33:35.75ID:IYPmJEac
>>55
なんでコロナの感染確率は試行でも何でもないランダム要素のない確定していることなのに確率なの?
0057132人目の素数さん
垢版 |
2024/03/05(火) 21:34:21.89ID:GpIsjTrm
ある確率モデルで「未知のものは確率変数」ならば、あらゆる確率モデルで「未知のものは確率変数」

バカ丸出し
0058132人目の素数さん
垢版 |
2024/03/05(火) 21:35:27.44ID:IYPmJEac
>>55
さっさと厚生労働省に文句言いに行けよ
ランダム要素がないものに確率を使うなって
0059132人目の素数さん
垢版 |
2024/03/05(火) 21:35:33.97ID:GpIsjTrm
>>56
どうでもいい
そんなことを論じてると思ってるの?
おまえ馬鹿だろ
0060132人目の素数さん
垢版 |
2024/03/05(火) 21:36:06.55ID:GpIsjTrm
>>58
論点のすり替えご苦労さん
0061132人目の素数さん
垢版 |
2024/03/05(火) 21:38:23.57ID:IYPmJEac
>>57
お前が確率変数を使わずにモデル化できる未知のものを探してくればいいじゃん
0062132人目の素数さん
垢版 |
2024/03/05(火) 21:40:17.58ID:GpIsjTrm
>>61
なんで?
0063132人目の素数さん
垢版 |
2024/03/05(火) 21:40:30.35ID:GpIsjTrm
だから言ってるだろ
屁理屈はいいから「未知のものは確率変数」と書かれた書籍を示せって
日本語がわからないなら小学校の国語からやり直し
0064132人目の素数さん
垢版 |
2024/03/05(火) 21:40:53.90ID:GpIsjTrm
ある確率モデルで「未知のものは確率変数」ならば、あらゆる確率モデルで「未知のものは確率変数」

バカ丸出し
0065132人目の素数さん
垢版 |
2024/03/05(火) 21:41:21.77ID:GpIsjTrm
PCR検査の確率モデルで「未知のものは確率変数」ならば、あらゆる確率モデルで「未知のものは確率変数」

バカ丸出し
0066132人目の素数さん
垢版 |
2024/03/05(火) 21:45:35.21ID:IYPmJEac
結局、こいつこんな職人の勘みたいなところをつつくしかやることないんだな
0067132人目の素数さん
垢版 |
2024/03/05(火) 21:50:35.47ID:GpIsjTrm
>>66
つまりおまえは「確率論において未知のものは確率変数」は間違いだと思ってるってことでよいの?
0068132人目の素数さん
垢版 |
2024/03/05(火) 21:53:20.30ID:GpIsjTrm
職人の勘に左右されるってことはそういうことやろ?
違うか?
0071132人目の素数さん
垢版 |
2024/03/05(火) 21:56:00.97ID:GpIsjTrm
>>70
だから話を逸らすなよ
聞いたことに答えろカス
0072132人目の素数さん
垢版 |
2024/03/05(火) 21:57:27.35ID:GpIsjTrm
>>69
じゃ失せろよカス
0073132人目の素数さん
垢版 |
2024/03/05(火) 22:01:01.36ID:IYPmJEac
ここで、発端になったレスを思い出してみよう
こいつがなんでこんなに必死になってるかって言うと、コロナの例が>837を完全に否定してるからなのですね
コロナに感染してるかどうかなんて最初にすでに決まっていて突然変化するわけないからね
1例挙げれば十分な命題だったでしょ

837 132人目の素数さん 2024/02/23(金) 02:14:00.13 ID:xKynRG52
>833
>最初に決めるから確率変数じゃないとかね
君は文盲かい?
試行毎に変化するものが確率変数 変化しないものは確率変数でない
と書いたんだけど読めないかい? なら小学校の国語からやり直そうね

838 132人目の素数さん sage 2024/02/23(金) 02:16:11.67 ID:0hiCCwLy
>837
それがおかしいって言ってんだよ
見えないものが確率変数ね
0075132人目の素数さん
垢版 |
2024/03/05(火) 22:05:29.77ID:GpIsjTrm
ここで、発端になったレスを思い出してみよう
こいつがなんでこんなに必死になってるかって言うと、二つの封筒の例が>838を完全に否定してるからなのですね
相手の封筒の中身が見えないからといって確率変数としてしまうとパラドックスになるからね
1例挙げれば十分な命題だったでしょ

838 132人目の素数さん sage 2024/02/23(金) 02:16:11.67 ID:0hiCCwLy
>837
それがおかしいって言ってんだよ
見えないものが確率変数ね
0076132人目の素数さん
垢版 |
2024/03/05(火) 22:09:38.39ID:GpIsjTrm
>>73
じゃ聞くけどPCR検査の例における試行、確率変数、確率空間ってそれぞれ何?
答えによっては
>試行毎に変化するものが確率変数 変化しないものは確率変数でない
に抵触しないかもよ? 心して答えてね
0077132人目の素数さん
垢版 |
2024/03/05(火) 22:14:24.62ID:IYPmJEac
>>76
試行なんてねーよ
感染してるかどうかは最初から確定してるんだから
確率変数だって0か1の値をとるだけだろ
確率空間なんて確率変数が定まればなんでもいい
0078132人目の素数さん
垢版 |
2024/03/05(火) 22:23:13.76ID:GpIsjTrm
>>77
>試行なんてねーよ
つまり君は確率論における反例を挙げたわけじゃないってことね?
君、要らないから失せていいよ

以下、長崎県立大学のページから引用
試行: 実験や観測を行うこと
標本点: その試行で生じる個々の結果(今後しばしば ω と書く) ←定義に試行が使われている
標本空間: 標本点の全体(今後しばしば Ω と書く) ←定義に標本点が使われている
事象: 標本空間Ωの部分集合 ←定義に標本空間が使われている
根元事象: 標本点が1つの事象 ←定義に標本点が使われている


試行が未定義ならすべて未定義 是すなわち確率論に非ず
0079132人目の素数さん
垢版 |
2024/03/05(火) 22:24:32.09ID:GpIsjTrm
確率論を論じてるのかと思いきや似非確率論だったとさ
やれやれ
0080132人目の素数さん
垢版 |
2024/03/05(火) 22:30:32.50ID:GpIsjTrm
ちなみに二つの封筒問題は純粋に数学の問題なので

838 132人目の素数さん sage 2024/02/23(金) 02:16:11.67 ID:0hiCCwLy
>837
それがおかしいって言ってんだよ
見えないものが確率変数ね

の完全な反例です
誰かさんの似非確率論を用いた言いがかりとは違います
0081132人目の素数さん
垢版 |
2024/03/05(火) 22:31:36.33ID:IYPmJEac
>>79
だから何度言えばいいのこれ
確率論の話をしてるんじゃねーよ
モデリングの話をしてるんだよ
0082132人目の素数さん
垢版 |
2024/03/05(火) 22:32:50.00ID:IYPmJEac
>>80
そう思ってるの君だけだから
>ちなみに二つの封筒問題は純粋に数学の問題なので
0083132人目の素数さん
垢版 |
2024/03/05(火) 22:34:36.90ID:GpIsjTrm
>>82
じゃ何の問題?
0084132人目の素数さん
垢版 |
2024/03/05(火) 22:35:20.82ID:GpIsjTrm
>>81
だから失せなって
君以外の誰もモデリングの話なんてしてないから
0085132人目の素数さん
垢版 |
2024/03/05(火) 22:36:15.26ID:IYPmJEac
>>83
封筒の問題はモデリングを適当にやると変な答がでるよって話だよ
箱入り無数目も同じだろ
0086132人目の素数さん
垢版 |
2024/03/05(火) 22:37:57.69ID:IYPmJEac
>>84
君以外はみんな箱の中身を確率変数でモデル化するとどうなるかの話してるんですけどー
0087132人目の素数さん
垢版 |
2024/03/05(火) 22:43:32.36ID:GpIsjTrm
>>85 >>86
君往生際悪いよ
「試行毎に変化するものが確率変数」の正否を論じてるのに試行の無いモデルを持ち出す馬鹿が居ても無意味だから失せなよ
0088132人目の素数さん
垢版 |
2024/03/05(火) 22:45:26.82ID:GpIsjTrm
0078132人目の素数さん
2024/03/05(火) 22:23:13.76ID:GpIsjTrm
>>77
>試行なんてねーよ


「試行毎に変化するものが確率変数」の正否を論じてるのにこの答えは馬鹿としか言い様がありません
0089132人目の素数さん
垢版 |
2024/03/05(火) 22:46:39.06ID:IYPmJEac
>>87
モデルに試行がないんじゃねーよ
現実の問題側に試行がないの
モデルにはΩからωがランダムに選ばれるという試行がある
0090132人目の素数さん
垢版 |
2024/03/05(火) 22:47:57.15ID:GpIsjTrm
試行が存在しないモデルを持ち出してどうやって「試行毎に変化するものが確率変数」の正否を論じるつもりだったのでしょう?
馬鹿のやることは理解できましぇーん
0091132人目の素数さん
垢版 |
2024/03/05(火) 22:54:15.82ID:GpIsjTrm
>>89
なに後出しで言ってんの?
おまえそんなこと一言も言ってなかったじゃん
指摘が刺さったので取り繕ったな?w

で?>>76に答える気あるの?>>77>>89も答えになってないよ?
0093132人目の素数さん
垢版 |
2024/03/05(火) 22:56:30.53ID:GpIsjTrm
言っとくけど書き散らすんじゃなく、1レスで完全な答えを書けよ?
書き散らしは採点せんぞ?
0094132人目の素数さん
垢版 |
2024/03/05(火) 22:56:52.38ID:GpIsjTrm
>>92
はい、落第
0097132人目の素数さん
垢版 |
2024/03/05(火) 22:59:44.50ID:IYPmJEac
そもそも確率空間に落とし込んだ時点で試行なんて言葉に意味なんてねーぞ
常にΩからωを選ぶって一回の試行になるんだからな
0098132人目の素数さん
垢版 |
2024/03/05(火) 23:10:23.44ID:GpIsjTrm
>>97
おまえΩが何か書いてないじゃん 頭イカレテる?
いいから落第者は去りましょう 往生際悪いよ
0100132人目の素数さん
垢版 |
2024/03/05(火) 23:33:22.80ID:GpIsjTrm
>>99
誰もおまえの持論を聞いてない
0102132人目の素数さん
垢版 |
2024/03/05(火) 23:40:26.44ID:GpIsjTrm
>>101
何を聞いても屁理屈しか言えないおまえに用は無い 失せろ
0104132人目の素数さん
垢版 |
2024/03/05(火) 23:49:57.24ID:GpIsjTrm
おまえから絡んできたんだろ
0106132人目の素数さん
垢版 |
2024/03/06(水) 00:07:25.52ID:UPLSLbzu
>>27 補足

 >>4より
http://www.ma.huji.ac.il/hart/
Sergiu Hart
http://www.ma.huji.ac.il/hart/#puzzle
Some nice puzzles:
http://www.ma.huji.ac.il/hart/puzzle/choice.pdf?
Choice Games November 4, 2013
P2
Remark. When the number of boxes is finite Player 1 can guarantee a win
with probability 1 in game1, and with probability 9/10 in game2, by choosing
the xi independently and uniformly on [0, 1] and {0, 1,..., 9}, respectively.
<google訳>
Remark. 箱が有限個の場合、プレイヤー 1 は勝利を保証できます。
ゲーム 1 では確率 1、ゲーム 2 では確率 9/10 で、
xi はそれぞれ [0, 1] と {0, 1,..., 9} で独立かつ一様で
(引用終り)

さて
1)Sergiu Hart氏は、有限個の場合は、従来の確率論通りだという
 つまり、区間[0, 1]の任意実数では確率0 (=Player 2の勝率)
 {0, 1,..., 9} では、確率 1/10 (=Player 2の勝率)
 ってこと
 この場合、箱の中の数は、従来の確率論通り 確率変数である
2)可算無限個の場合は?
 仮に百歩ゆずって、有限個の箱が確率1-εで当てられるとしても
 当たる箱以外に、やはり可算無限個の箱がある
 それは、上記Sergiu Hart氏の理論通りです
 すなわち、上記『プレイヤー 1 は勝利を保証できます
 ゲーム 1 では確率 1、ゲーム 2 では確率 9/10 で
 xi はそれぞれ [0, 1] と {0, 1,..., 9} で独立かつ一様で』の通りで
 この場合、箱の中の数は、従来の確率論通り 確率変数である

よって、『箱の中の数は、従来の確率論通り 確率変数である』! が結論です
0107132人目の素数さん
垢版 |
2024/03/06(水) 00:09:11.58ID:hk/0+lKf
>>105
いいから失せろ
0108132人目の素数さん
垢版 |
2024/03/06(水) 00:45:02.38ID:S5Dm0o31
今日はこれでご飯食べようかな

ここで、発端になったレスを思い出してみよう
こいつがなんでこんなに必死になってるかって言うと、コロナの例が>837を完全に否定してるからなのですね
コロナに感染してるかどうかなんて最初にすでに決まっていて突然変化するわけないからね
1例挙げれば十分な命題だったでしょ

837 132人目の素数さん 2024/02/23(金) 02:14:00.13 ID:xKynRG52
>833
>最初に決めるから確率変数じゃないとかね
君は文盲かい?
試行毎に変化するものが確率変数 変化しないものは確率変数でない
と書いたんだけど読めないかい? なら小学校の国語からやり直そうね

838 132人目の素数さん sage 2024/02/23(金) 02:16:11.67 ID:0hiCCwLy
>837
それがおかしいって言ってんだよ
見えないものが確率変数ね
0109132人目の素数さん
垢版 |
2024/03/06(水) 00:49:18.62ID:hk/0+lKf
>>108
これでおかわりすれば?

ここで、発端になったレスを思い出してみよう
こいつがなんでこんなに必死になってるかって言うと、二つの封筒問題の例が>838を完全に否定してるからなのですね
相手の封筒の中身が見えないからといって確率変数としてしまうとパラドックスになるからね
1例挙げれば十分な命題だったでしょ

838 132人目の素数さん sage 2024/02/23(金) 02:16:11.67 ID:0hiCCwLy
>837
それがおかしいって言ってんだよ
見えないものが確率変数ね
0111132人目の素数さん
垢版 |
2024/03/06(水) 00:56:38.66ID:hk/0+lKf
>>110
自分で勉強して下さい
0113132人目の素数さん
垢版 |
2024/03/06(水) 01:07:00.56ID:hk/0+lKf
>>112
理解できなかったんだね
0114132人目の素数さん
垢版 |
2024/03/06(水) 01:09:49.49ID:S5Dm0o31
封筒の中身を確率変数にすることの一番のポイントは、中身を定数にしたモデルの拡張になっていること
すなわち、定数のモデルから結論できることはすべて確率モデルでも正しい
これにより、定数によるモデル化よりも劣ってる点が全くない
0116132人目の素数さん
垢版 |
2024/03/06(水) 01:14:23.87ID:hk/0+lKf
>>114
やはり理解できてない
0118弥勒菩薩
垢版 |
2024/03/06(水) 02:18:41.93ID:1ZltP1Y1
>>26
箱入り無数目で目の値を二値、自然数数値にした時の尻尾同値類のボレル可測性はVitali集合のボレル可測性と同等である
飯ウマー
0119132人目の素数さん
垢版 |
2024/03/06(水) 05:53:53.58ID:IDPoig8I
>>106
>>46読んだかい?
決定番号が正則分布(幾何分布)になる無限列の分布を与えてるよ
https://ja.wikipedia.org/wiki/%E5%B9%BE%E4%BD%95%E5%88%86%E5%B8%83
しかも1さんがドヤってた確率過程(ベルヌーイ過程)を使ってね
https://ja.wikipedia.org/wiki/%E3%83%99%E3%83%AB%E3%83%8C%E3%83%BC%E3%82%A4%E9%81%8E%E7%A8%8B
いやぁ、滑稽滑稽
どうした?確率論
どうした?確率過程

まあ、決定番号が幾何分布になると分かっていれば
別に箱入り無数目戦略とらなくても
もっといい方法があるけどな
ただ箱入り無数目も100列の場合、確率99/100成功するし、計算できる

残念だったな 1さん 完全に終わったよ あなたの完全敗北 ご愁傷さま
さすが正則行列も可積分条件も無限集合も分からんド素人
幾何分布もベルヌーイ過程も知りませんでしたぁ!

ギャハハハハハハ!!!
0120132人目の素数さん
垢版 |
2024/03/06(水) 05:57:10.18ID:IDPoig8I
>>118 その”知識”だけでは意味がない 釈迦如来
0121弥勒菩薩
垢版 |
2024/03/06(水) 06:45:16.48ID:1ZltP1Y1
ド素人は気にするな
0122132人目の素数さん
垢版 |
2024/03/06(水) 07:03:58.57ID:IDPoig8I
>>121
似非玄人、>>46に何も反論出来ず
ヒャッハー!
0123弥勒菩薩
垢版 |
2024/03/06(水) 10:53:42.72ID:1ZltP1Y1
設定不明の問いのには答えようがない、ド素人め
0124132人目の素数さん
垢版 |
2024/03/06(水) 11:05:26.02ID:GkBPE511
>>122
ご苦労様です、スレ主です
 >>46って、0さんじゃない?w ;p)

>>188
>箱入り無数目で目の値を二値、自然数数値にした時の尻尾同値類のボレル可測性はVitali集合のボレル可測性と同等である

弥勒菩薩さま、救いのお言葉 ありがとうございます。
弥勒菩薩さまのお言葉を、私なりに解釈してみました

1)記号を用意しよう
 下記ヴィタリ集合 R/Q で、有理数Qの代わりに m進展開の有限小数の集合Umを使う
 Umは、Qとほとんど類似だが、巡回小数を含まない。なのでN⊂Um⊂Q、ゆえにUmは可算無限集合
 m=10なら10進展開で
 m=2では 2進展開で二値を使っている
 1/3は、10進小数では巡回小数だが、3進小数では0.1と有限小数になる(mの取り方に依存する)
2)いま、商集合 R/Umを考える。下記のヴィタリ集合と同じ論法で、選択公理を使って R/Umの代表から非可測集合を作ることができる(詳細略す)
 これを区間[0,1]に限定した集合をV(Um)とする
 補足すると、無理数 r1,r2∈R r1-r2∈V(Um)のとき、r1-r2は有限小数で よってr1とr2は 小数の第n+1から先しっぽが一致している
 つまり、v=r1-r2 とすると v=0.v1 v2 v3 ・・vn 0 0 0 ・・・と書ける(ここにvnは小数第n位の数 vn≠0)
3)箱入り無数目>>1との関係で、箱に区間[0,1]の実数のm進展開の無限小数を入れるとする
 先頭から、m1,m2,・・,mi,・・ となる ここに miは 0〜m-1の整数
 つまり、箱入り無数目のしっぽ同値類は、R/Umと対応づけができる
 m1,m2,・・,mi,・・ → 無限小数 s=0.m1 m2 ・・ mi ・・ s∈Um
 代表番号dは、あるm進数列で 代表r∈Umに対して s-r==0.m'1 m'2 ・・ m'd-1,0 0 0・・(小数d位以降は0)となること
4)繰り返すが、上記 m=2で2進展開で二値になり V(U2)が ヴィタリ集合と同様に 非可測集合を成す

弥勒菩薩さま、ありがとうございます!
迷える子羊に救いあれ! アーメン!

(参考)
https://ja.wikipedia.org/wiki/%E3%83%B4%E3%82%A3%E3%82%BF%E3%83%AA%E9%9B%86%E5%90%88
ヴィタリ集合(Vitali set)

https://alg-d.com/math/ac/lebesgue.html
トップ > 数学 > 選択公理 > Lebesgue非可測集合の存在
2011年10月12日 壱大整域
0125132人目の素数さん
垢版 |
2024/03/06(水) 11:08:35.55ID:GkBPE511
>>122 タイポ訂正

 Umは、Qとほとんど類似だが、巡回小数を含まない。
  ↓
 Umは、Qとほとんど類似だが、循環小数を含まない。

 1/3は、10進小数では巡回小数だが、3進小数では0.1と有限小数になる
  ↓
 1/3は、10進小数では循環小数だが、3進小数では0.1と有限小数になる
0126弥勒菩薩
垢版 |
2024/03/06(水) 11:10:46.59ID:1ZltP1Y1
0は以前おっちゃんになりすましたアホだろ
0127132人目の素数さん
垢版 |
2024/03/06(水) 11:11:09.48ID:GkBPE511
>>122 タイポ訂正追加

 代表番号dは、あるm進数列で 代表r∈Umに対して s-r==0.m'1 m'2 ・・ m'd-1,0 0 0・・(小数d位以降は0)となること
  ↓
 代表番号dは、あるm進数列で 代表r∈Umに対して s-r = 0.m'1 m'2 ・・ m'd-1,0 0 0・・(小数d位以降は0)となること
0129132人目の素数さん
垢版 |
2024/03/06(水) 11:24:59.00ID:IjGHMB8G
>>124
> 46って、0さんじゃない?
誰かは知らんが、>>46は決定番号の分布が正則になるように設定されている
>>119はそのポイントを見事にいい当てている

1が「ロジックがしっかりしている」と誉めた人物と
1が「こいつはサイコパス」と貶した人物が同一
だとするとまあジキルとハイドなわけですが・・・

閑話休題

>>188よりは>>46のほうが、1にとっては理解しやすいし有意義だと思うが
弥勒菩薩はまだ地上に出てくるのが5億年ほど早かったんじゃないかと・・・
0130132人目の素数さん
垢版 |
2024/03/06(水) 11:28:25.90ID:IjGHMB8G
>>124
>迷える子羊に救いあれ!

0がいうように、「無情報事前条件」なんていう
「エーテル」に固執するのをやめれば
悟りが開けるのではないかね 知らんけど
0131弥勒菩薩
垢版 |
2024/03/06(水) 11:36:36.38ID:1ZltP1Y1
ポーランド空間に反応できない確率専攻もどうかね
0132弥勒菩薩
垢版 |
2024/03/06(水) 11:47:09.87ID:1ZltP1Y1
最初にいったろ、こいつらド素人屁理屈上手相手に議論してもどうもならんと
0133132人目の素数さん
垢版 |
2024/03/06(水) 11:55:15.51ID:RxDn7n9W
ポーランド空間って言葉を振り回すだけの似非玄人のド素人もどうかね
弥勒菩薩?明王、天にも至らんよ
0134弥勒菩薩
垢版 |
2024/03/06(水) 12:00:08.36ID:1ZltP1Y1
サイコロ投げが分からない奴に言われてもなー
0135132人目の素数さん
垢版 |
2024/03/06(水) 12:10:16.79ID:hk/0+lKf
サイコロ投げが分かってるのは自分だけと思いたい妄想菩薩
0136132人目の素数さん
垢版 |
2024/03/06(水) 12:27:40.52ID:RxDn7n9W
弥勒菩薩は、1云うところの「鳥なき里の蝙蝠」

https://ja.wiktionary.org/wiki/%E9%B3%A5%E3%81%AA%E3%81%8D%E9%87%8C%E3%81%AE%E8%9D%99%E8%9D%A0
鳥がいないところでは、ただ飛べるというだけでコウモリが偉そうにする、あるいは偉そうに見えることから、
ある分野に関して、本当に優れた人がいないところでは、ちょっとその分野に知識等があるだけで、
その道の権威然とすることのたとえ。「鳥なき島の蝙蝠」とも。
0138弥勒菩薩
垢版 |
2024/03/06(水) 12:55:37.06ID:1ZltP1Y1
ド素人の遠吠え
0139132人目の素数さん
垢版 |
2024/03/06(水) 13:02:54.19ID:l+rk+Cj9
>>138 そういうあなたの専門、何?
0140弥勒菩薩
垢版 |
2024/03/06(水) 14:33:50.46ID:1ZltP1Y1
選択公理、選択公理と叫ぶんでこれ買っただけど全然読んでない

選択公理と数学 田中
§15位相数学と選択公理
ベール性とボレル集合について書いてある(記述集合論)
§28決定性公理
無限ゲームについて書いてある。箱入り無数の目もこういう形で定式化して議論すべきじゃないのか。
0141132人目の素数さん
垢版 |
2024/03/06(水) 14:45:36.18ID:yBkM/z01
>>140
>§28決定性公理
>無限ゲームについて書いてある。
>箱入り無数の目もこういう形で定式化して議論すべきじゃないのか。
素人わけもわからずイキる

ミロク 2*歳 
今、人生の絶頂期
この後、転落が待っているとは
その時、知る由もなかった
0142弥勒菩薩
垢版 |
2024/03/06(水) 15:14:22.74ID:1ZltP1Y1
ド素人同士論破ゲームを続けて
0143132人目の素数さん
垢版 |
2024/03/06(水) 15:43:20.40ID:GkBPE511
ポーランド空間か

https://ja.wikipedia.org/wiki/%E3%83%9D%E3%83%BC%E3%83%A9%E3%83%B3%E3%83%89%E7%A9%BA%E9%96%93
ポーランド空間
ポーランド空間(ポーランドくうかん)とは、可分で完備距離づけ可能な位相空間のことである。すなわち、可算な稠密部分集合をもつ完備距離空間と同相な空間のことである。名前の由来は、この空間が著名なポーランド人研究者たち(例えば、ヴァツワフ・シェルピニスキ、カジミェシュ・クラトフスキ、アルフレト・タルスキなど)によって研究され始めたことによる。今日では、Borel equivalence relationなどの研究を含んだ記述集合論の研究のための基礎としても重要視されている。

普通の距離づけでは完備でないがポーランド空間ではあるようなものも存在する。例えば、開区間 (0, 1) はポーランド空間である。

いかなる2つの不可算なポーランド空間の間にも、ボレル同型写像が存在する。すなわち、全単射でボレル構造を保つものが存在する。特に、不可算なポーランド空間の濃度は必ず連続体濃度となる。
0144132人目の素数さん
垢版 |
2024/03/06(水) 16:16:39.10ID:X//IEIZJ
弥勒菩薩は何故サイコロが正6面体の立方体だと
サイコロを平面上で投げたとき1から6の目が等確率で出ることになるのか
の理由でも考えていればよろしい
これにはれっきとした理由がある
0145弥勒菩薩
垢版 |
2024/03/06(水) 16:18:54.21ID:1ZltP1Y1
蛇足
俺は箱入り無数目に勝つ戦略がないとはいっていない、時枝記戦略の間違いを指摘しただけ
0147132人目の素数さん
垢版 |
2024/03/06(水) 16:35:32.38ID:GkBPE511
>>124 補足
> 下記ヴィタリ集合 R/Q で、有理数Qの代わりに m進展開の有限小数の集合Umを使う
>2)いま、商集合 R/Umを考える。下記のヴィタリ集合と同じ論法で、選択公理を使って R/Umの代表から非可測集合を作ることができる(詳細略す)

1)これは、いまふと考えると、下記のSergiu Hart Choice Gamesのgame2
 区間[0,1]の有理数の10進展開の各桁の数字を使う例の類似になっていることに気づいた
2)つまり、Sergiu Hart Choice Gamesのgame2 では、区間[0,1]のQに対して
 商集合 Q/U10 を使っている
 この場合、Qが可算だからQ/U10も当然可算で (だからフルパワー選択公理でなく、可算選択公理で間に合う)
 代表の集合もまた可算で、よって、代表の集合を区間[0,1]にとると、零集合(ルベーグ測度0)になる

Q/U10、R/U10(R/Um)いずれにせよ
確率計算に使える集合ではなさそう(全事象Ω に対して 1を与えられない)

(参考)
 >>4より
http://www.ma.huji.ac.il/hart/
Sergiu Hart
http://www.ma.huji.ac.il/hart/#puzzle
Some nice puzzles:
http://www.ma.huji.ac.il/hart/puzzle/choice.pdf?
Choice Games November 4, 2013

P2
A similar result,but now without using the Axiom of Choice.2
Consider the following two-person game game2:
・Player1 chooses a rational number in the interval [0,1] and writes down its infinite decimal expansion 0.x1x2...xn..., with all xn∈{0,1,...,9}.
・Player2 asks (in some order) what are the digits xn except one, say xi;
 then he writes down a digitξ∈{0,1,...,9}.
・ If xi=ξ then Player2 wins,and if xi=ξ then Player1 wins.
 By choosing i arbitrarily and ξ uniformly in {0,1,...,9},  Player2 can guarantee a win with probability 1/10.
 However, we have:
Theorem 2 For every ε>0 Player2 has a mixed strategy in game2 guaranteeing him a win with probability at least
1−ε.

https://wiis.info/math/measure/lebesgue-measure/null-set/
wiis
零集合/ルベーグ測度
外測度の値がゼロであるような集合を零集合と呼びます。零集合はルベーグ可測です。零集合の基本的な性質について解説します。関連して「ほとんどいたるところ」という用語の意味を解説します。
0149132人目の素数さん
垢版 |
2024/03/06(水) 16:45:07.91ID:GkBPE511
>>144
>弥勒菩薩は何故サイコロが正6面体の立方体だと
>サイコロを平面上で投げたとき1から6の目が等確率で出ることになるのか
>の理由でも考えていればよろしい
>これにはれっきとした理由がある

ご苦労様です、スレ主です
1)イカサマサイコロあるよ (なので、理由は「ちゃんとしたサイコロ」ってことですね)
2)1/6→1/n に一般化を考えると、鉛筆ころがしが適している
3)6面鉛筆→n面鉛筆 を考えればいい

(参考)
https://www.youtube.com/watch?v=5Tpbdaj62yI
【悪用禁止】ギャンブルでの使用は絶対ダメです。イカサマサイコロの作り方【種明かし】【手品】【マジック】
ユジックの手品教室
2022/06/14
@contactMiu
1 年前
任意の目を出せるサイコロは目からウロコでした!

https://gigazine.net/news/20100916_cheating_dice/
gigazine
2010年09月16日 15時44分動画
イカサマ用サイコロの簡単な作り方
古今東西ありとあらゆるテクニックを駆使してギャンブル・賭博などで思い通りの目を出す、あるいは任意の目が出やすい傾向にあるサイコロというのが作り出されてきましたが、そういったサイコロはすべて割と高度な制作技術が必要なケースばかりでした。
というわけで、もう少し簡単にイカサマ用のサイコロを作り出せないか?というのがこのハウツーの中身です。あくまでも個人で楽しむためのレベルです。うまくいくかどうかはあなた次第。
0150132人目の素数さん
垢版 |
2024/03/06(水) 16:48:24.89ID:BLtcDL0g
>>147 
>全事象Ω に対して 1を与えられない
一様性を求めないなら、全事象に対して1を与えられる
実際>>46はそれを実現している 
1君が幾何分布もベルヌーイ試行、ベルヌーイ過程も分かってないから理解できないだけ
0151132人目の素数さん
垢版 |
2024/03/06(水) 17:04:33.15ID:X//IEIZJ
>>149
正6面体の立方体が正12面体や正20面体に変わると、
サイコロを平面上で投げたとき、サイコロが平面上で転がり易くなり、
1から12の目(または1から20の目)の中に出易くなる傾向が生じる目が幾つかある
故に、サイコロを平面上で投げたときのサイコロの出る目に関する事象が
サイコロが平面上で転がったときに出る目の事象に変わる
故に、サイコロを平面上で投げたとき、1から12の目(または1から20の目)
が等確率で出るとはいえなくなる
正6面体のサイコロが正4面体に変わると、
サイコロを平面上で投げたとき、正4面体は正6面体より転がりにくく、
1から4の目の中に出易くなる傾向の目が1つに絞られ易くなる
故に、サイコロを平面上で投げたときのサイコロの出る目に関する事象が
サイコロが平面上で転がったときに出る目の事象に変わる
故に、サイコロを平面上で投げたとき、1から4の目が等確率で出るとはいえなくなる
正6面体のサイコロが正8面体に変わっても、正6面体の隣り合う面の角度は90度で唯1つ、
正8面体の隣り合う面の角度は2つあるから、同様なことがいえる
よって、サイコロを平面上で投げたときサイコロの目が等確率で出るのは
立方体の1から6の目のサイコロだけ
0152132人目の素数さん
垢版 |
2024/03/06(水) 17:05:44.48ID:GkBPE511
>>140
>選択公理と数学 田中
>§15位相数学と選択公理
>ベール性とボレル集合について書いてある(記述集合論)
>§28決定性公理
>無限ゲームについて書いてある。箱入り無数の目もこういう形で定式化して議論すべきじゃないのか。

弥勒菩薩さま、スレ主です
そこ同意です
時枝先生がね
数学セミナー誌に投稿する記事のクォリティーとして
当然そうあるべきです
0153132人目の素数さん
垢版 |
2024/03/06(水) 17:12:56.88ID:GkBPE511
>>150
>>全事象Ω に対して 1を与えられない
>一様性を求めないなら、全事象に対して1を与えられる
>実際>>46はそれを実現している

・回答者が、全事象Ωをいじくったら まずい
・題意外しですよ(下記)
・題意外しは、院試では0点です

(参考)時枝記事>>1より
https://imgur.com/a/8bqlb08
数学セミナー201511月号「箱入り無数目」
https://rio2016.5ch.net/test/read.cgi/math/1620904362/401-406
純粋・応用数学(含むガロア理論)8 より
1.時枝問題(数学セミナー201511月号の記事)の最初の設定はこうだった。
「箱がたくさん,可算無限個ある.箱それぞれに,私が実数を入れる.
どんな実数を入れるかはまったく自由,例えばn番目の箱にe^nを入れてもよいし,すべての箱にπを入れてもよい.
もちろんでたらめだって構わない.そして箱をみな閉じる.
今度はあなたの番である.片端から箱を開けてゆき中の実数を覗いてよいが,一つの箱は開けずに閉じたまま残さねばならぬとしよう.
どの箱を閉じたまま残すかはあなたが決めうる.
勝負のルールはこうだ. もし閉じた箱の中の実数をピタリと言い当てたら,あなたの勝ち. さもなくば負け.
勝つ戦略はあるでしょうか?」
0154132人目の素数さん
垢版 |
2024/03/06(水) 17:28:07.21ID:X//IEIZJ
>>153
>・回答者が、全事象Ωをいじくったら まずい
全事象Ωが起こる確率はコルモゴロフの公理から P(Ω)=1 である
0155弥勒菩薩
垢版 |
2024/03/06(水) 17:35:32.02ID:1ZltP1Y1
独自に戦略を考えればいいだけだろ
メンヘルババア戦略
ウマシカ戦略
成りすまし戦略
0156弥勒菩薩
垢版 |
2024/03/06(水) 17:36:52.63ID:1ZltP1Y1
無限帽子の人は独自に答えをだしていたぞ
0157132人目の素数さん
垢版 |
2024/03/06(水) 17:52:26.46ID:X//IEIZJ
>>155
弥勒は一々ポーランド空間をサイコロ投げの出た目に関する事象の確率に適用するのか
サイコロ投げの投げの事象にはポーランド空間なんていらん
0158132人目の素数さん
垢版 |
2024/03/06(水) 17:59:15.68ID:GkBPE511
>>154
>>・回答者が、全事象Ωをいじくったら まずい
>全事象Ωが起こる確率はコルモゴロフの公理から P(Ω)=1 である

常識のない人がいる
・世に、確率の公理を満たせないケースがある
・その一例が、下記の非正則分布です(「箱入り無数目」は」、これです)
・他にも、思わず知らず 非可測集合を使ってしまっている場合とか

 >>7より
(参考)
https://ai-trend.jp/basic-study/bayes/improper_prior/
AVILEN Inc. 2020
2020/04/14
非正則事前分布とは?〜完全なる無情報事前分布〜
ライター:古澤嘉啓
目次
1 非正則な分布とは?一様分布との比較
2 非正則分布は確率分布ではない!?
3 非正則事前分布は完全なる無情報事前分布
4 まとめ
(抜粋)
非正則な分布とは?一様分布との比較
非正則な分布は一様分布と非常に似ています。では、一様分布とどのように似ていて、どこが違うのでしょうか?
非正則分布は確率分布ではない!?
上で説明した非正則な分布ですが、よく見てみてください。確率の和が1ではありませんよね。
これを数式で表現してみましょう。事前分布をパラメータの取りうる区間で積分すると、
積分値が無限大に発散してしまいます。これは、全事象の確率は1であるというコルモゴロフの確率の公理に反しています。
よって、厳密には、非正則な分布は確率密度関数ではありません。なぜなら、確率の公理を満たしていないからです。
それでもこの分布が使われる理由は、この分布には特有の特徴があり、それが事前分布として機能する上でとても有用だからです。ではどのように有用なのでしょうか?
0159132人目の素数さん
垢版 |
2024/03/06(水) 18:08:19.49ID:X//IEIZJ
>>158
>・世に、確率の公理を満たせないケースがある
>・その一例が、下記の非正則分布です(「箱入り無数目」は」、これです)
箱入り無数目は非正則分布ではなく、同値類と選択公理がメインの問題で
確率は初歩的なことに過ぎない
箱入り無数目で確率論の確率測度を使いたいなら、
コルモゴロフの公理を満たすように全事象Ωが起きる確率を P(Ω)=1 とする
0160132人目の素数さん
垢版 |
2024/03/06(水) 18:08:53.39ID:GkBPE511
>>158 タイポ訂正と補足

・その一例が、下記の非正則分布です(「箱入り無数目」は」、これです)
 ↓
・その一例が、下記の非正則分布です(「箱入り無数目」は、これです)

<補足>
・総和ないし積分値が無限大に発散してしまうということです
(「箱入り無数目」は、一様分布とは似ても似つかない分布ですが、裾が減衰しないのは同じです)
・なお、総和ないし積分値が無限大に発散しないためには
 総和ないし積分で、分布の裾が1/xより早く減衰する必要ありです(1/x^ε で ε>1の必要あり)
 積分∫x=1〜∞ 1/x dx →∞
 挿話 Σ n=1〜∞ 1/n →∞
 となります。これは学部1年の数学からの必然の帰結です
0161132人目の素数さん
垢版 |
2024/03/06(水) 18:26:41.38ID:S5Dm0o31
P(Ω)=1なんて全く関係なくて、決定番号が確率変数かと思ってたけど、確率変数と仮定して像測度計算したら確率測度になってなかったって話だろ
ルベーグ非可測関数の存在証明とやってることは同じ
0162132人目の素数さん
垢版 |
2024/03/06(水) 18:28:07.71ID:X//IEIZJ
>>160
非正則分布は数学的裏付けがなされていない分布だから、確率分布としては扱わない
0163132人目の素数さん
垢版 |
2024/03/06(水) 18:34:03.69ID:X//IEIZJ
>>161
>決定番号が確率変数かと思ってたけど、
>確率変数と仮定して像測度計算したら確率測度になってなかったって話だろ
何のことか知らんから、そのことは今までやっていた人達に聞いてくれ
0166132人目の素数さん
垢版 |
2024/03/06(水) 18:52:31.25ID:hk/0+lKf
>>145
どこがどう間違ってると?
0167132人目の素数さん
垢版 |
2024/03/06(水) 18:54:16.58ID:S5Dm0o31
計算したい確率は
Xを解答者の答の確率変数
Yを正解の確率変数
Fをすでに開けた箱の中身からなるσ-alg
としたら
P(X=Y|F)
だけど、これは普通に計算したら0になる。
でも、最大の決定番号を持つ列の番号をKとして、Kで場合分けした計算は
P(X=Y|F)=Σ_k P(X=Y|F,K=k)P(K=k)
≧99/100
になるんだから、Kが確率変数なのがおかしいってことだろ
0168132人目の素数さん
垢版 |
2024/03/06(水) 18:54:44.27ID:hk/0+lKf
>>147
>Q/U10、R/U10(R/Um)いずれにせよ
>確率計算に使える集合ではなさそう(全事象Ω に対して 1を与えられない)
箱入り無数目とは何の関係も無い
箱入り無数目の標本空間は{1,2,・・・,100}だから
0170132人目の素数さん
垢版 |
2024/03/06(水) 19:05:25.99ID:hk/0+lKf
>>158
>・その一例が、下記の非正則分布です(「箱入り無数目」は」、これです)
嘘はダメ
箱入り無数目で用いられる分布は離散一様分布だけ
「さて, 1〜100 のいずれかをランダムに選ぶ. 例えばkが選ばれたとせよ. s^kの決定番号が他の列の決定番号どれよりも大きい確率は1/100に過ぎない.」

どうして息するように嘘つくのですか? あなたはサイコパスですか?
0171132人目の素数さん
垢版 |
2024/03/06(水) 19:13:21.91ID:IDPoig8I
>>153
>回答者が、全事象Ωをいじくったら まずい
>題意外しですよ 院試では0点です

「どんな実数を入れるかはまったく自由」
って書いてありますよ
どんな分布を考えるかも自由ですね

>>158
>常識のない人がいる
>確率の公理を満たせないケースがある
>その一例が、下記の非正則分布です(「箱入り無数目」は」、これです)
>他にも、思わず知らず 非可測集合を使ってしまっている場合とか

ルベーグ測度に固執するから、
非可測集合ダー、非正則分布ダー、と騒ぐ
別の測度を考えれば
可測集合になるし、正則分布になる

しかし、確率論ガー、確率過程ガー、と喚いてるくせに
基本中の基本である、ベルヌーイ試行、ベルヌーイ過程すら
扱えないってのは最低限の常識すらないド素人ですなー

>>160
>なお、総和ないし積分値が無限大に発散しないためには
>総和ないし積分で、分布の裾が1/xより早く減衰する必要ありです
>(1/x^ε で ε>1の必要あり)
> 積分∫x=1〜∞ 1/x dx →∞
> 挿話 Σ n=1〜∞ 1/n →∞
> となります。これは学部1年の数学からの必然の帰結です

指数関数的に減衰すれば問題ないですね
>>46はそうなってます

頭使えよ ア・タ・マ

ギャハハハハハハ!!!
0172132人目の素数さん
垢版 |
2024/03/06(水) 19:22:19.62ID:IDPoig8I
>>46では
代表の集合 A 確率ε
決定番号2の集合 R✕A-A 確率ε(1-ε)
決定番号3の集合 R^2✕A-R✕A 確率ε(1-ε)^2
・・・
決定番号nの集合 R^(n-1)✕A-R✕(n-2)✕A 確率ε(1-ε)^(n-1)
・・・
となってるから正則

まあ、R^Nの任意の尻尾同値類の代表元Aを考えるのが嫌なら
その部分集合で具体的に構成なものに制限してもいいよ

要は「非可測ガー」「非正則ガー」とかいう
🐎🦌な言いがかりをシャットアウトすればいいだけ
0173132人目の素数さん
垢版 |
2024/03/06(水) 19:28:20.76ID:IDPoig8I
実際に出題者の出題がどんな分布してるかなんてわかりようがない
だからあるべき「無情報事前分布」なんてあるわけないのである

列を100本に分けてその列の決定番号をみるのだから
>>46のような想定は別に不自然でもなんでもない
実現不能、計算不能な事前分布を考えて、
出来ないというのは🐎🦌というか”薄知”(←当て字)である
出来るようにすればいいだけであって、
そうすれば積分計算で99/100が導ける
0174132人目の素数さん
垢版 |
2024/03/06(水) 19:38:10.89ID:S5Dm0o31
>>173
攻略法があるんならどんな分布だろうが攻略できるはずだろ、任意の分布についての形で定式化して考察するのが一番自然
0175132人目の素数さん
垢版 |
2024/03/06(水) 20:03:23.86ID:IDPoig8I
>>174
>攻略法があるんならどんな分布だろうが攻略できるはずだろ

「攻略」という言い方がナイーブ

100列のどれを選んでも、必ず当てられるなんて、記事では言ってない
当てられない列が存在しないこともあるが、たいていは1列ある ただし2列以上はない

このことは確率論とは全く無関係に、自然数の順序から初等的に証明できるが
問題は、確率が99/100だという点である

これは正則分布ならもちろん証明できるが、そうでない場合はそもそも計算できない

>任意の分布についての形で定式化して考察するのが一番自然

素人はこういうナイーブなことを平気でいうが
積分をちょっとでも知ってる人ならこんな大胆なことは決して言わない
そんなの無理だから 例えば積分の順序交換なんていつでもできるわけではない
0176132人目の素数さん
垢版 |
2024/03/06(水) 20:15:05.45ID:IDPoig8I
2つの封筒でも箱入り無数目でも
一様分布を「無情報事前分布」だと言い張って当てはめると
大体おかしなことになる

箱入り無数目の場合、>>46のような分布を考えた上で
記事の戦略で当たる場合を考えると、例外なく
出題者が箱の中に勝手に入れた数以外のものだと分かる
(記事の文章だと全ての箱に出題者が数を入れてるが、
46の場合には、あくまで代表列は下敷きであるし、
選択関数から丸わかりなので、出題者が意図する
ランダムネスに全く関係がない)

箱入り無数目を読めば読むほど
よっぽど運が悪くなければはずれっこない
と思うようになる
(出題者が任意に数を入れられる箱はたかだか有限個だから)
0178132人目の素数さん
垢版 |
2024/03/06(水) 20:19:37.05ID:IDPoig8I
>>177
>任意の分布でやったら壊れる
「壊れる」という言葉で何をいいたいのかわからないが
正則分布のとき成り立つことを、
正則でない分布でやろうとしたら
うまくいかないのは当然のことであって
だから間違ってるというのはおかしなことである
0179132人目の素数さん
垢版 |
2024/03/06(水) 20:22:16.26ID:IDPoig8I
2つの封筒で、2人ともが「交換したら儲かる」という計算結果になるなら間違ってる
箱入り無数目で、100人とも「自分がはずれる」という計算結果になるなら間違ってる
要するにそういうこと
正則分布でない分布を使ってそういう結果が得られるのなら
その理由はそんなおかしな分布を使ったせいであるとしかいいようがない
0180132人目の素数さん
垢版 |
2024/03/06(水) 20:24:25.47ID:S5Dm0o31
>>178
何言ってんのかわからん
確率空間のPをいかように取ってもいいって話をしてるつもりなんだが…
もちろん確率測度の範囲で
0181132人目の素数さん
垢版 |
2024/03/06(水) 20:28:25.11ID:IDPoig8I
>>180
>確率空間のPをいかように取ってもいい
いつ誰がどこでそんなことが正しいといったんですか?
今ここであなたが何の根拠もなくナイーブにそういってるだけですよね?
0182132人目の素数さん
垢版 |
2024/03/06(水) 20:30:11.51ID:S5Dm0o31
>>179
封筒で交換したら儲かると両者が認識できる分布は存在するし、
箱入り無数目の100人は持っている情報が違うんだから全員確率0だと思ってても全く変ではない
0184132人目の素数さん
垢版 |
2024/03/06(水) 20:33:49.56ID:IDPoig8I
>>182
>封筒で交換したら儲かると両者が認識できる分布は存在するし
でも実際は儲からない
>箱入り無数目の100人は持っている情報が違うんだから
>全員確率0だと思ってても全く変ではない
でも全員が同時にはずれることは決してない
だから全員確率0は明らかに変だよ
0185132人目の素数さん
垢版 |
2024/03/06(水) 20:35:01.41ID:IDPoig8I
>>183
>どんな確率の問題でも、普通はPは任意で定式化するやろ
んなアホなことあるかい
0186132人目の素数さん
垢版 |
2024/03/06(水) 20:36:29.25ID:S5Dm0o31
>>184
実際儲かるし、誰かが必ず当てられるとしても、情報が違ったらみんな確率が0なのは何もおかしくない
0188132人目の素数さん
垢版 |
2024/03/06(水) 20:37:58.76ID:IDPoig8I
>>186
>実際儲かるし
儲からないよ 

君、医者で診てもらったほうがいい ●ってるよ
0190132人目の素数さん
垢版 |
2024/03/06(水) 20:38:52.75ID:IDPoig8I
>>187
君の常識は 全数学界の非常識

君、大学で数学学んだこと一度もないでしょ?
0191132人目の素数さん
垢版 |
2024/03/06(水) 20:39:33.86ID:IDPoig8I
>>189 君が数学書全く読めない素人なだけだよ
0192132人目の素数さん
垢版 |
2024/03/06(水) 20:40:13.39ID:IDPoig8I
ID:S5Dm0o31 は高卒かな?
0193132人目の素数さん
垢版 |
2024/03/06(水) 20:41:08.76ID:S5Dm0o31
>>188
金額が自然数しか取らないとして、封筒開けたら1円入ってたら、誰が見ても変えたほうが得やろ
ちゃんと問題を考察してレスしてんの?
0194132人目の素数さん
垢版 |
2024/03/06(水) 20:42:28.03ID:IDPoig8I
本を丸写しして玄人に見せかけてるけどちょっと語るとボロが出る感じ
0195132人目の素数さん
垢版 |
2024/03/06(水) 20:43:41.23ID:IDPoig8I
>>193 その場合だけな

君、マジでヤバいよ 医者で診てもらいな
0196132人目の素数さん
垢版 |
2024/03/06(水) 20:44:15.95ID:IDPoig8I
ID:S5Dm0o31は誇大妄想入ってるな
0198132人目の素数さん
垢版 |
2024/03/06(水) 20:46:43.62ID:IDPoig8I
>>197
誤 ご飯
正 アヘン

もう、完全に中毒患者だな
0199132人目の素数さん
垢版 |
2024/03/06(水) 20:59:42.75ID:UPLSLbzu
コーシー分布:x^−2 程度の減衰のため,減衰が遅い
裾の重い分布あるいはヘヴィーテイル です
裾の減衰は、必須です

https://mathlandscape.com/cauchy-distrib/
数学の景色
コーシー分布の定義と性質とその証明 2022.04.11
見ての通り,正規分布に比べて,コーシー分布の方が,
0 から遠いところでの減衰が遅く,裾の厚い分布 (heavy tailed) になっています。これは,正規分布の確率密度関数が指数的に減衰するのに対し,コーシー分布は
x^−2 程度の減衰のため,減衰が遅いわけです。
コーシー分布は,期待値が定義できず,正規分布より減衰が遅い,裾の厚い分布(裾の重い分布)として有名です。

https://ja.wikipedia.org/wiki/%E8%A3%BE%E3%81%AE%E9%87%8D%E3%81%84%E5%88%86%E5%B8%83
裾の重い分布あるいはヘヴィーテイルとは、確率分布の裾がガウス分布のように指数関数的には減衰せず[1]、それよりも緩やかに減衰する分布の総称。 また類似の用語に、ファットテイル、裾の厚い分布、ロングテール、劣指数的(subexponential)などがある。

https://en.wikipedia.org/wiki/Heavy-tailed_distribution
Heavy-tailed distribution

https://ja.wikipedia.org/wiki/%E3%82%B3%E3%83%BC%E3%82%B7%E3%83%BC%E5%88%86%E5%B8%83
コーシー分布
https://en.wikipedia.org/wiki/Cauchy_distribution
Cauchy distribution
0200132人目の素数さん
垢版 |
2024/03/06(水) 21:13:58.24ID:hk/0+lKf
>>193
わろた
それ 儲かる じゃなく 儲かる場合が無い訳じゃない だろw
日本語正しく使えよ C鮮人か?
0202132人目の素数さん
垢版 |
2024/03/06(水) 21:40:15.99ID:UPLSLbzu
>>197
>今日は1円の封筒でご飯だな

スレ主です
あなたが正しい
「金額が自然数しか取らないとして、封筒開けたら1円入ってたら、誰が見ても変えたほうが得」
ですね

なお、分布の話は >>199を見てください
0203132人目の素数さん
垢版 |
2024/03/06(水) 21:42:56.38ID:UPLSLbzu
>>198
違法薬物についての投稿
運営にアク禁にするように投稿しました
繰り返すなら、同じように運営にアク禁にするように投稿します!
0204132人目の素数さん
垢版 |
2024/03/06(水) 21:49:29.16ID:S5Dm0o31
実際、交換したら儲かるという計算結果にしかならんだろ。誰がどう計算してももう片方には2円入ってるんだから

179 132人目の素数さん 2024/03/06(水) 20:22:16.26 ID:IDPoig8I
2つの封筒で、2人ともが「交換したら儲かる」という計算結果になるなら間違ってる
0205132人目の素数さん
垢版 |
2024/03/06(水) 22:01:57.39ID:hk/0+lKf
>>204
それ
>2人とも
の要件満たしてなくね?
0207132人目の素数さん
垢版 |
2024/03/06(水) 22:44:34.04ID:hk/0+lKf
>>206
封筒の中身は自然数で、片方の封筒に1円が入っていた場合、他方の封筒には2円が入っている。
1円を引いた人は相手が2円と分かるから交換すれば得。
2円を引いた人は相手の金額が分からないから交換すれば得とは言えない。
すなわち、2人ともが「交換したら儲かる」という計算結果になっていない。

こう言ったときおまえはこう反論するかも知れない。
2円を引いた人は相手の金額が1円か4円かのどちらかだと分かっている。
期待値1円×1/2+4円×1/2は2円より大きい。よって交換したら得。

しかしこれは誤り。
なぜなら、上記の期待値計算が正当化されるには(2円,1円)という出題と(2円,4円)という出題が等確率で現れる必要があるが、そのような前提は無いから。
0208132人目の素数さん
垢版 |
2024/03/06(水) 22:49:19.23ID:S5Dm0o31
もしかして、この2人って解答者が2人いて別々の封筒を選んだときの話だったの?それ書かないと誰にも通じないよ。
もちろん、このときも2人ともが「交換したら儲かる」という計算結果になる分布を作るのは簡単にできる。1円の場合と大して変わらない演習問題レベル
このときは当然だけど、どっちかは入れ替えたら損する。でも、2人ともが「交換したら儲かる」という計算結果にはなる

179 132人目の素数さん 2024/03/06(水) 20:22:16.26 ID:IDPoig8I
2つの封筒で、2人ともが「交換したら儲かる」という計算結果になるなら間違ってる
0209132人目の素数さん
垢版 |
2024/03/06(水) 22:52:08.51ID:S5Dm0o31
>>207
2人ってのが出題者と解答者だと思ってたんだけど…
解答者が2人のときは交換したらどちらかは損するが、両方とも計算では得になる場合を1円と同じように作ればいい
簡単な練習問題
0210132人目の素数さん
垢版 |
2024/03/06(水) 22:55:09.06ID:UPLSLbzu
ほいよ
 前々スレより
https://rio2016.5ch.net/test/read.cgi/math/1707524330/735
「2つの封筒の問題」ね、下記ですね

(参考)
https://en.wikipedia.org/wiki/Two_envelopes_problem
Two envelopes problem
2 つの封筒の問題
(google訳 一部修正)
2つの封筒問題は交換パラドックスとしても知られ、確率論におけるパラドックスです。これは、決定理論と確率論のベイズ解釈において特に興味深いものです。これは、ネクタイのパラドックスとして知られる古い問題の変形です。この問題は通常、次の例のような 仮説的な課題を定式化することによって導入されます。

それぞれにお金が入った 2 つの同じ封筒が渡されたと想像してください。一方にはもう一方の2倍の量が含まれています。封筒を 1 つ選び、その中に含まれているお金を保管しておいてもよいでしょう。エンベロープを自由に選択しますが、それを検査する前に、エンベロープを切り替える機会が与えられます。切り替えたほうがいいでしょうか?

状況は対称であるため、エンベロープを切り替えることに意味がないことは明らかです。一方、期待値を使用した単純な計算では、逆の結論が示唆されます。つまり、封筒を交換すると常に 2 倍のお金を得ることができるため、封筒を交換することが常に有益である一方で、唯一のリスクは現在持っているお金が半分になることです。[1]

解決策の例
両方の封筒に入っている合計金額が一定であると仮定します。
略す
したがって、総額が固定されていると仮定すると、スワップは維持よりも優れているわけではありません。
期待値 E 略す は、どちらの封筒でも同じです。したがって、矛盾は存在しません。[5]

この有名な謎は、2 つの封筒の合計金額が固定されている状況と、1 つの封筒の金額が固定されており、もう 1 つの封筒の金額がその 2 倍または半分になる可能性がある状況を混同することによって引き起こされます。いわゆるパラドックスでは、すでに指定され、すでにロックされている 2 つの封筒が提示されます。
略す

https://researchmap.jp/blogs/blog_entries/view/92228/98e56ed2a2e2f485f4c22d2bcac369c0?frame_id=526781
2つの封筒問題
投稿日時 : 2014/04/07 関 勝寿
数年前に書いた文書ですが、要望によりアップします。
2つの封筒があり、それぞれにお金が入ってます。片方の封筒に入っている金額が、もう片方の封筒に入っている金額の2倍となっていることが分かっています。あなたは、最初にどちらか片方の封筒を選び、中身を見る事ができます。その後、改めてどちらの封筒を選ぶか決めることができます。二度目に選んだ封筒の中身をもらうことができます
1.最初の封筒に1万円入っていました。この時、封筒を交換する方が得か、交換しない方が得か、あるいはどちらでも同じか?最初に選んだ封筒を封筒Aとすると、ランダムに封筒を選んだことから、封筒Aが金額の小さい封筒である確率は1/2、金額の大きい封筒である確率は1/2です。すると、もう片方の封筒Bに入っている金額は、1/2の確率で2万円、1/2の確率で5000円となります。したがって、封筒Bに入っている金額の期待値は 1/2*20000+1/2*5000=12500 より、12500円となります。封筒Aを封筒Bに交換する事で、期待値が2500円増えますから、交換する方が得です
略す
0211132人目の素数さん
垢版 |
2024/03/06(水) 22:59:41.95ID:hk/0+lKf
A君とB君がじゃんけん勝負をする
あいこの場合は勝負がつくまでやり、毎回必ず一方が勝ち他方が負けるとする
さてA君が勝つ確率は1/2と言えるでしょうか?
0212132人目の素数さん
垢版 |
2024/03/06(水) 23:08:14.12ID:hk/0+lKf
>>210
>封筒Aが金額の小さい封筒である確率は1/2、金額の大きい封筒である確率は1/2です。
正しい

>すると、もう片方の封筒Bに入っている金額は、1/2の確率で2万円、1/2の確率で5000円となります。
間違い
理由は>>207で述べた
0213132人目の素数さん
垢版 |
2024/03/06(水) 23:10:56.48ID:hk/0+lKf
二つの封筒問題でパラドックスとなるのは間違った確率計算の結果
正しい確率計算ならパラドックスにならない
0214132人目の素数さん
垢版 |
2024/03/07(木) 00:04:29.32ID:C3Ro7iPT
2人いるときに、どちらの人も交換したら儲かる計算になる場合がある例を眺めながら飯を食べよ

179 132人目の素数さん 2024/03/06(水) 20:22:16.26 ID:IDPoig8I
2つの封筒で、2人ともが「交換したら儲かる」という計算結果になるなら間違ってる
0215132人目の素数さん
垢版 |
2024/03/07(木) 00:54:04.64ID:sZFxxPrG
>>214
未だわかってなかったのかw
頭わるw
0217132人目の素数さん
垢版 |
2024/03/07(木) 01:17:09.82ID:sZFxxPrG
>>216
君に言ってるんだよ
頭悪いね
0219132人目の素数さん
垢版 |
2024/03/07(木) 05:49:40.87ID:B1UavB2/
2つの封筒で、どの金額も同確率、とすると
2つの封筒の合計金額の期待値が確実に発散する

もちろん
「確率が合計金額に反比例する」
としても期待値は発散するが
それはギリギリの線である
(つまりより確率の減少度を増せば期待値が収束する)
そしてそのギリギリの線の分布で「交換しても同じ」になる
(それより減少度が増すと「交換すると損」になる)

箱入り無数目で、どの無限列も同確率、とすると
決定番号の期待値が発散する

期待値が収束する場合には、
もちろん計算で箱入り無数目の成功確率が計算できるし
その場合99/100より高くなる
ギリギリ収束しなくなるところで、99/100になる
0220132人目の素数さん
垢版 |
2024/03/07(木) 05:57:34.84ID:B1UavB2/
箱入り無数目は
「2つの封筒の問題で、”一方が他方の2倍”の条件をとっぱらった場合」
と同様と考えることができる
この場合も、「交換しても同じ」となるのは、
確率が2つの封筒の合計金額に反比例する場合
(ただし、ここでは金額は正の実数としている)
0221132人目の素数さん
垢版 |
2024/03/07(木) 06:22:28.58ID:B1UavB2/
>>220
>「交換しても同じ」となるのは、
>確率が2つの封筒の合計金額に反比例する場合

この根拠は 任意のxについて
0~xの区間と、x~∞の区間の量が同じ
というおかしな特性を満たすから

非負じゃなくR全体なら、ただの一様で構わないが
0222132人目の素数さん
垢版 |
2024/03/07(木) 06:48:38.81ID:B1UavB2/
要するに分布の異常な特性に基づいた結論を鵜呑みにすると誤る
0224132人目の素数さん
垢版 |
2024/03/07(木) 11:09:22.01ID:/uk4GHE2
別の言い方をすると、高校生が関数が連続であることを証明しようとしてるようなもの、絶対無理
0225132人目の素数さん
垢版 |
2024/03/07(木) 11:31:51.09ID:sZFxxPrG
>>224
じゃ諦めたら?
0226132人目の素数さん
垢版 |
2024/03/07(木) 11:32:15.63ID:o5d27C4l
>>223 「考えれない」とか残念な日本語使ってる分際で
「高校生が関数が連続であることを証明しようとしてるようなもの」
なんていってイキる

これが噂の「拡大中二病」たる大二病か
0227132人目の素数さん
垢版 |
2024/03/07(木) 11:34:07.68ID:o5d27C4l
そもそも、おかしな事前条件を前提しようとする点に、心の不自由さを感じる
0228132人目の素数さん
垢版 |
2024/03/07(木) 11:37:45.56ID:o5d27C4l
誤 事前条件
正 事前分布
0229132人目の素数さん
垢版 |
2024/03/07(木) 11:53:51.05ID:G6yRkQAe
>>223
>確率が考えれないのに分布を考える、屋上屋を架す

スレ主です
コメントありがとうございます
直接のお答えになっていないが、瞼の父などで説明しようと思う

1)「瞼の父」は、下記で”男性芸能人の前に見知らぬ子連れの女性が現れ、「私を覚えておいででしょうか?3年前、○○温泉にあなたがいらしたとき、一晩お相手をした明子でございます。これがその時に出来た子供で。(子供に)△△ちゃん、これがあなたのお父さんですよ」などと言い・・”
 という”どっきり”TV
2)さて、現在なら親子DNA鑑定 で、その確率は「DNA親子鑑定は尤度比と呼ばれる数値で表されます」とある
 実は、詳細は突っ込まれると答えられない。疑問点はご自分で検索請う
3)何を確率として扱うか? 下記「確率の歴史」ご参照
 下記「確率の歴史」の歴史が示すところ、確率の歴史→確率の対象の拡張の歴史でもあったわけです
 サイコロ賭博からはじまって、20世紀 仮説検定、ブラウン運動(確率過程論)、株式市場における不規則な変動
 そして、21世紀えは 親子DNA鑑定の確率がある

さて、言いたいことは
確率が考えられるかどうか?
それは、考えてみないと分からないってこと
考えてみた結果、分布などを考えると
”これは確率が考えられない”となるのであって
逆ではないと思う

(参考)
https://www.weblio.jp/content/%E7%9E%BC%E3%81%AE%E7%88%B6
瞼の父
出典: フリー百科事典『ウィキペディア(Wikipedia)』 (2022/06/05 01:37 UTC 版)
「スターどっきり(秘)報告」の記事における「瞼の父」の解説
初期にあったコーナー。男性芸能人の前に見知らぬ子連れの女性が現れ、「私を覚えておいででしょうか?3年前、○○温泉にあなたがいらしたとき、一晩お相手をした明子でございます。これがその時に出来た子供で。(子供に)△△ちゃん、これがあなたのお父さんですよ」などと言い、男性芸能人を無理矢理父親にしてしまう。三波伸介や小野ヤスシもやられたことがある。コーナーナレーションは芥川隆行が担当。
※この「瞼の父」の解説は、「スターどっきり(秘)報告」の解説の一部です。
「瞼の父」を含む「スターどっきり(秘)報告」の記事については、「スターどっきり(秘)報告」の概要を参照ください。

https://secure.dnajpn.com/column/7070/
dnajpn.com 2023.05.29
高確率で親子関係の有無を調べるDNA型鑑定。その根拠や、正確な結果を得るために大切なことを解説
DNA鑑定で父親であることの確率とは
親子関係を証明する父権肯定確率
DNA親子鑑定は尤度比と呼ばれる数値で表されます。この数値は「2つの異なった仮説の下で同じ証拠に対する確率の比」であり、この数値と「DNA鑑定結果以外の点についての確率」である事前確率によって父権肯定確率が算出されます。
父権肯定確率とは、親子関係の確率ではなく鑑定の信頼度を示している用語のことです。ただ、DNA鑑定の結果以外も含めて考慮した確率である父権肯定確率が99.99%以上であれば、生物学的親子関係があることがほぼ間違いありません

つづく
0231132人目の素数さん
垢版 |
2024/03/07(木) 11:54:06.86ID:G6yRkQAe
つづき

https://ja.wikipedia.org/wiki/%E7%A2%BA%E7%8E%87%E3%81%AE%E6%AD%B4%E5%8F%B2
確率の歴史
確率という言葉には二つの意味合いがある。一つはある仮説の、それにまつわる判断材料から導かれる蓋然性のことであり、もう一つはサイコロやコインを投げることのような確率過程的なふるまいを指す。証拠法のような前者の研究は歴史的により古い一方で、サイコロの数学的取り扱いは1650年代にパスカルとフェルマーの著作で始まった。確率は統計学とは区別される(統計学の歴史参照)。統計学がデータやそれによる推測を取り扱うのに対し、(確率論的な)確率はデータやその結果の裏にある確率論的(ランダム)な過程を取り扱う。

20世紀
確率と統計はロナルド・フィッシャーとイェジ・ネイマンの仮説検定の作業を通して密接に繋がった。そして現在広く生物学や心理学の実験や薬の治験、経済学や他のすべての分野においても同様に応用されている。たとえばある薬がいつも効果的だという仮説は、もしそれが正しければ観察されるであろう確率分布を引き起こす。もし観察がおおよそ仮説に合致していれば仮説は裏付けられたことになり、もし合致していなければ仮説は棄却される[6]。
確率過程論は マルコフ過程や、液体の中で浮遊する微粒子の不規則な動きであるブラウン運動のような領域の方へ広がった。
そのことが株式市場における不規則な変動の研究のためのモデルを提供した。同時にオプション評価(英語: Valuation of options)のための広範に使用されるブラック-ショールズ方程式としての成功を含む金融工学における洗練された確率論のモデルの使用へ導いた[7]。
20世紀にはまた確率解釈における長期にわたる論争があった。20世紀中盤には 頻度主義が支配的だった。そして確率が長期にわたる沢山の試行の相対的な頻度を意味するということが伴った。20世紀の最後には ベイズ確率の観点の復興があった。ベイズ確率によれば、根本的な確率概念というのはその根拠によって命題がどれほどよく支えられているかによる。

数学的な確率の扱いは、起こりうる結果が無数にあるときは、コルモゴロフによる公理的確率論 (1933) の導入によって容易になった
(引用終り)
以上
0232132人目の素数さん
垢版 |
2024/03/07(木) 11:59:47.27ID:G6yRkQAe
>>229 タイポ訂正

 そして、21世紀えは 親子DNA鑑定の確率がある
   ↓
 そして、21世紀では 親子DNA鑑定の確率がある
0233132人目の素数さん
垢版 |
2024/03/07(木) 12:28:46.08ID:+1eraRT4
>>229 
>親子DNA鑑定 で、その確率は「DNA親子鑑定は尤度比と呼ばれる数値で表されます」とある
>実は、詳細は突っ込まれると答えられない。疑問点はご自分で検索請う
答えられないのにわけもわからずしゃべりたがる もう病気だね

「受け売り症候群」の解決策 身の丈で考える変換力と自分主語
https://www.projectdesign.jp/201501/changeleader/001812.php
0236132人目の素数さん
垢版 |
2024/03/07(木) 12:46:54.10ID:WYi6Mr1Z
対象となる無限列がS^N(Sは元が二つ以上ある集合)全体なら、
尻尾同値類の代表が具体的に得られないから
「架空の問題」と思うだろうけど
無限列の範囲を限定し、それらの尻尾同値類の代表が具体的に得られるなら
出題から実現できそうに思うが?
(例えば、極端に言って、全部0の列に同値な列に限定してしまえば、代表は全部0の列一つでいい)
0237132人目の素数さん
垢版 |
2024/03/07(木) 13:51:21.59ID:G6yRkQAe
>>235-236
>箱入り無数の目は現実の問題か、数学の中にしか存在しないトイモデルだろ
>無限列の範囲を限定し、それらの尻尾同値類の代表が具体的に得られるなら
>出題から実現できそうに思うが?
>(例えば、極端に言って、全部0の列に同値な列に限定してしまえば、代表は全部0の列一つでいい)

ありがとうございます
スレ主です
(ID:/uk4GHE2 さんは、弥勒菩薩さまかな?)
ちょっと別の視点から

1)箱入り無数の目の無限の箱で、箱に入れる数を
 正規分布で、平均m 標準偏差σの整数を
 ランダムに入れたとする
 そこで、無限列を2列に分けて、奇数番列と偶数番列にする
 奇数番列をすべて開けて、統計処理をして、平均m 標準偏差σを求める
 偶数番列のあるn+1番目以降を全部開けて、平均m 標準偏差σを確認する
 この情報から、n番目を 平均mと唱える。的中確率は、正規分布から求まる
2)さていま、整数→実数 としてみよう
 ”平均m”と唱えてもダメ。∵実数の的中には、ある範囲を必要とする
 そこで、”平均m±1σの実数”と答える。的中確率は、正規分布から求まる

これが、スタンダードな確率論的な箱入り無数の目の解き方でしょう
さて、上記2)で、”平均m±1σの実数”で ”±1σ”をもっと狭い範囲に絞れないか?
絞るのは可能だが、狭くするほど的中確率が落ちるのは、自明

ところが、時枝さん「箱入り無数目」手法では、±1σ→±0 にできる
と主張する
これは、さすがに確率論の外でしょう!w

なお、上記1)の実数版 ”正規分布で、平均m 標準偏差σの実数をランダムに入れた”
は、「箱入り無数目」の設定条件(下記)に適合することをご確認ください

(参考)時枝記事 >>1より
https://imgur.com/a/8bqlb08
数学セミナー201511月号「箱入り無数目」
0238132人目の素数さん
垢版 |
2024/03/07(木) 14:07:44.64ID:G6yRkQAe
>>237
>さて、上記2)で、”平均m±1σの実数”で ”±1σ”をもっと狭い範囲に絞れないか?
>絞るのは可能だが、狭くするほど的中確率が落ちるのは、自明
>ところが、時枝さん「箱入り無数目」手法では、±1σ→±0 にできる
>と主張する
>これは、さすがに確率論の外でしょう!w

・ここまでは、学部3年で確率論を学べばすぐわかる
・問題は、なぜ時枝ほどの人が、「箱入り無数目」に乗せられたのか?w
・「二つの封筒」問題同様、確率論には 色々とおもしろい問題が多数ありまして
 それと同様に、「箱入り無数目」のパラドックスを論じようというのが、このスレの趣旨です!

時枝氏と同じように 「箱入り無数目」に乗せられている人が多いほど 面白いのです ;p)
0240132人目の素数さん
垢版 |
2024/03/07(木) 14:13:31.94ID:sZFxxPrG
>>235
数学パズル
0241132人目の素数さん
垢版 |
2024/03/07(木) 14:16:50.15ID:sZFxxPrG
>>237
>ところが、時枝さん「箱入り無数目」手法では、±1σ→±0 にできる
>と主張する
また嘘ついてるよこのサイコパス

>これは、さすがに確率論の外でしょう!w
一様分布は確率論のど真ん中
0242132人目の素数さん
垢版 |
2024/03/07(木) 14:21:59.70ID:sZFxxPrG
0237132人目の素数さん
2024/03/07(木) 13:51:21.59ID:G6yRkQAe
ありがとうございます
スレ主です
(ID:/uk4GHE2 さんは、弥勒菩薩さまかな?)

0239132人目の素数さん
2024/03/07(木) 14:13:31.76ID:/uk4GHE2
>>237
だめだこりゃ



笑えるなw
0243132人目の素数さん
垢版 |
2024/03/07(木) 14:34:36.30ID:WYi6Mr1Z
>>237
>時枝さん「箱入り無数目」手法では、±1σ→±0 にできる
全然箱入り無数目を理解してないな

箱に入れる数を{0,・・・,n-1}をランダムに入れる
ただし、ある箱に0をいれたら、その先の箱は全部0とする
この場合 生成される列は確率1で、全部0の列に尻尾同値になる
決定番号1の列 1個 確率1/n
決定番号2の列 n-1個 確率(n-1)/n^2
決定番号3の列 (n-1)^2個 確率(n-1)^2/n^3
・・・

この設定で、箱入り無数目の戦略を使うと、2列の場合
少なくとも確率1/2で、0の箱を選べる
まあ、別にそういう分布になってると分かってしまえば
あるm番目より先の1箱を選べば確率1/2で0
となるmが求まる

ここで分かるように、中身が1,…,n-1の箱は当てられない
箱入り無数目は1が考えるようなサイコロの目を当てる問題ではない
0244132人目の素数さん
垢版 |
2024/03/07(木) 14:42:11.71ID:G6yRkQAe
>>234
>「なんとなく分かった」症候群
>https://researchmap.jp/blogs/blog_entries/view/76190/9201d9d57eae83d83530047e2c78a44e?frame_id=562130

・「数学に王道なし」
・”一歩一歩 ちゃんと理解してから進みましょう”
・「なんとなく分かった」ではダメです!
・これを、実践して留年5年 下記のわんこらさん
 京都大学数学科生 解析入門1 杉浦光夫 を 「数学に王道なし」を実践して、ヒキコモリになった(youtubeご参照)

わんこらさん曰く、先に進めばわかること沢山あるよと
これ大事ですよ

(参考) 
https://youtu.be/aWPAHRsCU_Q?t=920 (ポイントは ここ 920sから)
https://www.youtube.com/watch?v=aWPAHRsCU_Q
僕がたどり着いた数学の勉強の仕方…わんこら式数学の勉強法はこうやって生まれた
わんこらチャンネル
2020/05/30
留年繰り返して7年で大学卒業した後
ニートになった僕ですが
そんな僕が挫折を繰り返してきた歴史と、たどり着いた数学の勉強の仕方について動画にしました
この勉強法がわんこら式と呼ばれるようになりました
大学の数学の専門書、解析入門1を使って
数学の勉強法について話します
色々な人の参考になれば嬉しいです

https://www.アマゾン
解析入門 T(基礎数学2) 単行本 – 1980/3/31
杉浦 光夫 (著)東京大学出版会
書評
seo
5つ星のうち3.0 入門書としては☆ひとつ
2018年6月30日
Amazonで購入
解析学という書名で良いと思います。
入門とわざわざ付けることは非合理的で、何も良いことはありません。
様々な数学的分野は互いに互いを前提とする必要があるので、縦割りに順番に習得するものではなく、混じり合い行ったり来たりしながら学ぶものです。
よって本書が要求するある程度以上の数学的知識の前提を満たす者は、ある程度解析学にも触れているでしょう。
そういう意味では、本書は解析学の入門者を対象にしておらず、解析学も含めたある程度の数学的形式が頭の中にすでに存在する人を対象にしています。
前提とするものを最小限にし、かつ理解しやすさと厳密性を可能な限り両立させる事ができている本、それがいわゆる良い入門書だと思います。
厳密性と網羅性が優れている本が良い入門書とは思えません。
0245132人目の素数さん
垢版 |
2024/03/07(木) 14:50:15.39ID:G6yRkQAe
>>241
>一様分布は確率論のど真ん中

一様分布は、必ず区間有限で[a,b]としなければいけません
例えば、b→∞ とすると、平均値も分散も →∞に発散します
この場合は、確率論の外
これが「箱入り無数目」の決定番号です(「箱入り無数目」の決定番号>>1には上限がない)

(参考)>>7
https://ai-trend.jp/basic-study/bayes/improper_prior/
AVILEN Inc. 2020
2020/04/14
非正則事前分布とは?〜完全なる無情報事前分布〜
ライター:古澤嘉啓
目次
1 非正則な分布とは?一様分布との比較
2 非正則分布は確率分布ではない!?
3 非正則事前分布は完全なる無情報事前分布
4 まとめ
0246132人目の素数さん
垢版 |
2024/03/07(木) 14:50:47.40ID:/uk4GHE2
>>237
仮に統計の問題とすると分布が分かったらそれが正しいかどうか検定するんだろ、どうやって検定するんだ?
0247132人目の素数さん
垢版 |
2024/03/07(木) 15:04:52.06ID:sZFxxPrG
>>245
箱入り無数目の確率計算は決定番号の分布を使ってないからナンセンス
未だ理解できないの? 馬鹿だね
0248132人目の素数さん
垢版 |
2024/03/07(木) 15:41:32.53ID:G6yRkQAe
>>246
>仮に統計の問題とすると分布が分かったらそれが正しいかどうか検定するんだろ、どうやって検定するんだ?

コメントありがとうございます
スレ主です

・確率の問題で、>>4のSergiu Hart氏ような 数当てゲームと思ってください
・よって、箱に数を入れる出題者と、箱の数をなんらかの方法で推定して当てようとする回答者の
 勝ち負けのゲームです
>>237で示したのは、 一つの方法として 箱に入れる数を
 a)正規分布で、平均m 標準偏差σの整数を ランダムに入れたとする
 b)同 整数→実数
 の両ケースで、箱は一つを残して 他の箱を全部開けて良く
 残る未開封の1つの箱の中の数を推定する方法を示したのです
・もちろん、上記は通常の確率論の範囲です
 全実数中 r∈R から選んで良いという条件(箱入り無数目>>1の通り)を、かなり当てやすくした簡易モデルです
・ゲームを繰り返せば 勝ち負けの勝率はでる
 それは、数当ての的中率と同じです
0249132人目の素数さん
垢版 |
2024/03/07(木) 15:50:06.29ID:+1eraRT4
>>248
>>統計の問題とすると分布が分かったらそれが正しいかどうか検定するんだろ、どうやって検定するんだ?
>ゲームを繰り返せば …

…正しいかどうか検定できる、といいたいんだろうが

それ、箱入り無数目とは全然別の話に逃げまくってるな

やっぱり高卒レベルの数学ド素人には無理だったか
0250132人目の素数さん
垢版 |
2024/03/07(木) 15:53:48.77ID:+1eraRT4
「箱入り無数目」の要は、決定番号の分布でもなんでもなくて
単に100列のうち、他の99列の決定番号より大きな決定番号を持つ列が2列以上存在することはない
(なぜならx<yかつx>yなんてことはないから)という点につきるんだが

自然数が全順序集合であることも知らんド素人が、いくら
「確率論ガー、確率過程ガー」と吠えたところで、
黒が白になることはないよ

南無阿弥陀仏
0251132人目の素数さん
垢版 |
2024/03/07(木) 16:11:13.83ID:ySP8iCZD
箱の中身が全公開されてるモデルで箱の中身を当てられたとかバカなんじゃないか
0252132人目の素数さん
垢版 |
2024/03/07(木) 16:17:19.66ID:MBLauWll
>>251
>箱の中身が全公開されてるモデルで
>箱の中身を当てられたとか

誤 全公開
正 ほとんど全て(=有限個の例外を除き)公開

そもそも、列の尻尾が分かればその尻尾同値類の代表列がわかるが
元の列は当然ながらその尻尾同値類の代表列とほとんど全て一致する

要するに開けてない箱が一致箇所に入ってるかどうかのバトル
0255132人目の素数さん
垢版 |
2024/03/07(木) 16:27:23.16ID:MBLauWll
>>253
そもそも時枝のオリジナルじゃないし
時枝自身 問題を(箱の中身を確率変数とする形に)拡大しても
成立するんじゃないかと誤解している
後半部の記載(非可測とか確率変数の無限族の強い独立性とか)はそういうこと
0256132人目の素数さん
垢版 |
2024/03/07(木) 16:28:19.13ID:MBLauWll
>>254 ちょっと何言ってるか分からない
0259132人目の素数さん
垢版 |
2024/03/07(木) 16:46:09.82ID:MBLauWll
>>258
なんで「最初に∀をつけたら全公開」だというのかが分からない
口から出まかせのハッタリとしか思えんが
0262132人目の素数さん
垢版 |
2024/03/07(木) 16:59:42.51ID:C3Ro7iPT
これまた書かないとだめなの?

370 132人目の素数さん sage 2024/02/15(木) 21:51:55.09 ID:Yql9K+Mt
例えばさ、箱の中に正の整数が入ってます。あなたはそれを見ずに何か正の整数を宣言します。あなたの答が箱の中の数以下なら勝利です。必勝法はありますか?
という問題なら、∃x.∀y. x≦y が成立するから必勝ですって誰でも答えられるでしょ
これを、∀y. ∃x. x≦yが成立するから必勝ですって言ったらおかしいでしょ
後者の命題は正の整数の代わりに整数にしても成り立つけど、明らかに整数では必勝法はない。
だから、箱の中を見てないと主張するには∀をなるべく内側に入れた命題を証明しないとだめなんじゃよ
0263132人目の素数さん
垢版 |
2024/03/07(木) 17:02:02.13ID:UBNnmiVc
>>260
>∀が先頭についてるんだから相手側に全公開してるだろ
全然分からん
>>261
>なんでこの話でまたループしないといけないの?
ループ以前 そもそも君の云ってることが初めから一貫してわからん
0264132人目の素数さん
垢版 |
2024/03/07(木) 17:04:15.46ID:sZFxxPrG
>>253
君のようなチンピラとは違うよ
0265132人目の素数さん
垢版 |
2024/03/07(木) 17:05:03.84ID:sZFxxPrG
>>254
量化と公開/非公開には何の関係も無い
阿呆?
0266132人目の素数さん
垢版 |
2024/03/07(木) 17:05:27.81ID:Lv61i9KN
>>262
そもそもそれが全然おかしい

「∃x.∀y. x≦y が成立するから必勝です」なんて誰もいってないんだが

>∀y. ∃x. x≦yが成立するから必勝ですって言ったらおかしいでしょ
そもそもそれ以前なんだが
0267132人目の素数さん
垢版 |
2024/03/07(木) 17:06:11.83ID:sZFxxPrG
>>257
おまえは分かってるの?
0268132人目の素数さん
垢版 |
2024/03/07(木) 17:06:50.11ID:sZFxxPrG
>>259
>口から出まかせのハッタリとしか思えんが
同意
0269132人目の素数さん
垢版 |
2024/03/07(木) 17:07:46.29ID:sZFxxPrG
>>260
いいから阿呆は口閉じな?
阿呆に口きく権利は無い
0270132人目の素数さん
垢版 |
2024/03/07(木) 17:08:13.33ID:Lv61i9KN
如何なる100列でも、100列中99列は選べば勝て残り1列に選んで負ける列がある、というだけなんだがな
なんでこんな簡単なことが分からんのか、そっちがわからん
0273132人目の素数さん
垢版 |
2024/03/07(木) 17:11:52.87ID:sZFxxPrG
ハッタリ野郎どもは記事を読むことすらしてない・出来てないんだろう
読んでたらこんな阿呆なことは抜かさない
読んで抜かしてたら真正の阿呆
0274132人目の素数さん
垢版 |
2024/03/07(木) 17:12:33.86ID:sZFxxPrG
>>271
じゃ去りな
数学板は阿呆の来る所じゃない
0275132人目の素数さん
垢版 |
2024/03/07(木) 17:14:15.96ID:sZFxxPrG
>>272
考察というのが記事後半ならそんなとこだね
後半はまったくの駄作で付け足すべきでなかった
0276132人目の素数さん
垢版 |
2024/03/07(木) 17:14:28.60ID:+1eraRT4
>>271
>だめだこりゃ話にならん
君がな 君、アスペ?
0277132人目の素数さん
垢版 |
2024/03/07(木) 17:17:19.79ID:+1eraRT4
∀と∃の話は、完全にゲームの意味を取り違えてる
文章が全く読めないアスペルガー君なのかな?
0278132人目の素数さん
垢版 |
2024/03/07(木) 17:17:43.07ID:C3Ro7iPT
>>270
出題者から見りゃ100人の解答者のうち少なくとも99人が正解するってだけだろ
解答者はみんな自分が正解する可能性は0だと思ってる
それで、もう問題はあらかた片付いてるじゃん
0279132人目の素数さん
垢版 |
2024/03/07(木) 17:19:22.71ID:+1eraRT4
>>278
>解答者はみんな自分が正解する可能性は0だと思ってる
でも100人全員間違うことはないんだから誤解
それでもう君が間違ってるってことで片付いてる

南無阿弥陀仏
0281132人目の素数さん
垢版 |
2024/03/07(木) 17:21:05.55ID:+1eraRT4
>>280 その名はstrea king
0283132人目の素数さん
垢版 |
2024/03/07(木) 17:26:03.62ID:sZFxxPrG
「ふしぎな戦略は,確率変数の無限族の独立性の微妙さをものがたる, といってもよい.」
ふしぎな戦略(前半)と確率変数の無限族(後半)はまったく無関係なのに、両者を結び付けて何か意味のあることを語った気になっている。
これは酷い。
0286132人目の素数さん
垢版 |
2024/03/07(木) 17:30:42.12ID:/uk4GHE2
R^N/〜の代表系を選んだところで選択公理を使っている。その結果R^N->R^N/〜の切断は非可測になる
0287132人目の素数さん
垢版 |
2024/03/07(木) 17:30:47.88ID:sZFxxPrG
>>278
>100人の解答者のうち少なくとも99人が正解する
は客観的事実だから
>出題者から見りゃ
は不要。

>解答者はみんな自分が正解する可能性は0だと思ってる
なぜ?
0288132人目の素数さん
垢版 |
2024/03/07(木) 17:32:29.28ID:+1eraRT4
>>285
1が言い出したことだから1に聞きな
はっきりいって1が当然だと思い込んでる前提を誰も認めてないけどね
0289132人目の素数さん
垢版 |
2024/03/07(木) 17:33:23.83ID:sZFxxPrG
>>282
なんの反論にもなってない
0290132人目の素数さん
垢版 |
2024/03/07(木) 17:33:29.60ID:/uk4GHE2
箱が無限に一列に並んでいるとおもったらいつの間にか百列になっている
0291132人目の素数さん
垢版 |
2024/03/07(木) 17:34:22.35ID:sZFxxPrG
>>286
だからなに?
0293132人目の素数さん
垢版 |
2024/03/07(木) 17:35:01.52ID:+1eraRT4
>>287
出題者は出題が分かってるから、誰が外れるか分かるでしょ

回答者は誰が外れるかは分からない
「自分が正解する可能性は0だと思ってる」
とかいうのは1が云ってる非正則分布のせいなので
そもそもそんな分布は間違ってるといって捨てればいいだけ
0294132人目の素数さん
垢版 |
2024/03/07(木) 17:36:00.65ID:/uk4GHE2
問題の設定が代わっているということだよ、ウマシカ絵文字おっさん
0295132人目の素数さん
垢版 |
2024/03/07(木) 17:36:34.40ID:+1eraRT4
>>292
なら口だすのやめな 間違ってる1よりもさらに何にも分かってないんだから
0296132人目の素数さん
垢版 |
2024/03/07(木) 17:36:35.68ID:sZFxxPrG
>>290
1本の無限列を100本の無限列に並べ替えることができることが分からないん?
君は阿呆かい?
0297132人目の素数さん
垢版 |
2024/03/07(木) 17:37:44.58ID:+1eraRT4
>>294
無限列1列を無限列100列に分けられることも理解できないってニワトリか?
0299132人目の素数さん
垢版 |
2024/03/07(木) 17:38:23.03ID:+1eraRT4
>>296 ニワトリ頭なんでしょうな
0300132人目の素数さん
垢版 |
2024/03/07(木) 17:38:43.35ID:G6yRkQAe
>>229
> 下記「確率の歴史」の歴史が示すところ、確率の歴史→確率の対象の拡張の歴史でもあったわけです
> サイコロ賭博からはじまって、20世紀 仮説検定、ブラウン運動(確率過程論)、株式市場における不規則な変動

補足投下します
なお「2000年代以降のフィールズ賞受賞者はすべて確率論に関連する研究者であった」を調べてみた結果を添付した
(2010年については、受賞者に確率論の該当する者がいるか判断できなかった)

(参考)
https://double-growth.com/ddm_ito_001/
みんなの運用会議
株価シナリオのモデルと伊藤のレンマ  by yamamoto
2019年11月26日
世界的な確率論の学者である伊藤清さんが1940年代に、Kolmogorovの論文「確率論における解析的方法」(Math Ann. 1931)を読んで、誘発されて、構想を立てたのが、以下の確率微分方程式でした。
{X}を連続時間tの変数としたとき、(X=株価と思ってください)

がKolmogorovの理論の出発点でしたが、そこから、以下を思いついたのです。
dX(t) = a(t,X)dt +√b(t,X) dB – – – (1)
これを解こうとしたわけですが、確率の項でもあるdBは、もちろん連続ではないからStieltjes積分ができない…
(伊藤先生の時代は、測度論が確立していませんでした…定義域上の関数の集合上で積分するというルベーグ積分の発想が斬新だった時代)
(1)に数学的な意味を与えるためには、積分の形に直して
X(t) = X(t=0) + ∫[from 0 to t] a(s,X) ds +∫[from 0 to t] σ(s,X)dB —-(1.1′)
(σ=√b)
とまずは書いてみたそうです。
dBは前述のようにStieltjes積分では定義できない。(1.1′)の最後の積分項をどう定義するのかが問題でした
略す

つづく
0301132人目の素数さん
垢版 |
2024/03/07(木) 17:39:05.54ID:G6yRkQAe
つづき

https://ja.wikipedia.org/wiki/%E7%A2%BA%E7%8E%87%E8%A7%A3%E6%9E%90
確率解析
確率解析学とは、伊藤清による確率積分、確率微分方程式、及び連鎖律に相当する伊藤の公式発表に端を発した数学の分野である。 伊藤清の「伊藤の公式」は米国科学アカデミーをして、ピタゴラスの定理に次ぎニュートンの微積分学の功績と並ぶ確率解析の基本定理だと言わしめている。
第二次世界大戦中の1942年に日本語で発表された確率微分方程式論は画期的な業績であり、 これによって非決定論的でランダムな時間発展の記述が可能となった。
いわゆる伊藤の公式は、 数学の諸分野に留まらず、例えば、物理学においては共形場理論、 工学においては制御理論、生物学においては集団遺伝学などに、さらに近年では、 経済学における数理ファイナンスに至るまで広範に応用されている。
伊藤清は第二次世界大戦中にマルコフ過程を定める微分方程式としてこの理論を発表した。
1960-1970年頃には渡辺信三、國田寛による確率積分のマルチンゲール理論化により伊藤理論は非常に使いやすい形に整備された。
また、1970年代以降のPaul Malliavinによる人類史上初の無限次元解析的視点が確率論の中で厳密に展開されることにより, 伊藤解析は大幅にその裾を拡げ, 他の数学分野を巻き込んで浸透した。
伊藤解析は, Malliavin解析(無限次元解析)と総称して, 確率解析と呼ばれることもある。
詳細は, Malliavin, Kusuoka-Stroock, Watanabeなどの原論文を参照せよ。
確率微分方程式の誕生レベルで、この分野は特に偏微分方程式論及び微分幾何学(無限次元空間上の幾何学)と深く関連している。
また、現在はLyonsに始まるラフパス解析理論、Hairerに始まる正則構造の理論などと強く融合するとともに、現代数学の中で更なる急激な発展が見込まれており、競争が激化している。
実際、2000年代以降のフィールズ賞受賞者はすべて確率論に関連する研究者であった。

https://en.wikipedia.org/wiki/Stochastic_calculus
Stochastic calculus
Stochastic calculus is a branch of mathematics that operates on stochastic processes. It allows a consistent theory of integration to be defined for integrals of stochastic processes with respect to stochastic processes. This field was created and started by the Japanese mathematician Kiyosi Itô during World War II.

つづく
0302132人目の素数さん
垢版 |
2024/03/07(木) 17:40:11.22ID:sZFxxPrG
>>293
誰が外れるか分からなくても
>100人の解答者のうち少なくとも99人が正解する
は分かるけど?
0303132人目の素数さん
垢版 |
2024/03/07(木) 17:40:12.85ID:G6yRkQAe
つづき

Applications
An important application of stochastic calculus is in mathematical finance, in which asset prices are often assumed to follow stochastic differential equations. For example, the Black–Scholes model prices options as if they follow a geometric Brownian motion, illustrating the opportunities and risks from applying stochastic calculus.
(google訳)
確率微積分の重要な応用例は数理金融であり、そこでは資産価格が確率微分方程式に従うと仮定されることがよくあります。たとえば、ブラック・ショールズ モデルは、あたかも幾何学的なブラウン運動に従うかのようにオプションの価格を設定し、確率計算を適用することによる機会とリスクを示しています

https://ja.wikipedia.org/wiki/%E3%83%95%E3%82%A3%E3%83%BC%E3%83%AB%E3%82%BA%E8%B3%9E
フィールズ賞
2006年(マドリード)
ウェンデリン・ウェルナー(Wendelin Werner, 1968年 - )フランス(ドイツ出身)
「 for his contributions to the development of stochastic Loewner evolution, the geometry of two-dimensional Brownian motion, and conformal field theory

2010年
確率論の数学者判別できず

2014年(ソウル)[19]
マルティン・ハイラー(Martin Hairer, 1975年 - ) オーストリア
「 for his outstanding contributions to the theory of stochastic partial differential equations, and in particular for the creation of a theory of regularity structures for such equations.

2018年
アレッシオ・フィガリ(Alessio Figalli, 1984年 -) イタリア
「 For contributions to the theory of optimal transport and its applications in partial differential equations, metric geometry and probability.

2022年(オンライン開催[注釈 3])[21]
ユーゴー・デュミニル=コパン(Hugo Duminil-Copin, 1985年 - ) フランス
「 For solving longstanding problems in the probabilistic theory of phase transitions in statistical physics, especially in dimensions three and four.
(引用終り)
以上
0304132人目の素数さん
垢版 |
2024/03/07(木) 17:41:04.63ID:sZFxxPrG
>>294
どの設定がどう変わってると?
0306132人目の素数さん
垢版 |
2024/03/07(木) 17:42:09.50ID:C3Ro7iPT
>>293
分布以前に可測関数じゃないのが問題なんだが…
最大の決定番号を持つ列をMとしたら
M=1,...,100でΩを100分割したときに可測集合で分割されない
0307132人目の素数さん
垢版 |
2024/03/07(木) 17:42:18.52ID:4JevvWVM
>>302
そこはわかる
で、「みな、自分だけが外れると思ってる」といいたいんでしょうな
その中で実際に外れるのは一人だけなんだが
0308132人目の素数さん
垢版 |
2024/03/07(木) 17:43:56.42ID:4JevvWVM
>>306
>M=1,...,100でΩを100分割したときに可測集合で分割されない
そもそもR^Nの測度をどうやって決めた?
なぜそれでいいと断言できる?
0309132人目の素数さん
垢版 |
2024/03/07(木) 17:44:03.30ID:sZFxxPrG
ID:G6yRkQAe
無意味なコピペやめろ レスが見づらくなる おまえは荒しか?
0310132人目の素数さん
垢版 |
2024/03/07(木) 17:45:07.74ID:4JevvWVM
>>309
もう、勝ち目がないので、誤魔化したいんでしょう
本当に自分のことしか考えない自己中ですね 1は
0312132人目の素数さん
垢版 |
2024/03/07(木) 17:46:00.50ID:4JevvWVM
>>305
誤 酒
正 ア●ン
0313132人目の素数さん
垢版 |
2024/03/07(木) 17:47:18.38ID:sZFxxPrG
>>305
自分が正しいと信じていれば酒もうまかろう
信じる者は救われる
南無阿弥陀仏
0314132人目の素数さん
垢版 |
2024/03/07(木) 17:48:05.58ID:4JevvWVM
>>311
ガロア理論どころか線形代数も微分積分も分からんかったそうな
確率論も幾何分布もベルヌーイ過程も分からんかった
多分ベイズの公式だけで誤魔化してきた口 算数だな
0315132人目の素数さん
垢版 |
2024/03/07(木) 17:49:26.43ID:sZFxxPrG
>>306
まずΩを書き下してみて
0318132人目の素数さん
垢版 |
2024/03/07(木) 17:51:54.25ID:4JevvWVM
>>316
測度が決まってないのに可測かどうか分かるの?
0319132人目の素数さん
垢版 |
2024/03/07(木) 17:52:36.70ID:sZFxxPrG
>>317
なんだ確率のど素人か
0320132人目の素数さん
垢版 |
2024/03/07(木) 17:52:47.29ID:4JevvWVM
>>317
数学知らんド素人でしたか
0321132人目の素数さん
垢版 |
2024/03/07(木) 17:56:42.42ID:sZFxxPrG
>>317
じゃ任意の確率空間においてΩ={}でいいのね?
0326132人目の素数さん
垢版 |
2024/03/07(木) 18:01:51.66ID:sZFxxPrG
>>322
P(Ω)=1にならないなら任意じゃダメってことじゃんw
おまえ馬鹿だろw
>Ωなんて任意でいいって言ってんだろ
0327132人目の素数さん
垢版 |
2024/03/07(木) 18:03:10.48ID:G6yRkQAe
>>285
 >>284
どこに分布がでてきたの?

>>288
 >>285
1が言い出したことだから1に聞きな
はっきりいって1が当然だと思い込んでる前提を誰も認めてないけどね
(引用終り)

呼んだ? スレ主です

・えーと、箱入り無数目は、下記だった
・まず、箱が1〜n個(有限)のミニモデルを考えよう
 簡単にサイコロの目を入れる
 各箱には、1〜6の整数が等確率で入る。すなわち、1〜6の整数の一様分布です
・さて、n→∞ 可算無限で、時枝さんはある一つの箱の確率が99/100にできるという
(成立派は、これ 100列が確率変数だとかウンヌンカンヌンw)
 百歩譲ってそれを認めたとして、残る可算無限の箱はどうか?
 うん? 箱が1〜n個(有限)のときと同じで、サイコロ 1〜6の整数の一様分布の確率変数?
 だったら、確率が99/100の箱は 突然変異で 確率変数が変わったのかな?w

ことほど左様に、「箱入り無数目」は箱に入れる数の確率分布は、本来は出題者(数を入れる側)の自由という設定で
ここに、分布が出てくる余地ありですね

(参考)時枝記事>>1より
https://imgur.com/a/8bqlb08
数学セミナー201511月号「箱入り無数目」
https://rio2016.5ch.net/test/read.cgi/math/1620904362/401-406
純粋・応用数学(含むガロア理論)8 より
1.時枝問題(数学セミナー201511月号の記事)の最初の設定はこうだった。
「箱がたくさん,可算無限個ある.箱それぞれに,私が実数を入れる.
どんな実数を入れるかはまったく自由,例えばn番目の箱にe^nを入れてもよいし,すべての箱にπを入れてもよい.
もちろんでたらめだって構わない.そして箱をみな閉じる.
今度はあなたの番である.片端から箱を開けてゆき中の実数を覗いてよいが,一つの箱は開けずに閉じたまま残さねばならぬとしよう.
どの箱を閉じたまま残すかはあなたが決めうる.
勝負のルールはこうだ. もし閉じた箱の中の実数をピタリと言い当てたら,あなたの勝ち. さもなくば負け.
勝つ戦略はあるでしょうか?」
0328132人目の素数さん
垢版 |
2024/03/07(木) 18:03:38.88ID:sZFxxPrG
今宵はこれを肴に呑るか

>Ωなんて任意でいいって言ってんだろ
0329132人目の素数さん
垢版 |
2024/03/07(木) 18:05:37.95ID:C3Ro7iPT
>>326
任意の確率空間(Ω,F,P)について、これこれが成り立ちますってのがこちらの主張だから、Ωが∅でも君がそれで満足ならこっちは気にしないよ
0330132人目の素数さん
垢版 |
2024/03/07(木) 18:05:58.24ID:sZFxxPrG
>>327
>・まず、箱が1〜n個(有限)のミニモデルを考えよう
考えても無駄
最後の箱の有無が有限列と無限列で違うから
まだ分からんの?馬鹿?
0331132人目の素数さん
垢版 |
2024/03/07(木) 18:11:38.87ID:sZFxxPrG
>>329
M=1,...,100で{}を100分割ってなに?
0333132人目の素数さん
垢版 |
2024/03/07(木) 18:16:57.91ID:/uk4GHE2
スレが伸びれば数学なんてどうでもいいガロア理論、おまえら踊らされてるんだぜ
0334132人目の素数さん
垢版 |
2024/03/07(木) 18:22:24.45ID:sZFxxPrG
>>332
>は?知らんがな
100分割の言い出しっぺなのに知らないの?

>∅100個並べてろよ
{}を100個並べたものが{}を100分割したものなの?
それ分割って言うの?日本語おかしくね?

>それより∅だと確率空間にならんだろ
確率空間にならないなら任意じゃダメってことじゃん
君馬鹿なの?
0335132人目の素数さん
垢版 |
2024/03/07(木) 18:24:07.36ID:sZFxxPrG
>>334
>>∅100個並べてろよ
じゃΩ={{}}の場合は?
0337132人目の素数さん
垢版 |
2024/03/07(木) 18:25:27.25ID:G6yRkQAe
>>300

追加
・伊藤レンマが、株価の確率的取り扱いと関係して
・「ブラックショールズ方程式で活躍。金融工学の全盛期を支えました」(下記)という話を知らない人がいるんだ

その人の名は、おサル=サイコパス*のピエロ(不遇な「一石」>>8

なんだかね ;p)

(参考)
https://double-growth.com/ddm_ito_001/
みんなの運用会議
株価シナリオのモデルと伊藤のレンマ  by yamamoto
2019年11月26日

株価SがdS/S = μdt +σdZに従うとき、μは年率連続複利のドリフト率でσはその年率の上昇率の変化率の標準偏差ですが、f(S,t)=log Sとしてこれを偏微分すると、
∂f/∂S = S^-1, ∂^2f/∂S^2=-S^-2
になり、一方で
∂f/∂t=0
です。

伊藤レンマ
伊藤レンマを計算すれば、dfが計算できて、
df= (μ-σ^2/2)dt +σdB
が結論になります。ここで(dB)^2=dtとなるのが先生の発見だと思います。それでdtの係数にσ^2/2の項が出てくる。これが投資家にとって極めて重要なのです。

株価が時刻tから時刻Tへと移るときに、Δlog S = log S(T)- log S(t)=(μ-σ^2/2)Δt + σ dBと置けるのですから、
log(S(T)/S(t))が平均(μ-σ^2/2)(T-t)、標準偏差σ(T-t)^0.5の正規分布に従うことになるからです。( – – – A)
この算出は、(dB^2=dt)の部分はそんなに簡単ではありません。S^2が自由度1の平均1で分散2のカイ二乗分布に従い、じゃあ、なぜ、S^2がカイ二乗分布に従うのかという点については、比較的容易にガンマ関数の初歩的な計算と置換積分でわかります。 nF(n)=F(n+1) (nは本来複素数まで拡張できる)
このレンマがブラックショールズ方程式で活躍。金融工学の全盛期を支えました。
0339132人目の素数さん
垢版 |
2024/03/07(木) 18:28:38.67ID:sZFxxPrG
>>338
100分割の定義を示してもらってないのでやり様が無いから聞いてるんだけど?
0340132人目の素数さん
垢版 |
2024/03/07(木) 18:31:44.37ID:sZFxxPrG
>>336
Ωが任意でいいって言ったのは君だよね
確率空間にならないなら任意じゃダメってことだろ
君こそ何言ってんの?
0341132人目の素数さん
垢版 |
2024/03/07(木) 18:32:36.45ID:C3Ro7iPT
丁寧に書けば
任意の確率空間(Ω,F,P)とℝ^ℕ値確率変数Xについて、最大の決定番号を与える関数をm: ℝ^ℕ→{1,2,...,100}とすると、
mは非可測関数で、Xが変に潰れてない限りm(X)も非可測
だよ
0343132人目の素数さん
垢版 |
2024/03/07(木) 18:43:53.53ID:sZFxxPrG
>>341
>最大の決定番号を与える関数をm: ℝ^ℕ→{1,2,...,100}とする
意味不明
任意の実数列に1から100の自然数を対応させる関数がどうして最大の決定番号を与えるの?
0344132人目の素数さん
垢版 |
2024/03/07(木) 18:46:24.59ID:sZFxxPrG
>>341
じゃ例えば m(∀s∈R^N)=1 とする
このmを使って最大の決定番号を与えてみて
0346132人目の素数さん
垢版 |
2024/03/07(木) 18:57:58.37ID:C3Ro7iPT
>>344
mは最大の決定番号を与える関数だって言ってんだろ
記事に構成方法が書いてある関数だよ
0347132人目の素数さん
垢版 |
2024/03/07(木) 19:20:29.74ID:B1UavB2/
箱入り無数目をゲームと考える場合

先手:出題者の出題
後手:回答者の選択する箱の集合

簡単のため、2列とする
∃x1,x2∈R^N ∀n1,n2∈N ¬(x1[n1]=r(x1)[n1])∧¬(x2[n2]=r(x2)[n2])
なら、先手(出題者)必勝
∀x1,x2∈R^N ∃n1,n2∈N (x1[n1]=r(x1)[n1])∨(x2[n2]=r(x2)[n2])
なら、後手(回答者)確率1/2(2つの候補のうち1つ)で勝ち

で、n1=d(x2),n2=d(x1)、とすれば後手が確率1/2で勝てるというのが箱入り無数目
R^Nの測度も、決定番号の確率分布も、出てこない
0349132人目の素数さん
垢版 |
2024/03/07(木) 19:30:39.60ID:C3Ro7iPT
もっと強い以下の主張が証明できますよ
∀x1,x2∈R^N ∃n1∈N (x1[n1]=r(x1)[n1])
なら、後手(回答者)確率1(1つの候補のうち1つ)で勝ち
0350132人目の素数さん
垢版 |
2024/03/07(木) 19:43:29.99ID:B1UavB2/
>>349
あはは、そうだね
でも、n1を具体的に決められないと意味ないよね?
箱入り無数目では、代表を選択する関数さえあれば
d(x1),d(x2)は求められる

>>348
全部ではないが見ることは許されている
そしてその情報だけで
x1を選べばd(x2)
x2を選べばd(x1)
は分かるから後手は戦略を実行できる

ちなみに先手がどんな手を売ってもその瞬間にd(x1)とd(x2)は決まる
つまり、避けようがない
0352132人目の素数さん
垢版 |
2024/03/07(木) 19:53:05.04ID:sZFxxPrG
>>348
>後手は先手の着手を見てから自分の着手を決める
それって至って普通じゃね?w
てかそれがダメなら先手後手の意味ねーじゃんw
0354132人目の素数さん
垢版 |
2024/03/07(木) 20:14:30.43ID:B1UavB2/
>>353
でも全部でなければ開けられる
そして開けなかったところで予測できれば勝ち
だから∃n1,n2の一方が他方の開けた情報から決められれば確率1/2で勝てる
0355132人目の素数さん
垢版 |
2024/03/07(木) 20:45:21.29ID:sZFxxPrG
>>353
日本語でお願いします
0357132人目の素数さん
垢版 |
2024/03/07(木) 23:49:31.64ID:HiCaDQeT
>>333
>スレが伸びれば数学なんてどうでもいいガロア理論、おまえら踊らされてるんだぜ

スレ主です
レスありがとう

ところで、ここに誤解があります
1)このスレの意義は、「箱入り無数目」>>1
 二つの封筒問題や、モンティホールくらいには(下記)
 確率のパラドックスとして解き明かしたいと思っているのです
2)勿論、私は不成立派で、解き明かすべきは
 「箱入り無数目」のトリック解明、当たらないのに なぜ当たるように見えるのか?
 って部分だ
3)もし、「箱入り無数目」が正しいという結論になっても構わないけど
 数学的にきちんと筋の通った議論が必要です
 ”箱の中には確率変数は入れられない。箱の中は定数だ”などという
 小学生レベルの主張はダメですよ(大学レベルの数学の確率変数の意味分かってない)

なので、正しくは『「箱入り無数目」のトリック解明、当たらないのに なぜ当たるように見えるのか?』
の議論を期待しているのです
しかし、「箱入り無数目」成立派のあたまが固いので、スレを無駄に消費している気がしています
これは、私の のぞむところでは ありません

(参考)>>210
https://en.wikipedia.org/wiki/Two_envelopes_problem
Two envelopes problem
2 つの封筒の問題
https://researchmap.jp/blogs/blog_entries/view/92228/98e56ed2a2e2f485f4c22d2bcac369c0?frame_id=526781
2つの封筒問題
投稿日時 : 2014/04/07 関 勝寿

https://ja.wikipedia.org/wiki/%E3%83%A2%E3%83%B3%E3%83%86%E3%82%A3%E3%83%BB%E3%83%9B%E3%83%BC%E3%83%AB%E5%95%8F%E9%A1%8C
モンティ・ホール問題
https://manabitimes.jp/math/989
高校数学の美しい物語
モンティ・ホール問題とその解説 2021/03/07
0358132人目の素数さん
垢版 |
2024/03/08(金) 00:00:48.27ID:q8NdBfwL
>>357
>勿論、私は不成立派
出題列を2列に並べ替えたときの決定番号d1,d2がいかなる自然数なら的中確率が1/2に満たないか答えて下さい
0359132人目の素数さん
垢版 |
2024/03/08(金) 00:11:05.18ID:KN2G6KLz
>>237 補足
>ところが、時枝さん「箱入り無数目」手法では、±1σ→±0 にできる
>と主張する
>これは、さすがに確率論の外でしょう!w

・±1σ→±0 は、連続区間の一点的中ですので、ルベーグ測度0です(下記)
・従って、連続区間の一点的中は ルベーグ測度では 99/100は正当化できないのです

これは、「箱入り無数目」不成立の傍証です

(参考)
https://ja.wikipedia.org/wiki/%E3%83%AB%E3%83%99%E3%83%BC%E3%82%B0%E6%B8%AC%E5%BA%A6
ルベーグ測度

・可算集合のルベーグ測度は必ず 0 である。
https://manabitimes.jp/math/2728
高校数学の美しい物語
ルベーグ測度2023/05/11
いくつかの例
・1点集合
1点集合 {p}(p∈R)で外測度を計算してみましょう。
μ ∗ ({p})=0 です。
0360132人目の素数さん
垢版 |
2024/03/08(金) 00:14:52.29ID:q8NdBfwL
>>357
>”箱の中には確率変数は入れられない。箱の中は定数だ”などという
> 小学生レベルの主張はダメですよ
箱の中身は定数だけど、仮に箱の中身を確率変数にできるとして、
箱の中身を確率変数にしなければ確率1-εで的中できるのだから、
「箱の中身を確率変数とできる」に何の意味も無い。

>大学レベルの数学の確率変数の意味分かってない
大学レベルの数学は「箱の中身を確率変数としなければならない」なんて言ってません。あなたの独善妄想です。
違うと言うならそう書かれている書籍を示して下さい。
0361132人目の素数さん
垢版 |
2024/03/08(金) 00:18:05.49ID:q8NdBfwL
>>359
間違いに補足しても正しくなることはありません
0362132人目の素数さん
垢版 |
2024/03/08(金) 00:20:14.64ID:lFqloK7L
悪魔の証明が好きなメンヘル婆、正しくないことを証明してください
0363132人目の素数さん
垢版 |
2024/03/08(金) 00:23:37.36ID:q8NdBfwL
間違いを正しくするには変更するしかない 補足ではダメ

証明?おまえがやれば?
0364132人目の素数さん
垢版 |
2024/03/08(金) 00:26:50.93ID:HdP6EaIr
先頭に∀がついてる定式化で議論しても時間のムダ
ゴミみたいな結果が出るだけ
0365132人目の素数さん
垢版 |
2024/03/08(金) 00:38:36.32ID:q8NdBfwL
確率1-εで的中できることが証明されている
不服なら証明のギャップか反例(>>358)を示せ
屁理屈並べても無駄
0367132人目の素数さん
垢版 |
2024/03/08(金) 00:54:15.19ID:q8NdBfwL
>>366
それってあなたの感想ですよね?
0368132人目の素数さん
垢版 |
2024/03/08(金) 05:08:28.34ID:lFqloK7L
しれっと書くペテン師、ハーバードの応用数学者
R^N/〜の代表系を選んだところで選択公理を使っている。その結果R^N->R^N/〜の切断は非可測になる
0369132人目の素数さん
垢版 |
2024/03/08(金) 05:12:26.48ID:lFqloK7L
ペテン師
同値類の中で勝つ戦略です。しかも問題の列が100個です。
非可測集合の中で確率を計算する。
0370132人目の素数さん
垢版 |
2024/03/08(金) 05:46:51.88ID:pedAqHFZ
>>354 >∀が先頭にあるってことは全部開ける許可を与えてることだろ
>>364 >先頭に∀がついてる定式化で議論しても時間のムダ
>>366 >先頭に∀ついてる時点でゴミ

全部誤解
アヘン喫ってる?

∀x∃yで言ってることは、
先手(出題者)がどんな手xを売っても
後手(回答者)が(xに合わせて)手yを打つことができ
箱入り無数目では、たかだか1つを除いて成功するってこと

逆に
∃x∀yでいえるのは
先手(出題者)が手xを打ったら
後手(回答者)がどんな手を打っても
箱入り無数目では、ほとんど全て失敗するってことだが
残念ながら、そんな手はない
0371132人目の素数さん
垢版 |
2024/03/08(金) 05:54:17.90ID:lFqloK7L
そうそう、同値類の中も変な集合
sの目が無限に0でないとすると任意のε≠0平行移動すると同値類の外へでる
sの目がある番号から先0、つまり有限の箱の目を当ててるような気がする
同値類の中は決定番号で可算個の直和にかけるけど平行移動不変とは限らない、痩せ集合のような気がする
0372132人目の素数さん
垢版 |
2024/03/08(金) 06:01:40.89ID:pedAqHFZ
>>371
半端な知識で考えて間違う典型例

箱入り無数目のポイントは、
いかにして、代表列の対応する項と中身が異なる箱を避けるか

ちなみに無限列の分布とかあらかじめ妄想するのは●違いの第一歩
0373132人目の素数さん
垢版 |
2024/03/08(金) 07:17:56.43ID:q8NdBfwL
>>368
R^N->R^N/〜の切断が非可測だとなぜペテンになるの?
0374132人目の素数さん
垢版 |
2024/03/08(金) 07:21:16.62ID:q8NdBfwL
>>369
>非可測集合の中で確率を計算する。
大間違い
確率計算は以下の通り非可測集合とは一切関係ありませんよ?
「さて, 1〜100 のいずれかをランダムに選ぶ. 例えばkが選ばれたとせよ. s^kの決定番号が他の列の決定番号どれよりも大きい確率は1/100に過ぎない. 」

それともR^N->R^N/〜の切断が非可測だと決定番号の大小比較ができないとでも言うんですか?自然数だから全順序なのに
0375132人目の素数さん
垢版 |
2024/03/08(金) 07:30:37.66ID:q8NdBfwL
箱入り無数目を間違う人の特徴

記事を読んでいない・読めていない
論理的思考ができない
独善持論を持ち出す
0376132人目の素数さん
垢版 |
2024/03/08(金) 08:38:10.07ID:lFqloK7L
1.時枝問題(数学セミナー201511月号の記事)の最初の設定はこうだった。
「箱がたくさん,可算無限個ある.箱それぞれに,私が実数を入れる.
どんな実数を入れるかはまったく自由,例えばn番目の箱にe^nを入れてもよいし,すべての箱にπを入れてもよい.
もちろんでたらめだって構わない.そして箱をみな閉じる.
今度はあなたの番である.片端から箱を開けてゆき中の実数を覗いてよいが,一つの箱は開けずに閉じたまま残さねばならぬとしよう.
どの箱を閉じたまま残すかはあなたが決めうる.
勝負のルールはこうだ. もし閉じた箱の中の実数をピタリと言い当てたら,あなたの勝ち. さもなくば負け.
勝つ戦略はあるでしょうか?」
0377132人目の素数さん
垢版 |
2024/03/08(金) 08:43:07.91ID:lFqloK7L
1.時枝問題(数学セミナー201511月号の記事)の実際の答え
箱の中味はある番号から先が0で100列あります。同じ同値類から答えを選べば99/100以上の確率で当るかもしれない
0379132人目の素数さん
垢版 |
2024/03/08(金) 08:46:56.39ID:lFqloK7L
>>376
ところがところが---本記事の目的は、確率99%で勝てそうな戦略を戦略を提供することにある。
0380132人目の素数さん
垢版 |
2024/03/08(金) 08:48:41.83ID:q8NdBfwL
ほらね

記事を読んでいない・読めていない
論理的思考ができない
独善持論を持ち出す
0381132人目の素数さん
垢版 |
2024/03/08(金) 09:01:44.25ID:TwTSIzlk
>>376
『箱それぞれに,私が実数を入れる.
どんな実数を入れるかはまったく自由,…
もちろんでたらめだって構わない.』

上記の文章から
1.箱の中の実数は一様分布
2.それぞれの箱は独立同分布
という「全く書かれてない」条件を読み取るのが1

精神医学ではパラノイアもしくは妄想性人格障害という
世間一般では思い込みの激しい人という
0382132人目の素数さん
垢版 |
2024/03/08(金) 09:06:05.63ID:TwTSIzlk
>>377
>箱の中味はある番号から先が0で100列あります。

とはいってないが、仮にそうなら話が早い
その場合、どの100列も、「全部の項が0の列」と尻尾同値
したがって唯一の同値類から答えの候補となる代表として「全部の項が0の列を」選べば
99/100以上の確率で「中身が0の箱を」当てられる
0383132人目の素数さん
垢版 |
2024/03/08(金) 09:11:40.67ID:TwTSIzlk
回答者が尻尾同値類の選択関数を持っている、ということは
ほとんどすべての回答を持っている、ということである
そして
回答者が1箱を除いて他のすべての箱を開ける、ということは
ほとんどすべての出題を見ている、ということである

つまり
ほとんどすべての出題を見て
ほとんどすべての回答を得る
肝心なのは見てない一箱について
回答が一致してるのかしてないのか

実は100列の場合、「たかだか1箱だけ不一致の100箱」がとれる
それが箱入り無数目 だから後手が確率1-1/100で勝てる
0384132人目の素数さん
垢版 |
2024/03/08(金) 09:14:59.15ID:TwTSIzlk
ここで重要なのは100列から「たかだか1箱だけ不一致の100箱」は確実に(つまり確率1で)取れる
その100箱から、一致する箱をとる確率が1-1/100だというだけ
つまり、100列それぞれについて
「選ばれた1箱が不一致となる(つまり他の99列より決定番号が大きい)確率」
なんて計算する必要はない
0387132人目の素数さん
垢版 |
2024/03/08(金) 10:27:27.28ID:MhH+/eu1
>>370
違法薬物に関する投稿者をアク禁にするよう、要請を出しました
繰り返せば、アク禁(永久追放)だろう
0389132人目の素数さん
垢版 |
2024/03/08(金) 14:01:46.42ID:t2p4H4Pu
結局、>>370でターンエー君は●首されちゃいましたね
0390132人目の素数さん
垢版 |
2024/03/08(金) 14:04:29.07ID:t2p4H4Pu
後手の勝利を覆すには、後手が何をやっても無理な最強の出題を提示するしかない
最初の∃を示すターンイー君はいつ現れるのかな?
0391132人目の素数さん
垢版 |
2024/03/08(金) 14:25:17.90ID:q8NdBfwL
出題としてしまうと後手の100列の作り方や代表の選び方に影響されてしまうので、100列の決定番号を直接指定してもよい
100列の決定番号がどんな100個の自然数の組なら後手の勝率が99/100に満たないか、それを示せばよい
だが不成立派は誰一人示そうとしないw
そもそも存在しないから示せるはずもないのだがw
0392132人目の素数さん
垢版 |
2024/03/08(金) 14:34:47.11ID:q8NdBfwL
自然数の全順序性から単独最大決定番号が複数になることはありえない
単独最大決定番号の列を選んだ場合だけ後手の負けだから勝率は99/100以上

選択公理を認めて決定番号がwell-definedとなった瞬間にこの簡単・単純な論理が成立してしまう
いい加減理解しようぜ?不成立派の諸君
0393132人目の素数さん
垢版 |
2024/03/08(金) 15:35:31.02ID:lFqloK7L
1つの同値類Cをとって同値類の中を決定番号dで直和に分ける:C=∪{E(d)|d=1,2,・・・}
濃度は
E(1)は代表元r一個
E(2)はℵ1-1個
E(3)はℵ1(ℵ1-1)個
・・・
E(d)は(ℵ1)^(d-2)((ℵ1-1)個

#E(1)<#E(2)<#E(3)<・・・
0394132人目の素数さん
垢版 |
2024/03/08(金) 15:43:44.36ID:q8NdBfwL
決定番号の分布なんて一切使わずに99/100が言えるからナンセンス
0395132人目の素数さん
垢版 |
2024/03/08(金) 16:03:53.07ID:xiXzr983
>>394 決定番号の分布? 必要ないけど

∀x1,...,x100∈R^N ∃n1,…,n100∈N (xi[ni]=r(xi)[ni]でないxiはたかだか1つ)
がいえるから、x1,...,x100からxi[ni]=r(xi)[ni]となる99個のxiのうちのどれかを選ぶ確率は99/100
(完)
0396132人目の素数さん
垢版 |
2024/03/08(金) 16:07:37.12ID:xiXzr983
2つ以上の要素をもつ任意の集合Sについて
∀x1,...,x100∈S^N ∃n1,…,n100∈N (xi[ni]=r(xi)[ni]でないxiはたかだか1つ)
がいえるから、x1,...,x100からxi[ni]=r(xi)[ni]となる99個のxiのうちのどれかを選ぶ確率は99/100

この時点で、各xi[ni]の確率分布とか、決定番号の分布とか、全然無関係と分かる
0397132人目の素数さん
垢版 |
2024/03/08(金) 16:12:13.51ID:m7A92kIK
各xi[ni]の確率分布とか、独立とかにこだわると、決定番号とか全く考えずに
「他の箱の情報がいくらわかっても、選んだ箱の中身と全く独立だから、
 いくら代表とかとってきても全く関係なく、Rの場合、xi[ni]=r(xi)[ni]となる確率は0」
といいきっちゃう

実は全然無関係の別の問題だと誤解してることに全く気付けない 
これはもう精神の不自由性に起因する病
0398132人目の素数さん
垢版 |
2024/03/08(金) 16:18:27.23ID:xiXzr983
xi[ni]=r(xi)[ni]となる確率は0、といっちゃう人は
xi[ni]がr(xi)[ni]以外の別の値をとる場合と比較してるが、それが誤り

比べるのは別の箱xj[nj]
100個が100個とも不一致、ということはない
不一致のはたかだか1個で、少なくとも99個は一致するのである

分母が間違ってるから間違った結果がでる
0399132人目の素数さん
垢版 |
2024/03/08(金) 16:32:38.85ID:lFqloK7L
自然数d1,・・・,d100を比較した場合なら一様分布なので場合の数/全体の数が確率になるが、dが決定数の場合は違うだろ
以前に指摘したはずだが
0400132人目の素数さん
垢版 |
2024/03/08(金) 16:36:16.84ID:t2p4H4Pu
>>399
決定番号は自然数 一様分布? 必要ないよ
100個だろうが10000個だろうが、他より大きな数はたかだか1つしかない
だからn個中、その1つを選ばない確率は1−1/n
さんざん指摘したはずだが全く理解できなかったのかね

君、自然数が全順序集合だって知らないの?
0401132人目の素数さん
垢版 |
2024/03/08(金) 16:55:36.61ID:rMmJzcaR
>>370
こいつ自分で、∀の手は全公開で箱の中に入れてないって白状してんじゃん
バカなのか…
0402132人目の素数さん
垢版 |
2024/03/08(金) 16:58:29.54ID:lFqloK7L
誤り
ところがところが---本記事の目的は、確率99%で勝てそうな戦略を戦略を提供することにある。
0403132人目の素数さん
垢版 |
2024/03/08(金) 17:01:34.65ID:lFqloK7L
>>402
字句訂正
ところがところが---本記事の目的は、確率99%で勝てそうな戦略を提供することにある。
0405132人目の素数さん
垢版 |
2024/03/08(金) 17:18:20.91ID:WBw2d/cm
>>401 >∀の手は全公開で箱の中に入れてない
まーた、ターンエー君は、幻聴が聞こえちゃったんですか
∀の文字を見ると「全公開だ!箱の外だ!」と脊髄反射しちゃいますねー
0406132人目の素数さん
垢版 |
2024/03/08(金) 17:19:58.13ID:WBw2d/cm
>>404
誤 酒がうめー
正 ア●ンがキモチええ

ハイ人 ID:lFqloK7L 誕生
0409132人目の素数さん
垢版 |
2024/03/08(金) 17:24:43.25ID:t2p4H4Pu
>>407-408 そんなに自分を蔑みなさんな ID:lFqloK7L
0413132人目の素数さん
垢版 |
2024/03/08(金) 17:54:48.40ID:q8NdBfwL
今日は一段と馬鹿で溢れてるな
春だからかな
0414132人目の素数さん
垢版 |
2024/03/08(金) 18:01:30.61ID:q8NdBfwL
ℵ_1って可算無限の次に大きい濃度?
例えばどんな集合がℵ_1?
0415132人目の素数さん
垢版 |
2024/03/08(金) 18:02:17.16ID:MhH+/eu1
>>410-411
これは、弥勒菩薩様か。スレ主です
ありがとうございます。
弥勒菩薩様の一撃は強烈ですね

無知な数学オチコボレの亡者2人を
弥勒菩薩様の力でお救いください
アーメン

このスレの目的は
『「箱入り無数目」のトリック解明、当たらないのに なぜ当たるように見えるのか?』
です。亡者2人の幼稚な議論は無用です。二人が居なくなればスレのバトルも無用になります

(参考)>>210
https://en.wikipedia.org/wiki/Two_envelopes_problem
Two envelopes problem
2 つの封筒の問題
https://researchmap.jp/blogs/blog_entries/view/92228/98e56ed2a2e2f485f4c22d2bcac369c0?frame_id=526781
2つの封筒問題
投稿日時 : 2014/04/07 関 勝寿

https://ja.wikipedia.org/wiki/%E3%83%A2%E3%83%B3%E3%83%86%E3%82%A3%E3%83%BB%E3%83%9B%E3%83%BC%E3%83%AB%E5%95%8F%E9%A1%8C
モンティ・ホール問題
https://manabitimes.jp/math/989
高校数学の美しい物語
モンティ・ホール問題とその解説 2021/03/07
0416132人目の素数さん
垢版 |
2024/03/08(金) 19:17:45.43ID:pedAqHFZ
>>415
>無知な数学オチコボレの亡者2人
1とターンエーか

>このスレの目的は

『「箱入り無数目」のメカニズム解明、当たるのに なぜ当たらないと誤解するのか?』

亡者の一人1は、勝手にR^N上の一様分布を前提し
その上で、R^N上の決定番号の分布が”非正則”だから確率0だとかいう、
測度論の初歩(可算加法性)すら否定する大馬鹿な誤りを声高に叫ぶ

亡者のもう一人ターンエーは∀xが先頭だと確率変数だの全部丸見えだのと
そのときどきでいうことがコロコロ変わる醜態ぶり
まあ、具体的手順なしに選択公理で代表が選ばれるというのが理解できないんでしょう 
要するに論理が全然わかってない 大学数学全滅のド素人

まあ、この二人が自分の誤りを理解できるほど賢くなることは死ぬまでないでしょう

南無阿弥陀仏
0417132人目の素数さん
垢版 |
2024/03/08(金) 19:54:52.48ID:rMmJzcaR
>>416
先頭に∀を付けたモデルだけ考えて、それで満足してるなら勝手にすればいいだろ
他の人はみんなトリックがどういう理屈で働いてるかの話をしてるんだから邪魔すんな
0418132人目の素数さん
垢版 |
2024/03/08(金) 20:52:06.07ID:pedAqHFZ
>他の人はみんなトリックがどういう理屈で働いてるかの話を
してないね 私ともう一人以外はね 見当違いの発言しかしてない
非可測ガーとか、非正則ガーとか、先頭の∀ガーとか、
アタマ冷やせよ 自分が大数学者だとでも思ってんのか?
0420132人目の素数さん
垢版 |
2024/03/08(金) 22:11:25.78ID:q8NdBfwL
>>417
まあトリックと言えるか知らんが
無限個の箱
選択公理
くらいかな どっちもZFCでは空気みたいな存在だが

考える対象が無限の場合、有限しか直観できない人間には直観に合わないことが現れる
だから直観頼りの無学者は受け入れ難く感じる それだけのこと
0421132人目の素数さん
垢版 |
2024/03/08(金) 22:42:12.72ID:rMmJzcaR
>>420
それだと結論として情報が漏れてることが分かるだけじゃん
如何にして情報が漏れてるかを考察しないとつまらん
0422132人目の素数さん
垢版 |
2024/03/08(金) 23:00:51.39ID:q8NdBfwL
>>421
考察も何も記事に全部書かれてるよ
君が読めてないだけ
0424132人目の素数さん
垢版 |
2024/03/08(金) 23:37:37.93ID:lFqloK7L
箱入り無数目の定義、定理すらけかない度素人が数学やってるつもり
0428132人目の素数さん
垢版 |
2024/03/08(金) 23:58:08.63ID:lFqloK7L
問題を定式化するのも数学、定式化せずに解けるのは高校数学の延長にあるものだけだよ
0429132人目の素数さん
垢版 |
2024/03/09(土) 00:27:19.91ID:fy/7ggA0
あの記事が読めないんじゃ数学以前だね
諦めた方が良い
0432132人目の素数さん
垢版 |
2024/03/09(土) 00:49:19.44ID:fy/7ggA0
>>430
ペテン?
じゃあ記事のどこがどう間違ってるか指摘したら?
ハッタリくんには無理かな?
0433132人目の素数さん
垢版 |
2024/03/09(土) 00:50:52.63ID:fy/7ggA0
>>431
ハッタリくん
記事の間違い箇所を指摘できず今宵も酒で現実逃避
0435132人目の素数さん
垢版 |
2024/03/09(土) 01:26:05.27ID:iC63zgm6
そもそも情報自体にすら言及がない
σ-algで情報を表現したらどうなるか考えるとした場合、先頭に∀がある時点で無理
0436132人目の素数さん
垢版 |
2024/03/09(土) 01:27:32.55ID:fy/7ggA0
情報漏洩?なにそれw
0437132人目の素数さん
垢版 |
2024/03/09(土) 01:35:51.69ID:fy/7ggA0
>>435
君の言う情報が何か知らんが、言及が無いってことは証明に不要ってことだよw
不要なものを持ち出すのは馬鹿

非可測ガー 決定番号の分布ガー 確率過程論ガー 確率変数の無限族ガー IIDガー 先頭の∀ガー ミニモデルガー 宝くじガー 地球ガー

馬鹿が持ち出した不要なもの
0438132人目の素数さん
垢版 |
2024/03/09(土) 01:42:07.16ID:fy/7ggA0
>>435
>σ-algで情報を表現したらどうなるか考えるとした場合、先頭に∀がある時点で無理
「君の方法論が通用しない対象は間違い」は妄想
世界は君を中心に回っている訳ではない
0440132人目の素数さん
垢版 |
2024/03/09(土) 01:45:31.12ID:fy/7ggA0
証明に不要って書いたんだけど文盲なのかな?
0441132人目の素数さん
垢版 |
2024/03/09(土) 01:47:16.75ID:fy/7ggA0
で?
記事の間違い箇所の指摘はまだ? やっぱハッタリ?
0442132人目の素数さん
垢版 |
2024/03/09(土) 01:51:47.14ID:iC63zgm6
情報の漏洩の仕方を議論するには必要だろ
やっぱ記事には書いてないんじゃん
0443132人目の素数さん
垢版 |
2024/03/09(土) 01:55:35.90ID:4SY0KF1u
時枝@ハーバード大は同様に確からしいと条件付確率が分からなかった
なので度素人が分からないのは仕方がない
0444132人目の素数さん
垢版 |
2024/03/09(土) 02:03:14.50ID:fy/7ggA0
だからおまえの言う情報漏洩ってなんだよw
0445132人目の素数さん
垢版 |
2024/03/09(土) 02:05:18.04ID:fy/7ggA0
箱の中身を見ることか?
後手は一つの箱を除きすべての箱の中身を見ることができるぞ?
閉じたままにする箱は後手が決めてよいぞ?
ぜんぶ記事に書かれてるじゃん どこ読んでんだよw
0446132人目の素数さん
垢版 |
2024/03/09(土) 02:06:29.17ID:fy/7ggA0
記事が読めないって数学以前だなおまえ
なんで数学板に居るんだよ
0448132人目の素数さん
垢版 |
2024/03/09(土) 02:28:37.87ID:fy/7ggA0
>>447
なんだおまえ箱入り無数目を一つも理解してなかったんだな
相手して損した アホくさ
0449132人目の素数さん
垢版 |
2024/03/09(土) 02:36:37.04ID:iC63zgm6
>>448
結局、∀が外側にあるモデルじゃ漏洩自体も記述できねーじゃん
外側にある∀で最初から全部公開してるんだから情報の漏洩を問題として見てすらもいないんだろ
0450132人目の素数さん
垢版 |
2024/03/09(土) 02:38:16.99ID:fy/7ggA0
>>449
漏洩していないものは記述しようが無いw
0451132人目の素数さん
垢版 |
2024/03/09(土) 02:39:30.19ID:fy/7ggA0
>>449
量化と公開状態は何の関係も無いことがまだ理解できんのか?
アホやのう
0452132人目の素数さん
垢版 |
2024/03/09(土) 02:41:26.38ID:iC63zgm6
しばらくこれで飯食べるわ

451 132人目の素数さん 2024/03/09(土) 02:39:30.19 ID:fy/7ggA0
>449
量化と公開状態は何の関係も無いことがまだ理解できんのか?
アホやのう
0453132人目の素数さん
垢版 |
2024/03/09(土) 02:44:56.25ID:fy/7ggA0
>>452
てかなんで関係あると思ったの?
どこにそんなデマ書いてあった?
0455132人目の素数さん
垢版 |
2024/03/09(土) 06:56:53.91ID:RwepsQi7
今日のア●ン中毒患者★と医師☆の会話

★=ID:iC63zgm6
☆=ID:fy/7ggA0

★2024/03/09(土) 01:13:19.54 記事には全く説明されてない情報の漏洩で飯でも食うか
☆2024/03/09(土) 01:27:32.55 情報漏洩?なにそれ
☆2024/03/09(土) 01:35:51.69 君の言う情報が何か知らんが、言及が無いってことは証明に不要ってことだよ
★2024/03/09(土) 01:51:47.14 情報の漏洩の仕方を議論するには必要だろ やっぱ記事には書いてないんじゃん
☆2024/03/09(土) 02:03:14.50 だからおまえの言う情報漏洩ってなんだよ
☆2024/03/09(土) 02:05:18.04 箱の中身を見ることか?後手は一つの箱を除きすべての箱の中身を見ることができるぞ?
★2024/03/09(土) 02:08:35.30 開けてない箱の中身が当たるんだから情報が漏洩してんだろ

「開けてない箱の中身が当たる」情報漏洩の話なら、記事にバッチリ書いてあるけど
ーーーーーーーーーーーーーーーーーーーーーーーーーーーーーーーーーーーー
更に,何らかの事情によりdが知らされていなくても,
あるD>=d についてsD+1, sD+2,sD+3,・・・が知らされたとするならば,
それだけの情報で既に r = r(s)は取り出せ, したがってd= d(s)も決まり,
結局sd (実はsd,sd+1,・・・,sD ごっそり)が決められることに注意しよう.
ーーーーーーーーーーーーーーーーーーーーーーーーーーーーーーーーーーーー

σ-algで情報を表現? どうやって?
外側にある∀で最初から全部公開? なんだその妄想
0456132人目の素数さん
垢版 |
2024/03/09(土) 07:37:30.76ID:fy/7ggA0
>>454
つまり君の妄想が生み出した独善持論ってことね?
0457132人目の素数さん
垢版 |
2024/03/09(土) 07:58:43.08ID:RwepsQi7
【結論】
ア●中のターンエー君は「箱入り無数目」記事が全然読めてませんでしたぁ!
(完)
0458132人目の素数さん
垢版 |
2024/03/09(土) 08:45:51.51ID:3L5u7doY
>>447
>開けてない箱の中身が当たるんだから情報が漏洩してんだろ

ありがとうございます
スレ主です

それ賛成です
・箱の中身が、iidである確率事象に従って実数が入れられているとすれば
 一つを除いて、他を開けて統計処理をすれば、平均と標準偏差が求まる
 その情報をもとに、区間[a,b]なら1σで、広げて区間[a',b']なら2σとできる
・一方、箱にカメラが仕込んであれば、箱の中の情報が分かる
 開けてない箱の中身が当たる
 即ち、情報漏洩です

時枝さんの「箱入り無数目」は、見せかけで当たるふり(下記)
実際は、当たらない

(参考)時枝記事>>1
https://imgur.com/a/8bqlb08
数学セミナー201511月号「箱入り無数目」
0459132人目の素数さん
垢版 |
2024/03/09(土) 08:59:22.23ID:fy/7ggA0
証明のギャップも反例も示さずに直観で当たらない当たらないと繰り返す
これが不成立派
数学になってないw
0460132人目の素数さん
垢版 |
2024/03/09(土) 09:42:50.42ID:3L5u7doY
まず、前振り
>>415 より
https://researchmap.jp/blogs/blog_entries/view/92228/98e56ed2a2e2f485f4c22d2bcac369c0?frame_id=526781
2つの封筒問題 2014/04/07 関 勝寿

1.2つの封筒があり、それぞれにお金が入ってます。片方の封筒に入っている金額が、もう片方の封筒に入っている金額の2倍となっていることが分かっています。
 あなたは、最初にどちらか片方の封筒を選び、中身を見る事ができます。その後、改めてどちらの封筒を選ぶか決めることができます。二度目に選んだ封筒の中身をもらうことができます。

 最初の封筒に1万円入っていました。この時、封筒を交換する方が得か、交換しない方が得か、あるいはどちらでも同じか?
 最初に選んだ封筒を封筒Aとすると、ランダムに封筒を選んだことから、封筒Aが金額の小さい封筒である確率は1/2、金額の大きい封筒である確率は1/2です。すると、もう片方の封筒Bに入っている金額は、1/2の確率で2万円、1/2の確率で5000円となります。したがって、封筒Bに入っている金額の期待値は 1/2*20000+1/2*5000=12500 より、12500円となります。封筒Aを封筒Bに交換する事で、期待値が2500円増えますから、交換する方が得です。

ゲームのルールに関する補足
封筒を開けた後に「封筒を変えてもいいよ」と言われる、というバージョンをよく見かけますが、その場合には、自分が大きい金額の封筒を取った場合にのみ「封筒を変えてもいいよ」と言われる、小さい金額の封筒を取ったらそのように言われなかった、という可能性があります。そのような可能性を考えて「変えてもいいよと言われたからには、封筒を変えない方がいい」という回答が成立します。封筒を開ける前から、封筒の交換が許される事をルールとして設定しておくことで、そのような可能性について考える必要はなくなり、純粋にパラドックスの部分を議論することができます。

期待値計算はこれでいいのか?
2.(1)の議論とはまた別に、そもそもこの期待値計算はおかしいんじゃないの?という議論がされることもあります。では、こう考えたらどうでしょうか。封筒を開けて1万円得た時点で、残りの封筒には2万円入っている確率と5000円が入っている確率が等しくなっている。そこで、その1万円を払って、コインを投げて、表が出たら2万円をもらい、裏が出たら5000円をもらう、という賭けをするかどうか。こういう条件だったら、1万円を出して期待値12500円の賭けをするから得だ、だからその賭けをしよう、という期待値計算は問題ないように思えます。だとすると、封筒問題はそれとはどう違うのでしょうか?

つづく
0461132人目の素数さん
垢版 |
2024/03/09(土) 09:43:01.95ID:3L5u7doY
つづき

さいころによる説明
封筒に入れる可能性のある金額をしぼって、問題を単純化します。さいころの各面に 100, 200, 400, 800, 1600, 3200 と書いて、振って出た金額とその目の2倍を封筒に入れます。この時に、片方の封筒を開けて
1.封筒の中身が100円ならば、封筒を換えて200円になる。
2.封筒の中身が6400円ならば、封筒を換えない。
3.(1) (2) 以外であれば、換えることで期待値が1.25倍になる。
封筒を換えない方が得なのは(2)の一通りだけですが、(2)で封筒を換えることで期待値がぐんと(3200円も)下がります。
封筒を換えない場合、換える場合の期待値を正確に計算すると、それぞれ18900円と等しくなり、封筒を換えても換えなくても同じになります。
このように、封筒に入れる可能性のある金額とその確率を明確に定義づけると、多くの場合は矛盾なく説明ができます。「多くの場合は」としたのは、次のバリエーションBが例外として考えられるためです。
略す
(引用終り)

さて、本題
上記”さいころによる説明”のように 金額の分布が影響するという話です
a)”さいころによる説明”の場合、封筒の中の金額が分布を持ちます
 その結果は、上記説明の通りです
b)”期待値計算はこれでいいのか?”の2項の場合は、5000円、1万円、2万円が等確率として
 1万円を見たら、変える方が得になります
 2万円を見たら、変えない方が得になります
c)さて、そもそもは 5000円、1万円のみが等確率で入るとすると
 1万円を見たら、変えない方が得になります
 これは”5000円、1万円が等確率”を知っている場合です
 では、”5000円、1万円が等確率”を知らないとして
 封筒Aを見ると、5000円 or 1万円が分かります
 このそれぞれで、封筒を変える or 変えない の場合分けをすると
 結局、期待値は7500円です(下記補足)

よって、封筒問題を、A,B二つで Aの金額をa、Bの金額をbとしたとき
確率 a<b がどうかと読み替えて
その確率は、1)封筒に入る金額の分布、2)開ける封筒Aの金額 の二つの要素で決まるのです
よって、「箱入り無数目」でも、分布は重要です!
(”分布つかってない”は、”分かってない”と同義です)

補足:具体的には下記
1万円:変えない=1万円、変える=5000円
5千円:変えない=5000円、変える=1万円
4通りで総計3万円 3万/4=7500円
以上
0463132人目の素数さん
垢版 |
2024/03/09(土) 09:54:43.92ID:fy/7ggA0
>>460
>封筒Aが金額の小さい封筒である確率は1/2、金額の大きい封筒である確率は1/2です。
正しい

>すると、もう片方の封筒Bに入っている金額は、1/2の確率で2万円、1/2の確率で5000円となります。
間違い
そんな前提は無い
前提に無いことを勝手に仮定したら間違う
0464132人目の素数さん
垢版 |
2024/03/09(土) 11:20:05.40ID:fy/7ggA0
>>460
「2つの封筒があり、それぞれにお金が入ってます。片方の封筒に入っている金額が、もう片方の封筒に入っている金額の2倍となっていることが分かっています。」
封筒の中身は2種類である。

「最初の封筒に1万円入っていました。」
その2種類とは{1万円、5千円}か{1万円、2万円}かのいずれか。
どちらかは不明だが、封筒の中身は最初に定まっているからどちらか一方。

「最初に選んだ封筒を封筒Aとすると、ランダムに封筒を選んだことから、封筒Aが金額の小さい封筒である確率は1/2、金額の大きい封筒である確率は1/2です。」
正しい

「もう片方の封筒Bに入っている金額は、1/2の確率で2万円、1/2の確率で5000円となります。」
封筒の中身は2種類なのに3種類登場させるのはおかしい。
「1/2の確率で2万円、1/2の確率で5千円」は誤りで、正しくは「1の確率で2万円、0の確率で5千円」か「0の確率で2万円、1の確率で5千円」のいずれか。

「したがって、封筒Bに入っている金額の期待値は 1/2*20000+1/2*5000=12500 より、12500円となります。」
したがってこの期待値計算は誤り。

「封筒Aを封筒Bに交換する事で、期待値が2500円増えますから、交換する方が得です。」
したがってこの結論も誤り。
0465132人目の素数さん
垢版 |
2024/03/09(土) 12:48:47.67ID:3L5u7doY
>>464
>「もう片方の封筒Bに入っている金額は、1/2の確率で2万円、1/2の確率で5000円となります。」
>封筒の中身は2種類なのに3種類登場させるのはおかしい。
>「1/2の確率で2万円、1/2の確率で5千円」は誤りで、正しくは「1の確率で2万円、0の確率で5千円」か「0の確率で2万円、1の確率で5千円」のいずれか。

具体例で説明しよう

1)ある大学において 学生の奨励として、学長賞で賞金を出すことにした
 1年に10回(夏休み8月とクリスマス休暇の12月を除く)、学年のトップ(1番の人)に
 封筒は二つ使う。そして組合わせが二つ、{5千円、1万円}と{1万円、2万円}と
 どの組合わせを使うかは、ランダムで等確率として、学長のみが知る
 授賞式の事務員は知らない
2)事務員がルールを説明する
「封筒二つで、片方の倍か半分かで。一つの封筒を開けて見て良い。別の封筒に取り替える権利がある。
 もちろん、取り替えないのも可」と
 但し、具体的金額は教えない(説明する事務員も知らない)
3)この場合
 開けた封筒が、1万円ならば
 {5千円、1万円}と{1万円、2万円}が等確率で考えられる
 従って、取り替えると 5千円と2万円が等確率で出現するので
 期待値は、1万2千500円です
4)この確率は、賞金をもらう学生は知らないが 多数例を統計処理すれば
 各金額と期待値は計算できて、期待値1万2千500円は出せる

この例の教訓
1)封筒の金額の分布が重要(よって、「分布は使ってない}という言い訳は通用しない!)
2)開けた封筒は確率ではない。開けていない封筒は確率。両者は峻別されるべき!
以上
0466132人目の素数さん
垢版 |
2024/03/09(土) 12:57:42.52ID:fy/7ggA0
>>465
>具体例で説明しよう
封筒問題が具体例なので他を持ち出す必要無し
持ち出しても間違うだけ

>封筒は二つ使う。そして組合わせが二つ、{5千円、1万円}と{1万円、2万円}と
>どの組合わせを使うかは、ランダムで等確率として、学長のみが知る
ほら言わんこっちゃない
問題が変わってるw

>封筒の金額の分布が重要
じゃあオリジナルの方は問題になってないね
封筒の金額の分布が示されてないんだから

馬鹿だねえ
0468132人目の素数さん
垢版 |
2024/03/09(土) 13:33:24.95ID:fy/7ggA0
はじゃねーよw
0470132人目の素数さん
垢版 |
2024/03/09(土) 14:49:01.22ID:jr6HgB36
箱入り無数目では無限の箱の中に出題者側が
・同じ数を何度入れても構わない
・全部同じだって構わない
なんでしょ?
だからそもそも回答者が答える箱の中の数と他の箱は一切関係無いよ
見る必要も無かったんだよ
回答者が選ぶ“最後に開ける為に残す箱”の中に何が入ってるかは無限の可能性を絞りきれてないよね?
更に言えばそもそもがこの問題の“嘘”レトリックだよね
「無限の箱の最後の箱」って何?
無限の箱には最後の箱なんて永久に存在し無いよね?
0471132人目の素数さん
垢版 |
2024/03/09(土) 15:42:01.32ID:RwepsQi7
>>458
>箱の中身が、iidである確率事象に従って実数が入れられているとすれば
そんな前提は、あなたが勝手に妄想してるだけですが

>一方、箱にカメラが仕込んであれば、箱の中の情報が分かる
>開けてない箱の中身が当たる
>即ち、情報漏洩です
カメラは仕込んでない だからチートではない

しかし当たる
これを「情報漏洩」というなら、そうなんでしょう

ーーーーーーーーーーーーーーーーーーーーーーーーーーーーーーーーーーーー
更に,何らかの事情によりdが知らされていなくても,
あるD>=d についてsD+1, sD+2,sD+3,・・・が知らされたとするならば,
それだけの情報で既に r = r(s)は取り出せ, したがってd= d(s)も決まり,
結局sd (実はsd,sd+1,・・・,sD ごっそり)が決められることに注意しよう.
ーーーーーーーーーーーーーーーーーーーーーーーーーーーーーーーーーーーー

1.任意の尻尾同値類から、選択公理により
それぞれその代表となる無限列を取ることができる
2.任意の無限列sはそれぞれが属する尻尾同値類の代表r(s)と
一致する尻尾を持つからその先頭箇所head(s,r(s))を
列の決定番号d(s)と定義する
3.無限列sは任意の自然数nから先の尻尾tail(s,n)によって
その同値類r(s)を知ることができる
r(s)=r(tail(s,n))
4.自然数nがsの決定番号d(s)よりも大きいならば
条件d(s)<=m<nを満たすmに対して以下が成り立つ
s[m]=r(tail(s,n))[m]
0472132人目の素数さん
垢版 |
2024/03/09(土) 15:51:36.40ID:fy/7ggA0
>>470
>回答者が答える箱の中の数と他の箱は一切関係無いよ
列sの決定番号以降の項は代表列rの対応する項と一致している。
すなわち、s_dとs_(d+1)は「rの対応する項と一致している」という関係性において関係がある。
「箱にどんな実数を入れるか自由」だからといって「箱どうしは無関係」と結論付けるのは浅はか。

>回答者が選ぶ“最後に開ける為に残す箱”の中に何が入ってるかは無限の可能性を絞りきれてないよね?
箱入り無数目記事で述べられてる確率は「ある箱の中身を当てる確率」ではない、「代表列の対応する箱と中身が一致している箱を当てる確率」である。
記事は「後者では当てられる」と言ってるのであって、君の発言「前者では当てられない」はナンセンス。

>無限の箱には最後の箱なんて永久に存在し無いよね?
はい それが何か?
0473132人目の素数さん
垢版 |
2024/03/09(土) 15:51:49.79ID:RwepsQi7
>>461
>さいころの各面に 100, 200, 400, 800, 1600, 3200 と書いて、
>振って出た金額とその目の2倍を封筒に入れます。
>この時に、片方の封筒を開けて
>1.封筒の中身が100円ならば、封筒を換えて200円になる。
>2.封筒の中身が6400円ならば、封筒を換えない。

さいころ振らずに5000円と10000円を封筒に入れる
この時に、片方の封筒を開けて
1.封筒の中身が5000円ならば、封筒を換えて10000円になる
2.封筒の中身が10000円ならば、封筒を換えても5000円になるだけ
3.(1) (2) 以外であれば、換えることで期待値が1.25倍になる。

ん?(1)(2)以外ってある?ないよね?
0474132人目の素数さん
垢版 |
2024/03/09(土) 15:57:19.55ID:RwepsQi7
>>470
>回答者が選ぶ“最後に開ける為に残す箱”の中に何が入ってるかは
>無限の可能性を絞りきれてないよね?

箱入り無数目では s(i)[m]=r(tail(s(i),n))[m] と予想します (m<nとする)
この予想がはずれる箱は、無限個の箱の中のたかだか有限個ですが、何か?
0475132人目の素数さん
垢版 |
2024/03/09(土) 16:04:11.66ID:RwepsQi7
>>474
>箱入り無数目では s(i)[m]=r(tail(s(i),n))[m] と予想します (m<nとする)
>この予想がはずれる箱は、無限個の箱の中のたかだか有限個ですが、何か?

d(s(i))<=mであれば s(i)[m]=r(tail(s(i),n))[m]
つまり、この予想がはずれるには
d(s(i))>mである必要があるが
そのようなmは有限個

mがどのd(s(i))よりも大きければ、当然成立する
100個のうち99個のd(s(i))をとりその最大値を取る
これが100個のd(s(i))の最大値であるならば問題ない

そうでない場合は残ったd(s(i))が他の99個より大きい場合
そのような場合は100中1個しかない
つまり、ランダムに1列選べば成功確率は少なくとも1-1/100
0476132人目の素数さん
垢版 |
2024/03/09(土) 16:06:24.03ID:RwepsQi7
>>475
誤 mがどのd(s(i))よりも大きければ、当然成立する
正 mがどのd(s(i))以上であれば、当然成立する
0477132人目の素数さん
垢版 |
2024/03/09(土) 16:18:19.18ID:RwepsQi7
箱入り無数目の場合
無限個の箱に対して、有限個の箱を除いた箱全ての情報から
「全箱の情報の候補」を得ることができる
その「全箱の情報の候補」は有限個の箱を除いて当たっている

ここで「常識人」は

「んなこというても「開示した情報」>「獲得した情報」やろ」
(注:朝ドラ「まんぷく」の世良勝男の口調でいうてなw)

と思うだろうけど、実は豈図らんや(あにはからんや、と読むw)

「実は、「開示した情報」<「獲得した情報」となることがあるんですよ!」
(注:朝ドラ「まんぷく」の立花萬平の口調でいってね)

まあ、ここで

「そんな、アホなことあるわけないやないの」
(注:朝ドラ「まんぷく」の今井鈴の声でいうてな)

という人もおりましょうが

「お母さん、それがあるんですよ。
 萬平さんは、その方法をみつけたんです。」
(注:朝ドラ「まんぷく」の立花福子の声でいうてな)

これが(選択公理が成立する)数学における真実なんです
0478132人目の素数さん
垢版 |
2024/03/09(土) 16:23:32.37ID:3L5u7doY
マルチレス失礼します

>>473
>ん?(1)(2)以外ってある?ないよね?

(1)(2)以外があるかないかは
前提条件のお金の種類で決まる
だから、入れるお金(金額)の種類が変われば
(1)(2)以外があるかないかも変わりますよ

>>471
>>箱の中身が、iidである確率事象に従って実数が入れられているとすれば
>そんな前提は、あなたが勝手に妄想してるだけですが

「箱入り無数目」の許容範囲です

(参考)時枝記事>>1
https://imgur.com/a/8bqlb08
数学セミナー201511月号「箱入り無数目」
https://rio2016.5ch.net/test/read.cgi/math/1620904362/401-406
純粋・応用数学(含むガロア理論)8 より
1.時枝問題(数学セミナー201511月号の記事)の最初の設定はこうだった。
「箱がたくさん,可算無限個ある.箱それぞれに,私が実数を入れる.
どんな実数を入れるかはまったく自由,例えばn番目の箱にe^nを入れてもよいし,すべての箱にπを入れてもよい.
もちろんでたらめだって構わない.
(引用終り)

>>470
>だからそもそも回答者が答える箱の中の数と他の箱は一切関係無いよ
>見る必要も無かったんだよ
>回答者が選ぶ“最後に開ける為に残す箱”の中に何が入ってるかは無限の可能性を絞りきれてないよね?
>更に言えばそもそもがこの問題の“嘘”レトリックだよね

同意です
『“嘘”レトリック』です
0479132人目の素数さん
垢版 |
2024/03/09(土) 16:27:26.26ID:RwepsQi7
>>478
>>ん?(1)(2)以外ってある?ないよね?
>(1)(2)以外があるかないかは
>前提条件のお金の種類で決まる
>だから、入れるお金(金額)の種類が変われば
>(1)(2)以外があるかないかも変わりますよ

(1)(2)は当然あるよね?
ほな(3)はなくてええんちゃう? (世良勝男)

>>そんな前提は、あなたが勝手に妄想してるだけですが
>「箱入り無数目」の許容範囲です

そんなん、きみが勝手にいうてるだけや (世良勝男)

ああ、いいなあ 世良勝男
https://mantan-web.jp/article/20231118dog00m200019000c.html
0481132人目の素数さん
垢版 |
2024/03/09(土) 16:58:26.39ID:fy/7ggA0
>>480
そんなものが要ると妄想してるからいつまでも理解できないんだよ
0483132人目の素数さん
垢版 |
2024/03/09(土) 17:18:43.39ID:fy/7ggA0
>>482
はい要らないです
記事を読める国語と数学の学力があれば足ります
0485132人目の素数さん
垢版 |
2024/03/09(土) 17:31:00.46ID:RwepsQi7
>>482
>じゃあなんで上の人は情報の不等号を使ってるんだよ

正確には包含関係

つまり選んだ列に関して
「箱の中身が分かった場所」⊂「得た情報」
となるということ

具体的には>>471の通り
開けた箇所の開始位置Dが列の決定番号dより大きければ
d<=m<Dとなるmの箇所の箱の中身がわかる
Dが大きければ大きいほど、わかる場所が大きくなる
0486132人目の素数さん
垢版 |
2024/03/09(土) 17:33:06.96ID:fy/7ggA0
>>484
却下
あなたが必要と思ってやるのはあなたの自由
それに対して論評するのは私の自由
0487132人目の素数さん
垢版 |
2024/03/09(土) 17:39:02.46ID:3L5u7doY
>>479
>ほな(3)はなくてええんちゃう? (世良勝男)

なくても良いが、あっても良いのよw
詳しくは下記
https://researchmap.jp/blogs/blog_entries/view/92228/98e56ed2a2e2f485f4c22d2bcac369c0?frame_id=526781
2つの封筒問題 2014/04/07 関 勝寿
期待値計算はこれでいいのか?
2.(1)の議論とはまた別に、そもそもこの期待値計算はおかしいんじゃないの?という議論がされることもあります。では、こう考えたらどうでしょうか。封筒を開けて1万円得た時点で、残りの封筒には2万円入っている確率と5000円が入っている確率が等しくなっている。そこで、その1万円を払って、コインを投げて、表が出たら2万円をもらい、裏が出たら5000円をもらう、という賭けをするかどうか。こういう条件だったら、1万円を出して期待値12500円の賭けをするから得だ、だからその賭けをしよう、という期待値計算は問題ないように思えます。

>>>471
>>>そんな前提は、あなたが勝手に妄想してるだけですが
>>「箱入り無数目」の許容範囲です
>そんなん、きみが勝手にいうてるだけや (世良勝男)

「箱入り無数目」の許容範囲です(下記)
時枝:どんな実数を入れるかはまったく自由
出題者:箱の中身、iidである確率事象に従って実数が入れられているとする

回答者:それ困る
時枝:どんな実数を入れるかはまったく自由だから、そこは出題者の勝手ですよwww

(参考)時枝記事>>1
https://imgur.com/a/8bqlb08
数学セミナー201511月号「箱入り無数目」
https://rio2016.5ch.net/test/read.cgi/math/1620904362/401-406
純粋・応用数学(含むガロア理論)8 より
1.時枝問題(数学セミナー201511月号の記事)の最初の設定はこうだった。
「箱がたくさん,可算無限個ある.箱それぞれに,私が実数を入れる.
どんな実数を入れるかはまったく自由,例えばn番目の箱にe^nを入れてもよいし,すべての箱にπを入れてもよい.
もちろんでたらめだって構わない.
(引用終り)
0488132人目の素数さん
垢版 |
2024/03/09(土) 17:39:20.19ID:iC63zgm6
>>485
先頭に∀がついてるんだから全部の箱が最初から分かってるじゃん
そこを直してからじゃないと情報の大小は比較できないでしょ
0489132人目の素数さん
垢版 |
2024/03/09(土) 17:41:57.16ID:RwepsQi7
100列から1列選ぶ
選んだ列の決定番号をd
選ばなかった99列の決定番号の最大値をDとする
d<D+1なら、選んだ列の代表の値から
選んだ列のd番目からD番目までの情報がわかってしまう
d>=D+1なら、何も情報はわからない

ここで重要なのは
「100列のどれを選んでもd>=D+1」
となるようなことは決してないということ

「d>=D+1」となるような列はたかだか1列しかない

だから不運な1列以外のどの列を選んでも必ず新情報がゲットできる

萬平「ということなんですよ、世良さん」
世良「ボクは信じへんよ 萬平クン」

関西人は頑固で困るw
0490132人目の素数さん
垢版 |
2024/03/09(土) 17:44:21.85ID:RwepsQi7
>>487
>なくても良いが、あっても良いのよ
世良「なくてもええなら、なしで」

>>488
>先頭に∀がついてるんだから全部の箱が最初から分かってるじゃん
意味がわからん

君、以下の文章読んだ?理解した? これが全てよ
ーーーーーーーーーーーーーーーーーーーーーーーーーーーーーーーーーーーー
更に,何らかの事情によりdが知らされていなくても,
あるD>=d についてsD+1, sD+2,sD+3,・・・が知らされたとするならば,
それだけの情報で既に r = r(s)は取り出せ, したがってd= d(s)も決まり,
結局sd (実はsd,sd+1,・・・,sD ごっそり)が決められることに注意しよう.
ーーーーーーーーーーーーーーーーーーーーーーーーーーーーーーーーーーーー
0491132人目の素数さん
垢版 |
2024/03/09(土) 17:49:54.18ID:3L5u7doY
>>486
>あなたが必要と思ってやるのはあなたの自由
>それに対して論評するのは私の自由

論評するのは自由だが
あまりにも、無知・アホ・間抜け・トンチンカン
そう言われているのでは? ;p)
0492132人目の素数さん
垢版 |
2024/03/09(土) 17:50:33.75ID:iC63zgm6
>>490
何らかのの事情以前に∀が先頭についてるんだから公開情報だろ、公開されてるのを見なかったことにしたつもりが見えてるんだよ
0494132人目の素数さん
垢版 |
2024/03/09(土) 17:53:20.16ID:fy/7ggA0
>>487
>出題者:箱の中身、iidである確率事象に従って実数が入れられているとする
試行の概念があって初めて確率事象になります。
1回の出題において何が試行ですか?
0495132人目の素数さん
垢版 |
2024/03/09(土) 17:55:17.96ID:3L5u7doY
>>490
>>>487
>>なくても良いが、あっても良いのよ
>世良「なくてもええなら、なしで」

こちらが言っていることは
・数学的には、なくても良いバージョンと、あっても良いバージョンと二つ可能だと
・よって、数学的には、両方のバージョンを考察するのが正しいのです
0496132人目の素数さん
垢版 |
2024/03/09(土) 17:58:25.94ID:fy/7ggA0
>>488
>先頭に∀がついてるんだから全部の箱が最初から分かってるじゃん
量化と公開状態がリンクしてるという主張はあなたの独善持論ですよね?
0497132人目の素数さん
垢版 |
2024/03/09(土) 17:59:20.77ID:RwepsQi7
>>492
>∀が先頭についてるんだから公開情報だろ
なぜそう妄想するんですか?

ずっと同じことを尋ねられてますが一度も答えませんね
0498132人目の素数さん
垢版 |
2024/03/09(土) 18:00:02.80ID:fy/7ggA0
>>491
では無知・アホ・間抜け・トンチンカンと思う理由を具体的にどうぞ
0500132人目の素数さん
垢版 |
2024/03/09(土) 18:00:59.23ID:fy/7ggA0
>>492
>∀が先頭についてるんだから公開情報だろ
妄想が激しいようですね
お薬飲み忘れましたか?
0501132人目の素数さん
垢版 |
2024/03/09(土) 18:02:12.93ID:fy/7ggA0
>>493
>見てないって言うなら最初から完全に隠してやってみろよ
任意に選んだひと箱以外は見てもいいんですよ?
ルールを変えちゃ駄目
0502132人目の素数さん
垢版 |
2024/03/09(土) 18:02:57.37ID:RwepsQi7
>>495
>・数学的には、なくても良いバージョンと、あっても良いバージョンと二つ可能だと
>・よって、数学的には、両方のバージョンを考察するのが正しいのです

数学的には、そもそも分布を考えないのが正しい
金額Xと2Xで、Xなら交換したほうが得だし、2Xなら交換しないほうが得
でも自分の金額だけわかっても、Xと2Xのどっちかは分からん、それだけのこと
世良「わからんことをわかったと思うとかアタマおかしいで」
(完)
0503132人目の素数さん
垢版 |
2024/03/09(土) 18:04:50.60ID:fy/7ggA0
>>499
いいえ
あなたの常識は世間の非常識です
0506132人目の素数さん
垢版 |
2024/03/09(土) 18:05:50.60ID:RwepsQi7
>>499
>こんなん常識だろ…
なぜそう妄想するんですか?

ずっと同じことを尋ねられてますが一度も答えませんね

>>500
>妄想が激しいようですね お薬飲み忘れましたか?
ID:iC63zgm6 セレネース飲んでる?
0507132人目の素数さん
垢版 |
2024/03/09(土) 18:07:59.84ID:iC63zgm6
これまた書かないとだめなの?何回目?
めんどくさいんだけど

370 132人目の素数さん sage 2024/02/15(木) 21:51:55.09 ID:Yql9K+Mt
例えばさ、箱の中に正の整数が入ってます。あなたはそれを見ずに何か正の整数を宣言します。あなたの答が箱の中の数以下なら勝利です。必勝法はありますか?
という問題なら、∃x.∀y. x≦y が成立するから必勝ですって誰でも答えられるでしょ
これを、∀y. ∃x. x≦yが成立するから必勝ですって言ったらおかしいでしょ
後者の命題は正の整数の代わりに整数にしても成り立つけど、明らかに整数では必勝法はない。
だから、箱の中を見てないと主張するには∀をなるべく内側に入れた命題を証明しないとだめなんじゃよ
0508132人目の素数さん
垢版 |
2024/03/09(土) 18:08:31.52ID:fy/7ggA0
>>504
だからそれが箱入り無数目記事ですけど?
あなた記事読みました?読まずに妄想語ってませんか?
0509132人目の素数さん
垢版 |
2024/03/09(土) 18:10:28.05ID:fy/7ggA0
>>505
常識だとおっしゃるなら書籍を引用するでも何でも良いので立証して下さい
常識だーと叫んだところで非常識が常識になることはありません
0511132人目の素数さん
垢版 |
2024/03/09(土) 18:13:47.52ID:iC63zgm6
>>509
どうせ引用しても読まねーじゃん
前も確率論の本に載ってるって教えてやったのに読んでねーだろ
0512132人目の素数さん
垢版 |
2024/03/09(土) 18:54:57.24ID:RwepsQi7
>>507
>例えばさ、箱の中に正の整数が入ってます。
>あなたはそれを見ずに何か正の整数を宣言します。
>あなたの答が箱の中の数以下なら勝利です。
>必勝法はありますか?
>という問題なら、
>∃x.∀y. x≦y が成立するから必勝ですって
>誰でも答えられるでしょ

それ見る見ないと関係ないけど

>これを、∀y. ∃x. x≦yが成立するから
>必勝ですって言ったらおかしいでしょ

別におかしくないよ
例えば、ある人がある数を言って、
あなたがその数より大きな数をいったら勝ち
としようか

まあ、あなたが勝てるのは明らかだけどそれはなぜ?
∀x∈N.∃y∈N.x<y
だからだよね?

箱入り無数目も箱の中身全部を隠蔽してないよ
っていうか、1個以外全部公開してる
で、どの列も尻尾が分かれば同値類は分かるから代表もわかる
あとはその代表の対応する項と隠してる箱の値が一致するかどうかだけ

候補として選べる100箱のうち、一致しないのはたかだか1箱だと分かってる
2箱以上あったら矛盾するように選べるってこと この時点で勝負あった
それがわからないってのは、自然数の順序が分かってないってことだから
自然数の定義からやり直したほうがいいね
0513132人目の素数さん
垢版 |
2024/03/09(土) 18:56:21.11ID:RwepsQi7
>>511 そもそも君自身、確率論の本一冊も読めないド素人でしょ
0514132人目の素数さん
垢版 |
2024/03/09(土) 19:00:19.68ID:fy/7ggA0
>>507
あなたは、命題「∀y. ∃x. x≦y」 と 定理「箱の中に正の整数が入ってます。あなたはそれを見ずに何か正の整数を宣言します。あなたの答が箱の中の数以下なら勝利です。必勝法はあります。」の間にギャップがあると主張しています。
では、箱入り無数目記事に記載されている証明のどこにギャップがあるか示して下さい。
0515132人目の素数さん
垢版 |
2024/03/09(土) 19:02:22.39ID:KYqvXqU8
>>512
先手が数を箱にいれる問題にたいして、最初に∀がついてるほうの論理式が証明できたから必勝だって主張したらおかしいでしょって書いたんだけど
0516132人目の素数さん
垢版 |
2024/03/09(土) 19:04:00.58ID:fy/7ggA0
>>510
>先頭に∀が入ってるのになんでそんな嘘つくの?
嘘だと言うなら、>>514を実行して下さいね 実行せずに嘘と断定することはできないはずです
0517132人目の素数さん
垢版 |
2024/03/09(土) 19:04:07.70ID:KYqvXqU8
>>514
箱入り無数目の証明は前半を証明しただけでしょ、それをもって必勝法があるという主張をするのが間違ってるんだよ
0518132人目の素数さん
垢版 |
2024/03/09(土) 19:07:01.73ID:RwepsQi7
>>515
そもそもその言いがかりが狂ってる 
無意味なウソを信じるとウソに殺されて死ぬよ
0519132人目の素数さん
垢版 |
2024/03/09(土) 19:09:55.73ID:fy/7ggA0
>>515
>>507の例で言えることがなぜ箱入り無数目でも言えるのかが示されてませんけど? なぜあなたの妄想でないと言えるのですか?

まあいいから>>514を実行して下さい あなたの妄想を聞いても仕方ありません
0520132人目の素数さん
垢版 |
2024/03/09(土) 19:10:06.65ID:RwepsQi7
最初に∃がつくってことは
「箱の中身をまったく見るとことなく、ある箱を選べば勝てる」
という意味になるけど、別にそんな厳しい条件を満たす必要がない
問題を理解してないから、こういう●違ったことをイキっている
独善的な素人の典型
0523132人目の素数さん
垢版 |
2024/03/09(土) 19:13:33.71ID:fy/7ggA0
>>517
前半とは?
どこにギャップがあるのか具体的に明示して下さい。
0524132人目の素数さん
垢版 |
2024/03/09(土) 19:13:33.80ID:RwepsQi7
箱入り無数目の戦略を見れば明らかだが
実は当てる箱を決めるのに、箱を開けている
100列のうち、選んだ1列以外の99列を開けるところ
そうしないと、選んだ1列の中のどの箱を開けるか決まらないから
つまり、何もあけずに、当てるべき1箱を選んでいるわけではない
0526132人目の素数さん
垢版 |
2024/03/09(土) 19:16:06.74ID:fy/7ggA0
>>522
はい、聞いてません 妄想を聞く耳は持ってません
いいから記事のどこにギャップがあるのか具体的に明示して下さい 妄想はもういいです
0528132人目の素数さん
垢版 |
2024/03/09(土) 19:17:57.20ID:fy/7ggA0
>>521
あなたが威張ってよいのはギャップを見事言い当てた時です
あなたがやったことは妄想を語っただけです そんなの威張れませんよ?
0530132人目の素数さん
垢版 |
2024/03/09(土) 19:18:58.06ID:RwepsQi7
>>521
>だから開けてない箱の∀だけ後ろに移動しろよ
これ馬鹿発言

なぜなら100列全体に対して100箱が決まるから
で、これを回答者が全て知る必要はない
選ばなかった列を選んだ場合にどの箱を選ぶかわかりようがないが
もしその列を選んでいればその箱しか選びようがないから
候補の100箱は決まっている
0532132人目の素数さん
垢版 |
2024/03/09(土) 19:20:47.99ID:fy/7ggA0
>>525
ギャップは具体的に記事のどこかを答えて下さい
妄想で語られても困ります
0534132人目の素数さん
垢版 |
2024/03/09(土) 19:21:55.26ID:fy/7ggA0
>>527
ギャップを示す気あるの?無いの? はっきりしてもらえません?
無いならこれ以上相手しても無駄なので
0535132人目の素数さん
垢版 |
2024/03/09(土) 19:23:02.09ID:KYqvXqU8
>>532
それは先頭に∀がついてる論理式を証明して、見てない箱を見ずに攻略したって主張してる箇所だろ
0536132人目の素数さん
垢版 |
2024/03/09(土) 19:23:20.60ID:RwepsQi7
ーーーーーーーーーーーーーーーーーーーーーーーーー
片端から箱を開けてゆき中の実数を覗いてよいが,
一つの箱は開けずに閉じたまま残さねばならぬとしよう.
どの箱を閉じたまま残すかはあなたが決めうる.
ーーーーーーーーーーーーーーーーーーーーーーーーー

もし、以下のような文章だったら「当たる戦略」はないだろう

どれか一つを閉じるかはあなたが決めうる.
ただし決める前に一切箱を開けてはならない.
決めた後なら片端から箱を開けてゆき中の実数を覗いてよい.
0537132人目の素数さん
垢版 |
2024/03/09(土) 19:24:44.79ID:fy/7ggA0
>>529
はい、逃げたw

妄想語られても不成立の証拠になりませんよ?
ギャップを言い当ててこそ証拠になります あなたはそこから逃げました さよなら
0538132人目の素数さん
垢版 |
2024/03/09(土) 19:26:00.20ID:fy/7ggA0
>>531
己の独善持論に賛同しない者は馬鹿ですか
妄想激しいですね
0539132人目の素数さん
垢版 |
2024/03/09(土) 19:27:24.19ID:fy/7ggA0
>>535
記事を引用して具体的に言わないとダメ
あなたの妄想は聞いてない
0540132人目の素数さん
垢版 |
2024/03/09(土) 19:30:16.49ID:KYqvXqU8
>>539
都合が悪くなると何か引用して示せといつものことだね
前に示した確率論の本を読んでからにしてよ
0541132人目の素数さん
垢版 |
2024/03/09(土) 19:34:13.55ID:fy/7ggA0
>>540
記事にギャップが存在するなら引用して具体的に示せるはずですけど?
妄想で語っても無駄です
0542132人目の素数さん
垢版 |
2024/03/09(土) 19:36:26.16ID:KYqvXqU8
>>541
証明してるのは先頭に∀がついてる論理式であって、それは箱を見ずに答える問題の定式化になってないって何回も言ってるだろ
0543132人目の素数さん
垢版 |
2024/03/09(土) 19:38:51.49ID:fy/7ggA0
自分が持ち出した例ではちゃんとギャップを示してますよね?
 命題「∀y. ∃x. x≦y」 と 定理「箱の中に正の整数が入ってます。あなたはそれを見ずに何か正の整数を宣言します。あなたの答が箱の中の数以下なら勝利です。必勝法はあります。」の間にギャップがある。

なんで箱入り無数目のギャップは示さないのでしょう?
0544132人目の素数さん
垢版 |
2024/03/09(土) 19:40:34.97ID:fy/7ggA0
>>542
あんた日本語読めないの? 記事のどこかって聞いてるんだけど
あんたの独自語で語られてもこちらは理解できません
0545132人目の素数さん
垢版 |
2024/03/09(土) 19:43:53.79ID:fy/7ggA0
>>542
あんたそもそも記事読んでないんでしょ?白状しなさい
だから記事のどこか?って聞かれても何も言えないんでしょ?
0546132人目の素数さん
垢版 |
2024/03/09(土) 19:45:23.91ID:KYqvXqU8
>>544
そもそも記事では触れられてない情報漏洩の仕組みを考えてるのに、なんでそんなことする必要があるんだよ
0547132人目の素数さん
垢版 |
2024/03/09(土) 19:46:19.25ID:fy/7ggA0
>>507の例を持ってきたのが記事読んでない証拠w
白状しなさい
0549132人目の素数さん
垢版 |
2024/03/09(土) 19:47:56.37ID:fy/7ggA0
>>546
「ギャップがあるというのはあんたの妄想以外のなにものでもない」が結論でよいのね?w
0550132人目の素数さん
垢版 |
2024/03/09(土) 19:48:36.76ID:fy/7ggA0
>>548
それが妄想だと言ってるんだけど
あんたも分からん人やねえ
0551132人目の素数さん
垢版 |
2024/03/09(土) 19:49:38.34ID:fy/7ggA0
>>548
チラ見して同じだと妄想しました
となぜ白状しないのか?
0553132人目の素数さん
垢版 |
2024/03/09(土) 19:53:05.35ID:fy/7ggA0
>>552
つまりおまえは箱入り無数目のギャップを見つけられてないってことね?
はい、白状しましたね
0555132人目の素数さん
垢版 |
2024/03/09(土) 20:59:11.20ID:fy/7ggA0
>>554
つまりギャップは無いと?
0556132人目の素数さん
垢版 |
2024/03/09(土) 20:59:36.58ID:RwepsQi7
>>546
>そもそも記事では触れられてない情報漏洩の仕組み
はい、大嘘

合法的な情報漏洩術は、記事にて記載されてると>>471で示してます
あなたが理解できないだけです 
ーーーーーーーーーーーーーーーーーーーーーーーーーーーーーーーーーーーー
更に,何らかの事情によりdが知らされていなくても,
あるD>=d についてsD+1, sD+2,sD+3,・・・が知らされたとするならば,
それだけの情報で既に r = r(s)は取り出せ, したがってd= d(s)も決まり,
結局sd (実はsd,sd+1,・・・,sD ごっそり)が決められることに注意しよう.
ーーーーーーーーーーーーーーーーーーーーーーーーーーーーーーーーーーーー

南無阿弥陀仏
0557132人目の素数さん
垢版 |
2024/03/09(土) 21:03:36.95ID:3L5u7doY
>>494
>>出題者:箱の中身、iidである確率事象に従って実数が入れられているとする
>試行の概念があって初めて確率事象になります。
>1回の出題において何が試行ですか?

1)広義の試行は、下記の「箱入り無数目」の通りです
 「箱がたくさん,可算無限個ある.箱それぞれに,私が実数を入れる.
 どんな実数を入れるかはまったく自由,例えばn番目の箱にe^nを入れてもよいし,すべての箱にπを入れてもよい.
 もちろんでたらめだって構わない.」
2)そして、>>487より「出題者:箱の中身、iidである確率事象に従って実数が入れられているとする」
 これは、下記重川の確率論基礎の射程内ですので、確率事象!
3)時枝「箱入り無数目」記事の試行で、確率事象にならないことは
 ”しっぽの同値類から代表→決定番号→決定番号の大小確率99/100”
 これは、確率事象にならない!(測度の裏付けない。だから、確率空間が書けない!w)

まあ、「箱入り無数目」のギャップは、上記の3)項です

(参考)時枝記事>>1より
https://imgur.com/a/8bqlb08
数学セミナー201511月号「箱入り無数目」
https://rio2016.5ch.net/test/read.cgi/math/1620904362/401-406
純粋・応用数学(含むガロア理論)8 より
1.時枝問題(数学セミナー201511月号の記事)の最初の設定はこうだった。
「箱がたくさん,可算無限個ある.箱それぞれに,私が実数を入れる.
どんな実数を入れるかはまったく自由,例えばn番目の箱にe^nを入れてもよいし,すべての箱にπを入れてもよい.
もちろんでたらめだって構わない.
(引用終り)

(参考)前スレ>>119より再録
https://www.math.kyoto-u.ac.jp/~ichiro/lectures/2013bpr.pdf スレ15>>397より再録
確率論基礎 重川一郎 平成26年8月11日 京大
P47
第4章ランダム・ウォーク
この章では,最も簡単な確率過程としてランダム・ウォークを扱う.
定義1.1 確率変数の族(Xt) TとしてZ+={0,1,2・・}
定義1.2 X1,X2,・・をi.i.d.
(引用終り)
0560132人目の素数さん
垢版 |
2024/03/09(土) 21:05:49.93ID:3L5u7doY
>>555
>>>554
>つまりギャップは無いと?

ほいよ >>557
つまりギャップはあるよ! w
0561132人目の素数さん
垢版 |
2024/03/09(土) 21:40:18.96ID:3L5u7doY
>>560
>ほいよ >>557
>つまりギャップはあるよ! w
 (補足)まず>>465より
具体例で説明しよう
1)ある大学において 学生の奨励として、学長賞で賞金を出すことにした
 1年に10回(夏休み8月とクリスマス休暇の12月を除く)、学年のトップ(1番の人)に
 封筒は二つ使う。そして組合わせが二つ、{5千円、1万円}と{1万円、2万円}と
 どの組合わせを使うかは、ランダムで等確率として、学長のみが知る
 授賞式の事務員は知らない
2)事務員がルールを説明する
「封筒二つで、片方の倍か半分かで。一つの封筒を開けて見て良い。別の封筒に取り替える権利がある。
 もちろん、取り替えないのも可」と
 但し、具体的金額は教えない(説明する事務員も知らない)
3)この場合
 開けた封筒が、1万円ならば
 {5千円、1万円}と{1万円、2万円}が等確率で考えられる
 従って、取り替えると 5千円と2万円が等確率で出現するので
 期待値は、1万2千500円です
4)この確率は、賞金をもらう学生は知らないが 多数例を統計処理すれば
 各金額と期待値は計算できて、期待値1万2千500円は出せる
この例の教訓
1)封筒の金額の分布が重要(よって、「分布は使ってない}という言い訳は通用しない!)
2)開けた封筒は確率ではない。開けていない封筒は確率。両者は峻別されるべき!
(引用終り)

・さて”分布”について
1)簡単に下記「箱入り無数目」で、2列X,Yの並び替えで考える
 X,Yの決定番号をdx,dyとする。dx,dy∈N(自然数)で全体を渡る
2)N(自然数)は減衰しないので、確率分布たりえない!
(”非正則分布”(参考)>>7より)

・開けたものと 開けていないもので 両者は峻別されるべきこと
1)列Xを開けて dx=mを得たとする
2)開けていない dyとmとの比較になる
3)dyは N(自然数)で全体を渡るので、dy<mは有限だが m<dyは無限
4)強いて形式的に書けばP(m<dy)=1 (∵m<dyの領域は無限)
5)つまり、P(m<dy)≠1/2。「箱入り無数目」不成立!

(参考)時枝記事>>1より
https://imgur.com/a/8bqlb08
数学セミナー201511月号「箱入り無数目」

(参考)>>7より
https://ai-trend.jp/basic-study/bayes/improper_prior/
AVILEN Inc. 2020
2020/04/14
非正則事前分布とは?〜完全なる無情報事前分布〜
ライター:古澤嘉啓
0562132人目の素数さん
垢版 |
2024/03/10(日) 06:14:25.21ID:ll3Pb1E3
>>557
1)
>広義の試行は、下記の「箱入り無数目」の通りです
>ーーーーーーーーーーーーーーーーーーーーーーーーーーーーーーーーーー
>箱がたくさん,可算無限個ある.箱それぞれに,私が実数を入れる.
>どんな実数を入れるかはまったく自由,
>例えばn番目の箱にe^nを入れてもよいし,すべての箱にπを入れてもよい.
>もちろんでたらめだって構わない.
>ーーーーーーーーーーーーーーーーーーーーーーーーーーーーーーーーーー

出題は、狭義も広義も、試行ではない

2)
>そして、
>「出題者:箱の中身、iidである確率事象に従って実数が入れられているとする」
>これは、重川の確率論基礎の射程内ですので、確率事象!

重川は「箱入り無数目」について全く言及してないので、
「箱の中身がiidである確率事象」というのは、勝手な妄想

3)
>時枝「箱入り無数目」記事の試行で、確率事象にならない
>”しっぽの同値類から代表→決定番号→決定番号の大小確率99/100”
>これは、確率事象にならない!
>(測度の裏付けない。だから、確率空間が書けない!)

無限列100列に対して、
第1列~第100列のそれぞれの決定番号が単独最大になるもの
の確率測度を求める必要がある、と勝手に決めつけてるが
そんな必要はない

出題によって具体的に100列が決まる
そして、単独最大列が存在する場合、どの列がそうなるかも決まる
あとは、その列を回答者がランダムに選ぶ場合に避けられるか
これこそ確率事象
({1,…,100}の各要素の単集合の測度が1/100とするだけ
 これが確率空間、完全に測度で裏付けられてる)

>まあ、「箱入り無数目」のギャップは、3)です
まあ、君の誤りは2)の以下の文章に尽きる
「出題者:箱の中身、iidである確率事象に従って実数が入れられているとする」
0563132人目の素数さん
垢版 |
2024/03/10(日) 06:17:41.10ID:ll3Pb1E3
>>559
>∀を先頭に置いてる時点でおかしいでしょ

箱入り無数目の問題文に以下のように書かれてるので
君のトンチンカンな言いがかりは却下される
何も見ずして、閉じたままの箱を決めるのではない
ーーーーーーーーーーーーーーーーーーーーーーーーーー
今度はあなたの番である.
片端から箱を開けてゆき中の実数を覗いてよいが,
一つの箱は開けずに閉じたまま残さねばならぬとしよう.
ーーーーーーーーーーーーーーーーーーーーーーーーーー
0564132人目の素数さん
垢版 |
2024/03/10(日) 06:21:26.92ID:ll3Pb1E3
1は、
「箱の中身は未知だから確率変数だ」
という誤った考えにとらわれ
ターンエーは
「他の箱の中身を見てから当てる箱を選ぶのはおかしい(∀が先、はNG)
 当てる箱は他の箱を見ずに最初に決めろ(∃が先、のみOK)」
とか問題文に反する条件を喚き散らす

どっちも妄想性人格障害といわざるを得ない
0565132人目の素数さん
垢版 |
2024/03/10(日) 06:33:17.28ID:ll3Pb1E3
>>561
全体では交換しようがしまいが期待値は
5000✕1/4+10000✕1/2+20000✕1/4
=1250+5000+5000
=11250

5000円の場合交換で10000円 (+5000)
20000円の場合交換で10000円 (ー10000)
10000円の場合交換で
5000✕1/2+20000✕1/2=2500+10000=12500 (+2500)

交換時の増減の期待値を改めて計算すると
5000✕1/4+2500✕1/2+(-10000)✕1/4
=1250+1250-2500
=0

この例の真の教訓
1)最低額では交換で得し、最高額では交換で損する
2)得より損のほうが大きいので、中間では交換で得する形になる
0566132人目の素数さん
垢版 |
2024/03/10(日) 07:24:16.39ID:ll3Pb1E3
>>565 追記
仮に、最低額での損と最高額の得を相殺しようとするなら
金額と確率が反比例する分布とせざるを得ず
その場合には、中間では交換によって全く得しない
0567132人目の素数さん
垢版 |
2024/03/10(日) 08:23:34.92ID:UDtm9Rl+
>>562
あなたのおっしゃる通りだと思いますが、
「箱の中身がiid」を招くのが「出題が試行」でしょうな。「出題が試行」でなければ「箱の中身がiid」が意味を持たないので。
そして箱入り無数目ではひとつの出題が定められた後の回答者の戦略を問われているのだから「出題が試行」は誤り。よって「箱の中身がiid」も誤り。
0568132人目の素数さん
垢版 |
2024/03/10(日) 08:27:00.94ID:UDtm9Rl+
>>558
つまりギャップはあると?
0569132人目の素数さん
垢版 |
2024/03/10(日) 08:27:38.05ID:UDtm9Rl+
>>559
お前の幻想だろ
0570132人目の素数さん
垢版 |
2024/03/10(日) 08:27:56.30ID:RM//RX8S
>>565
>この例の真の教訓
>1)最低額では交換で得し、最高額では交換で損する
>2)得より損のほうが大きいので、中間では交換で得する形になる

最低額、最高額、中間値は教えられていない
さて、毎月1番の ”できすぎ君”がいました

・彼は考えた。最初は、常に封筒を交換しよう
 そうすると、封筒二つ分の情報が得られる
・彼は、1年の前半で情報を集めて
 最低額、最高額、中間値を把握した
・その後は、最低額では交換し、最高額では交換せず
 中間値では交換する という戦略を実行した

この例の真の教訓
「確率分布を把握せよ!」
0571132人目の素数さん
垢版 |
2024/03/10(日) 08:46:57.69ID:RM//RX8S
>>562
>>「出題者:箱の中身、iidである確率事象に従って実数が入れられているとする」
>>これは、の確率論基礎の射程内ですので、確率事象!
>
>重川は「箱入り無数目」について全く言及してないので、
>「箱の中身がiidである確率事象」というのは、勝手な妄想

面白いやつだな
・中学生が連立方程式で、つるかめ算を解いた
 それを見た小学生が、「その連立方程式の教科書には つるかめ算の例題がない」と言った
 (小学生は、連立方程式の なんたるかが 分かっていなかったのです。あんた重川「確率論基礎」分かってないぞw)
・重川の確率論基礎は、可算無限個の箱の中の数を
 確率過程論で扱う方法を提示する
・逆に、「箱入り無数目」(下記)の後半では、時枝氏が重川と同様の可算無限個の
 独立な確率変数の無限族 X1,X2,X3,… に言及しているよw>>3 ;p)

つづく
0572132人目の素数さん
垢版 |
2024/03/10(日) 08:47:14.65ID:RM//RX8S
つづき

(参考)前スレ>>119より再録
https://www.math.kyoto-u.ac.jp/~ichiro/lectures/2013bpr.pdf
確率論基礎 重川一郎 平成26年8月11日 京大
P47
第4章ランダム・ウォーク
この章では,最も簡単な確率過程としてランダム・ウォークを扱う.
定義1.1 確率変数の族(Xt) TとしてZ+={0,1,2・・}
定義1.2 X1,X2,・・をi.i.d.
(引用終り)

(参考)時枝記事>>1より
https://imgur.com/a/8bqlb08
数学セミナー201511月号「箱入り無数目」
https://rio2016.5ch.net/test/read.cgi/math/1620904362/401-406
純粋・応用数学(含むガロア理論)8 より
1.時枝問題(数学セミナー201511月号の記事)の最初の設定はこうだった。
「箱がたくさん,可算無限個ある.箱それぞれに,私が実数を入れる.
どんな実数を入れるかはまったく自由,例えばn番目の箱にe^nを入れてもよいし,すべての箱にπを入れてもよい.
もちろんでたらめだって構わない.
(引用終り)
以上
0573132人目の素数さん
垢版 |
2024/03/10(日) 09:02:52.58ID:UDtm9Rl+
>>571
>・重川の確率論基礎は、可算無限個の箱の中の数を
> 確率過程論で扱う方法を提示する
だから箱入り無数目でもその方法を適用できると妄想してるの?
箱入り無数目では出題は試行でないので適用できません

>・逆に、「箱入り無数目」(下記)の後半では、時枝氏が重川と同様の可算無限個の
> 独立な確率変数の無限族 X1,X2,X3,… に言及しているよw>>3 ;p)
だからなに?
後半で何を言おうと前半に微塵も影響しないけど
0574132人目の素数さん
垢版 |
2024/03/10(日) 09:15:32.77ID:ll3Pb1E3
>>571
>重川の確率論基礎は、
>可算無限個の箱の中の数を確率過程論で扱う方法を
>提示する
>逆に、「箱入り無数目」の後半では、時枝氏が
>可算無限個の独立な確率変数の無限族 X1,X2,X3,… に
>言及している

だから何?

「箱入り無数目」の前半では
箱の中身を確率変数として扱っていない
「箱入り無数目」唯一の確率変数は
回答者が100列からどの1列を選ぶか
それはどの列も確率1/100

問題が決まれば、100列それぞれの決定番号も決まるので
100列それぞれに対して、
他の99列の決定番号の最大値番目を選ぶことは
決定事項(つまり確率変数ではない)
0575132人目の素数さん
垢版 |
2024/03/10(日) 09:22:08.11ID:RM//RX8S
>>574 再録しますw ;p)
ほいよ >>557
つまりギャップはあるよ! w
 (補足)まず>>465より
具体例で説明しよう
1)ある大学において 学生の奨励として、学長賞で賞金を出すことにした
 1年に10回(夏休み8月とクリスマス休暇の12月を除く)、学年のトップ(1番の人)に
 封筒は二つ使う。そして組合わせが二つ、{5千円、1万円}と{1万円、2万円}と
 どの組合わせを使うかは、ランダムで等確率として、学長のみが知る
 授賞式の事務員は知らない
2)事務員がルールを説明する
「封筒二つで、片方の倍か半分かで。一つの封筒を開けて見て良い。別の封筒に取り替える権利がある。
 もちろん、取り替えないのも可」と
 但し、具体的金額は教えない(説明する事務員も知らない)
3)この場合
 開けた封筒が、1万円ならば
 {5千円、1万円}と{1万円、2万円}が等確率で考えられる
 従って、取り替えると 5千円と2万円が等確率で出現するので
 期待値は、1万2千500円です
4)この確率は、賞金をもらう学生は知らないが 多数例を統計処理すれば
 各金額と期待値は計算できて、期待値1万2千500円は出せる
この例の教訓
1)封筒の金額の分布が重要(よって、「分布は使ってない}という言い訳は通用しない!)
2)開けた封筒は確率ではない。開けていない封筒は確率。両者は峻別されるべき!
(引用終り)

・さて”分布”について
1)簡単に下記「箱入り無数目」で、2列X,Yの並び替えで考える
 X,Yの決定番号をdx,dyとする。dx,dy∈N(自然数)で全体を渡る
2)N(自然数)は減衰しないので、確率分布たりえない!
(”非正則分布”(参考)>>7より)

・開けたものと 開けていないもので 両者は峻別されるべきこと
1)列Xを開けて dx=mを得たとする
2)開けていない dyとmとの比較になる
3)dyは N(自然数)で全体を渡るので、dy<mは有限だが m<dyは無限
4)強いて形式的に書けばP(m<dy)=1 (∵m<dyの領域は無限)
5)つまり、P(m<dy)≠1/2。「箱入り無数目」不成立!

(参考)時枝記事>>1より
https://imgur.com/a/8bqlb08
数学セミナー201511月号「箱入り無数目」

(参考)>>7より
https://ai-trend.jp/basic-study/bayes/improper_prior/
AVILEN Inc. 2020
2020/04/14
非正則事前分布とは?〜完全なる無情報事前分布〜
ライター:古澤嘉啓
0576132人目の素数さん
垢版 |
2024/03/10(日) 09:24:37.90ID:ll3Pb1E3
失敗確率1/100を求めるのに
「選んだk番目の列の決定番号が単独最大である確率」
を求める必要はない
『100列のうちから決定番号が単独最大の列を選ぶ確率」
を求めればいい

「」と『』は全く異なる問題
「」は選ぶ列の番号kを定数として、問題100列の全体(R^N)^100を確率変数とするが
『』は問題100列を定数として、選ぶ列の番号の全体{1,…,100}を確率変数とする

なお、当てるべき1箱を決めるにあたって、
事前に他の(無限個の)箱の中身をみてよい
これがもし、決して他の箱を見てはならないならそれは無理ゲー

また、選んだ箱の中身を当てるにあたって
開けられる箱の数がたかだか有限個と限定されても無理ゲー
0577132人目の素数さん
垢版 |
2024/03/10(日) 09:35:36.29ID:ll3Pb1E3
>>575
>さて”分布”について
>簡単に下記「箱入り無数目」で、2列X,Yの並び替えで考える
>X,Yの決定番号をdx,dyとする。dx,dy∈N(自然数)で全体を渡る
>N(自然数)は減衰しないので、確率分布たりえない!

そもそも決定番号の分布が事前に決められると思うのがおかしい
(2つの封筒でも箱入り無数目でも
 封筒やら箱やらの中身について
 「ぼくの考えた無条件事前分布」
 とかいうものがあると妄想するのが誤り)

>開けたものと 開けていないもので 両者は峻別されるべきこと
「ベイジアン教」に洗脳されてますな

>列Xを開けて dx=mを得たとする
>開けていない dyとmとの比較になる

dxはmという定数で、dyは確率変数のままだといいたいらしい
しかし、おかしな事前分布の上では、条件付き確率による計算が失敗する
だから「ベイジアン教」の教えは(箱入り無数目については)間違ってる

おそらく誤りの根源は、おかしな無情報事前分布だろう

P.S
>dyは N(自然数)で全体を渡るので、dy<mは有限だが m<dyは無限
>強いて形式的に書けばP(m<dy)=1 (∵m<dyの領域は無限)

強いて形式的に書いたのが誤り
「dy<mは有限だが m<dyは無限」から「P(m<dy)=1」は導けない
測度の可算加法性を知らないド素人が必ず犯す誤り

>つまり、P(m<dy)≠1/2。「箱入り無数目」不成立!

そもそも、君のやり方ではP(m<dy)が計算できない、というのが正解
したがって、P(dx<dy)≠1/2ともいえない
0578132人目の素数さん
垢版 |
2024/03/10(日) 09:40:33.58ID:RM//RX8S
>>573
>箱入り無数目では出題は試行でないので適用できません

わっはっは ;p)

https://ja.wikipedia.org/wiki/%E8%A9%A6%E8%A1%8C_(%E7%A2%BA%E7%8E%87%E8%AB%96)
試行 (確率論)
確率論において、試行(しこう、英: trial, experiment)とは、起こりうる結果がいくつかあり、そのどれか1つだけが偶然で起こる流れのことである[1]。試行の結果全体の集合は標本空間(全事象)と呼ばれる。

特に起こりうる結果が2つしかない試行はベルヌーイ試行と呼ばれる[2]。

試行の結果のいくつかからなる集合で、起こる割合が決まっていると考えられるものを事象という。事象に対してそれの起こる割合を確率という。

1つの試行を繰り返すことにより、事象の確率を評価することができる(統計的確率)。根元事象に確率変数(一般には確率要素)を割り当てることにより確率質量関数か確率密度関数が決まり、試行は確率分布として定量化できる

https://en.wikipedia.org/wiki/Experiment_(probability_theory)
Experiment (probability theory)

In probability theory, an experiment or trial (see below) is any procedure that can be infinitely repeated and has a well-defined set of possible outcomes, known as the sample space.[1] An experiment is said to be random if it has more than one possible outcome, and deterministic if it has only one. A random experiment that has exactly two (mutually exclusive) possible outcomes is known as a Bernoulli trial.[2]

When an experiment is conducted, one (and only one) outcome results— although this outcome may be included in any number of events, all of which would be said to have occurred on that trial. After conducting many trials of the same experiment and pooling the results, an experimenter can begin to assess the empirical probabilities of the various outcomes and events that can occur in the experiment and apply the methods of statistical analysis.

Experiments and trials
Random experiments are often conducted repeatedly, so that the collective results may be subjected to statistical analysis. A fixed number of repetitions of the same experiment can be thought of as a composed experiment, in which case the individual repetitions are called trials. For example, if one were to toss the same coin one hundred times and record each result, each toss would be considered a trial within the experiment composed of all hundred tosses.[3]

Mathematical description
Main article: Probability space
0579132人目の素数さん
垢版 |
2024/03/10(日) 09:48:12.44ID:ll3Pb1E3
>>578
ーーーーーーーーーーーーーーーーーーーーーーーーーーーーーーーーーーーーーーーーー
https://ja.wikipedia.org/wiki/%E8%A9%A6%E8%A1%8C_(%E7%A2%BA%E7%8E%87%E8%AB%96)
試行 (確率論)
確率論において、試行(しこう、英: trial, experiment)とは、
起こりうる結果がいくつかあり、そのどれか1つだけが偶然で起こる流れのことである。
試行の結果全体の集合は標本空間(全事象)と呼ばれる。
ーーーーーーーーーーーーーーーーーーーーーーーーーーーーーーーーーーーーーーーーー

だろ?
問題は1度出題したらそれで終わり 2つも3つもないんだよ
だから出題は試行ではない
0580132人目の素数さん
垢版 |
2024/03/10(日) 09:48:59.72ID:RM//RX8S
>>573
>>・逆に、「箱入り無数目」(下記)の後半では、時枝氏が重川と同様の可算無限個の
>> 独立な確率変数の無限族 X1,X2,X3,… に言及しているよw>>3 ;p)
>だからなに?
>後半で何を言おうと前半に微塵も影響しないけど

・あらら、時枝さんは後半で、「反省しています」!w ;p)
・時枝さん後半の反省が正しければ、前半は否定されますよ!w ;p)

 >>3より
「もうちょっと面白いのは,独立性に関する反省だと思う.
確率の中心的対象は,独立な確率変数の無限族
X1,X2,X3,…である.
いったい無限を扱うには,
(1)無限を直接扱う,
(2)有限の極限として間接に扱う,
二つの方針が可能である.
確率変数の無限族は,任意の有限部分族が独立のとき,独立,と定義されるから,(2)の扱いだ.
(独立とは限らない状況におけるコルモゴロフの拡張定理なども有限性を介する.)
しかし,素朴に,無限族を直接扱えないのか?
扱えるとすると私たちの戦略は頓挫してしまう.
n番目の箱にXnのランダムな値を入れられて,ある箱の中身を当てようとしたって,
その箱のX と他のX1,X2,X3,・・・がまるまる無限族として独立なら,
当てられっこないではないか−−他の箱から情報は一切もらえないのだから.
勝つ戦略なんかある筈ない,と感じた私たちの直観は,無意識に(1)に根ざしていた,といえる.
ふしぎな戦略は,確率変数の無限族の独立性の微妙さをものがたる, といってもよい.」

(参考)時枝記事>>1
https://imgur.com/a/8bqlb08
数学セミナー201511月号「箱入り無数目」
0581132人目の素数さん
垢版 |
2024/03/10(日) 09:52:42.14ID:RM//RX8S
>>579
(引用開始)
ーーーーーーーーーーーーーーーーーーーーーーーーーーーーーーーーーーーーーーーーー
https://ja.wikipedia.org/wiki/%E8%A9%A6%E8%A1%8C_(%E7%A2%BA%E7%8E%87%E8%AB%96)
試行 (確率論)
確率論において、試行(しこう、英: trial, experiment)とは、
起こりうる結果がいくつかあり、そのどれか1つだけが偶然で起こる流れのことである。
試行の結果全体の集合は標本空間(全事象)と呼ばれる。
ーーーーーーーーーーーーーーーーーーーーーーーーーーーーーーーーーーーーーーーーー
だろ?
問題は1度出題したらそれで終わり 2つも3つもないんだよ
だから出題は試行ではない
(引用終り)

・ぼく、小学生?
・ぼく、サイコロで1度 ”3”とか出たら、もうそれ以上はサイコロの試行はできないの?
・ぼく、もっと勉強しようねwww
www
0582132人目の素数さん
垢版 |
2024/03/10(日) 09:56:58.34ID:ll3Pb1E3
>>580
時枝氏は「箱入り無数目」が成り立たないとはいってない
成り立たたないとわめく連中に対して
何が成り立つための障害となってるのか考察している

非可測もそう、確率変数の無限族の独立もそう

「まるごと独立ならあたりっこない」というのは
箱入り無数目に反対する側が抱く妄想を指している

実際は「任意の有限個に関する独立性」でしかないから
無限個の箱の情報を見た場合には通用しない、という指摘
0583132人目の素数さん
垢版 |
2024/03/10(日) 09:57:46.74ID:UDtm9Rl+
>>575
>1)簡単に下記「箱入り無数目」で、2列X,Yの並び替えで考える
> X,Yの決定番号をdx,dyとする。dx,dy∈N(自然数)で全体を渡る
渡るのは出題前ですよね?
一旦出題を固定したらdx,dyも固定されるので渡りませんよ?理解できないんですか?
そして箱入り無数目で問われてる回答者の戦略は、出題が固定された状況での戦略ですよ?理解できないんですか?

さて固定されたdx,dyがどんな自然数なら的中確率が1/2に満たないか答えて下さい
0584132人目の素数さん
垢版 |
2024/03/10(日) 09:59:46.76ID:ll3Pb1E3
>>581
>ぼく、サイコロで1度 ”3”とか出たら、もうそれ以上はサイコロの試行はできないの?
はい

壺振りは、壺を振ってから、丁半どっちに賭けるか尋ねます 逆ではないですよ

>ぼく、もっと勉強しようね
キミも、日本語、勉強しようね
0585132人目の素数さん
垢版 |
2024/03/10(日) 10:02:02.30ID:ll3Pb1E3
>X,Yの決定番号をdx,dyとする。dx,dy∈N(自然数)で全体を渡る
ただ、どう渡ってるかはわかりませんね

1はそこで「無条件事前分布」とかいうベイジアン教にたよる だから間違う
0586132人目の素数さん
垢版 |
2024/03/10(日) 10:02:58.88ID:UDtm9Rl+
>>578
>わっはっは ;p)
どうした?w
自分の間違いに気づいて発狂した?
0587132人目の素数さん
垢版 |
2024/03/10(日) 10:07:21.97ID:ll3Pb1E3
1にしてもターンエーにしても
自分の思い込み(無情報事前分布とか、∀で束縛すると全情報公開とか)の根拠問われると
何も答えられずにおかしな行動とるよね

やっぱ病気か
0588132人目の素数さん
垢版 |
2024/03/10(日) 10:09:16.40ID:ll3Pb1E3
こっちは
「選択公理を採用したら「箱入り無数目」の戦略は成立するよね」
といってるだけで
「選択公理は絶対の真理」
なんてことはいってない

「あたりっこないから、選択公理はおかしい」
というんなら、ふーん左様ですか、というまで
0589132人目の素数さん
垢版 |
2024/03/10(日) 10:12:39.53ID:UDtm9Rl+
>>579
>問題は1度出題したらそれで終わり 2つも3つもないんだよ
>だから出題は試行ではない
その通りですね
記事にもちゃんと書かれてます

「箱がたくさん,可算無限個ある.箱それぞれに,私が実数を入れる. ・・・そして箱をみな閉じる. 」

箱を閉じた後に箱の中身は変化しない

「今度はあなたの番である.・・・」

箱が閉じられた後に後手のターンとなる
つまり出題は1度だけ、つまり出題は試行足り得ない
0590132人目の素数さん
垢版 |
2024/03/10(日) 10:18:13.28ID:UDtm9Rl+
>>580
>・時枝さん後半の反省が正しければ、前半は否定されますよ!w ;p)
反省が正しい証拠が無い
仮に正しくても前半には影響しない なぜなら前半の論証の中で後半を参照していないから、つまり前半は後半と独立

口から出まかせに適当なこと言うのやめませんか?
0591132人目の素数さん
垢版 |
2024/03/10(日) 10:21:12.55ID:UDtm9Rl+
>>580
肝心な部分が抜けてますよ?

ばかばかしい,当てられる筈があるものか,と感じられるだろう.
何か条件が抜け落ちているのではないか,と疑う読者もあろう.問題を読み直していただきたい.
条件はほんとうに上記のとおり.無限個の実数が与えられ,一個を除いてそれらを見た上で,除いた一個を当てよ,というのだ.
ところがところが--本記事の目的は,確率99%で勝てそうな戦略を供することにある.
0592132人目の素数さん
垢版 |
2024/03/10(日) 10:24:31.88ID:UDtm9Rl+
>>581
>・ぼく、小学生?
>・ぼく、サイコロで1度 ”3”とか出たら、もうそれ以上はサイコロの試行はできないの?
>・ぼく、もっと勉強しようねwww
・ぼく、小学生?
・ぼく、サイコロに相当するのは出題者の出題ではなく回答者の列選択であることが理解できないの?
・ぼく、もっと勉強しようねwww
0593132人目の素数さん
垢版 |
2024/03/10(日) 11:05:13.81ID:UDtm9Rl+
>>581
「さて, 1〜100 のいずれかをランダムに選ぶ. 例えばkが選ばれたとせよ. s^kの決定番号が他の列の決定番号どれよりも大きい確率は1/100に過ぎない.」
ほら、サイコロのランダム性に相当する部分がちゃんと書かれてますよ?
ぼく、日本語が読めないようなのでもっと国語を勉強しようね
0594132人目の素数さん
垢版 |
2024/03/10(日) 12:53:49.81ID:UDtm9Rl+
今日もフルボッコされる不成立派の図
0595132人目の素数さん
垢版 |
2024/03/10(日) 14:42:25.58ID:RM//RX8S
これくらい確率論に無知な二人も珍しいな
いまさら、「箱入り無数目 不成立」は 認めたくないと
必死の強弁
笑える ;p)
0596132人目の素数さん
垢版 |
2024/03/10(日) 14:55:04.14ID:RM//RX8S
あまりにも
アホなことが大杉
メシウマさんも
どれをメシのネタにするか
困るくらいだろうさ ;p)
0597132人目の素数さん
垢版 |
2024/03/10(日) 15:16:06.30ID:UDtm9Rl+
>>595
>これくらい確率論に無知な二人も珍しいな
はい、確率論に無知なので確率を一切使わない100人の数学者バージョンでお願いします
100人の数学者バージョンは成立だと思いますか?不成立だと思いますか?
0598132人目の素数さん
垢版 |
2024/03/10(日) 15:26:28.40ID:ll3Pb1E3
>>595
1、いまさら、「箱入り無数目 成立」は 認めたくないと、必死の強弁
いえばいうほど恥晒す
>>596
1、ア●中に頼りまくり
キミも、喫ったら? ア●ン
0599132人目の素数さん
垢版 |
2024/03/10(日) 16:02:25.00ID:ll3Pb1E3
今日のまとめ

>>564
1は、
「箱の中身は未知だから確率変数だ」
という誤った考えにとらわれ
ターンエーは
「他の箱の中身を見てから当てる箱を選ぶのはおかしい(∀が先、はNG)
 当てる箱は他の箱を見ずに最初に決めろ(∃が先、のみOK)」
とか問題文に反する条件を喚き散らす

>>576
失敗確率1/100を求めるのに
「選んだk番目の列の決定番号が単独最大である確率」
を求める必要はない
『100列のうちから決定番号が単独最大の列を選ぶ確率」
を求めればいい

「」と『』は全く異なる問題
「」は選ぶ列の番号kを定数として、問題100列の全体(R^N)^100を確率変数とするが
『』は問題100列を定数として、選ぶ列の番号の全体{1,…,100}を確率変数とする
0601132人目の素数さん
垢版 |
2024/03/10(日) 18:29:49.50ID:UDtm9Rl+
>>600
先手の任意の手に対して後手の有効手が存在するという形の定理なんだから∀は先頭だろ
先頭だから箱の中身を見ているというおまえの妄想が間違いだと何度言わせるのか
0602132人目の素数さん
垢版 |
2024/03/10(日) 18:40:11.19ID:mo+X3rAk
>>601
これまた書かないとだめなの?何回目?
めんどくさいんだけど

370 132人目の素数さん sage 2024/02/15(木) 21:51:55.09 ID:Yql9K+Mt
例えばさ、箱の中に正の整数が入ってます。あなたはそれを見ずに何か正の整数を宣言します。あなたの答が箱の中の数以下なら勝利です。必勝法はありますか?
という問題なら、∃x.∀y. x≦y が成立するから必勝ですって誰でも答えられるでしょ
これを、∀y. ∃x. x≦yが成立するから必勝ですって言ったらおかしいでしょ
後者の命題は正の整数の代わりに整数にしても成り立つけど、明らかに整数では必勝法はない。
だから、箱の中を見てないと主張するには∀をなるべく内側に入れた命題を証明しないとだめなんじゃよ
0603132人目の素数さん
垢版 |
2024/03/10(日) 18:47:39.98ID:ll3Pb1E3
>>600 ならないよ
>>601 その通り
>>602 何度書いても間違ってるから無意味
先手の出題100列に対して
1列を選択し、99列を見てどの番目の箱か決める
つまり、選べる候補となる100箱は、出題に依存するし
実際99列の決定番号の最大値をとっている
だからいきなり∃n(番目)ではなく
∀x(100列)∃n(それぞれの番目)
0604132人目の素数さん
垢版 |
2024/03/10(日) 18:59:58.90ID:ll3Pb1E3
>>602
例えば、こんな問題を出したとしよう。

100箱の中に正の整数が入っている。
キミは、その中の1箱を選び、他の99箱を見た上で、何か自然数を宣言する。
キミの宣言した自然数が、キミの選んだ箱の中の数以下ならキミの勝利。
勝つ方法はありますか?

さて、実は
∀x_1,…,x_100∈N ∃y_1,…,y_100 (x_i<y_iとならないiはたかだか1個)
が成り立つ

yi=max(x_1,…,x_(i-1),x_(i+1),…,x_100)+1 とすればいい

y_iを決めるのにx_1,…x_(iー1),x_(i+1),…,x_100は使ってるから、見る必要がある
一方x_iは使ってないから、見ていない、と言える

全然おかしくない
だから、当てる箱の中を見てないと主張するのに
∀を内側に入れた命題を証明する必要はない
0605132人目の素数さん
垢版 |
2024/03/10(日) 19:01:35.21ID:ll3Pb1E3
>>602
例えば、こんな問題を出したとしよう。

100箱の中に自然数が入っている。
キミは、その中の1箱を選び、他の99箱を見た上で、何か自然数を宣言する。
キミの宣言した自然数が、キミの選んだ箱の中の数以下ならキミの勝利。
勝つ方法はありますか?

さて、実は
∀x_1,…,x_100∈N ∃y_1,…,y_100∈N (x_i<y_iとならないiはたかだか1個)
が成り立つ

yi=max(x_1,…,x_(i-1),x_(i+1),…,x_100)+1 とすればいい

y_iを決めるのにx_1,…x_(iー1),x_(i+1),…,x_100は使ってるから、見る必要がある
一方x_iは使ってないから、見ていない、と言える

全然おかしくない
だから、当てる箱の中を見てないと主張するのに
∀を内側に入れた命題を証明する必要はない
0607132人目の素数さん
垢版 |
2024/03/10(日) 19:06:04.94ID:ll3Pb1E3
>>606
ほらターンエーは全然問題を理解できないア●中だろ
0608132人目の素数さん
垢版 |
2024/03/10(日) 19:15:44.83ID:mo+X3rAk
こいつ関数が一様連続とは
∀x∀ε∃δなんちゃら
であって、xに依存しないδを取って証明できたことをいうとか言い出すタイプだろ
0609132人目の素数さん
垢版 |
2024/03/10(日) 19:32:10.89ID:ll3Pb1E3
>>608
ターンエー君、一様連続知ってるんだ、エラいね− ボク

一様連続、全然関係ないけどな(ボソッ)
0610132人目の素数さん
垢版 |
2024/03/10(日) 19:38:08.64ID:mo+X3rAk
あー一様連続も通じないやつだったか
もっと基礎解析やって∀と∃を理解してから来てね
0611132人目の素数さん
垢版 |
2024/03/10(日) 19:56:21.88ID:UDtm9Rl+
>>606
それがおまえ
0612132人目の素数さん
垢版 |
2024/03/10(日) 19:57:33.36ID:UDtm9Rl+
>>610
箱入り無数目と一様連続がどう関係するのか述べよ
0613132人目の素数さん
垢版 |
2024/03/10(日) 20:01:20.96ID:mo+X3rAk
>>612
お前が上で書いたことは、∀が外側にあっても一様連続だって主張してるのと同じじゃねーか
0615132人目の素数さん
垢版 |
2024/03/10(日) 20:03:14.93ID:RM//RX8S
スレ主です
私と、メシウマさん、弥勒菩薩さん、某プロ数学者
みんな時枝さんの「箱入り無数目」前半には、納得していない
お二人は、その場の取り繕いに終始している
それって数学かい?(^^
0616132人目の素数さん
垢版 |
2024/03/10(日) 20:03:19.30ID:ll3Pb1E3
ターンエー君は例えば
∀x_1,…,x_99 ∃y_100 ∀x_100 P(x_100<y_100)=99/100
を証明しろ、といってるみたいだけど、
それ、確率空間を取り違えてるトンチンカンな要求なんだよな

確率事象は
(x_1,y_1),…,(x_100,y_100)
の百個なんだよな

そこ、取り違えると、トンチンカン
0617132人目の素数さん
垢版 |
2024/03/10(日) 20:06:39.37ID:ll3Pb1E3
ア●中と弥勒って別人だっけ?
ターンエーはア●中だよな?

某氏はもう懲りたと思う
0619132人目の素数さん
垢版 |
2024/03/10(日) 20:15:48.61ID:ll3Pb1E3
>>618
ベイジアン狂徒はアタマが悪い
0621132人目の素数さん
垢版 |
2024/03/10(日) 20:22:54.71ID:UDtm9Rl+
>>613
妄想は聞き飽きた
0622132人目の素数さん
垢版 |
2024/03/10(日) 20:24:45.05ID:UDtm9Rl+
>>615
>>583を黙殺するおまえがどの口で言うのか
0623132人目の素数さん
垢版 |
2024/03/10(日) 20:30:37.93ID:UDtm9Rl+
>>597も黙殺しとる
その場の取り繕いに終始しているのはおまえ
0624132人目の素数さん
垢版 |
2024/03/10(日) 20:37:55.43ID:mo+X3rAk
Pの定義は何ですか?ってセミナーで聞かれるに決まってるんだから準備して臨めよ
0625132人目の素数さん
垢版 |
2024/03/10(日) 23:21:29.81ID:RM//RX8S
>>617
某氏(プロ数学者)は、賢明だよ
”あぶない数学者”の二の舞を演じる愚はおかさない
時枝を厳しく糾したところで、一文の いや”一目の得にならない”ことを知っているw

かつ、「箱入り無数目」のようなアホ記事に乗せられるアホなプロ数学者がいないことも分かっている
もし居たら、個別にアホ数学者をたしなめればいいだけ
だから、5ch数学板では軽くサバキで打っているんだ

一方、こっちはアマだし
「箱入り無数目」は、もとは欧米でmathoverflow>>4など 2013年あたりで話題になっているが
二つの封筒>>487 や モンティ・ホール問題>>415
ほど解明されていない

それが数学パラドックスとして、「箱入り無数目」を叩く面白さなのです ;p)
0626132人目の素数さん
垢版 |
2024/03/10(日) 23:25:21.49ID:UDtm9Rl+
>>625
>583 >597を黙殺するのは何故ですか?
0627132人目の素数さん
垢版 |
2024/03/11(月) 03:22:12.76ID:8cdYhrps
結局、未定義のPを使ってなんか証明したつもりになって満足してたわけか…
0628132人目の素数さん
垢版 |
2024/03/11(月) 05:38:34.01ID:kEMMPsib
>>627
>未定義のP
いや、定義されてるよ
i=1~100について
P(i番目の列を選ぶ)=1/100
こんな自明なことセミナーで尋ねないよ
薄知じゃないんだから
0630132人目の素数さん
垢版 |
2024/03/11(月) 05:45:31.08ID:kEMMPsib
>>625
>こっちはアマだし
アマじゃなくてド素人

で、>>605の以下の言明は理解したかい?大学数学全滅の落ちこぼれド素人君
ーーーーーーーーーーーーーーーーーーーーーーーーーーーーーーーーーーーーーーーーー
∀x_1,…,x_100∈N ∃y_1,…,y_100∈N (x_i<y_iとならないiはたかだか1個)
が成り立つ
yi=max(x_1,…,x_(i-1),x_(i+1),…,x_100)+1 とすればいい
ーーーーーーーーーーーーーーーーーーーーーーーーーーーーーーーーーーーーーーーーー
0632132人目の素数さん
垢版 |
2024/03/11(月) 06:19:21.84ID:kEMMPsib
>>631
ああ、やっぱりキミ、全然分かってなかったね

P(x_100<y_100)は99/100じゃないよ、0か1かのいずれかだから

求めるべき確率は以下

(i=1~100) P(i番目の列を選ぶ)*P(x_i<y_i)
=99((1/100)*1)+1((1/100)*0)
=99/100

な、全然想定外だったろ? キミ、全然わかってなかったんだよ
0633132人目の素数さん
垢版 |
2024/03/11(月) 06:26:23.22ID:8cdYhrps
>>632
これはなにを計算したんだよ
計算すべきはP(なんか)だろ
これだと計算結果が確率じゃねーじゃん
0635132人目の素数さん
垢版 |
2024/03/11(月) 08:52:30.60ID:d0ha74te
>>634
なにからなにまで教えてもらおうとせず少しは自分の頭で考えたら?
君の頭は何のために付いてんだ?
0636132人目の素数さん
垢版 |
2024/03/11(月) 10:13:01.68ID:oo9XsptK
>>633
P(選んだ列xiについてx_i<y_i)じゃね? だから
Σ (i=1~100) P(i番目の列を選ぶ)*P(x_i<y_i)
0637132人目の素数さん
垢版 |
2024/03/11(月) 10:21:04.91ID:8zwIdoY6
例えばツボの中のサイコロの目が4だったとしよう
しかし、かける方はそんなの知らないから
1から6まで当確率でかけるよな
だから当たる確率が
Σ (i=1〜6) P(iにかける)*P(壺の中の目がi)
₌1/6*0+1/6*0+1/6*0+1/6*1+1/6*0+1/6*0
₌0+0+0+1/6+0+0
₌1/6

この場合、サイコロの目は実は確率変数ではない
かける人がどの目を選択するかが確率変数
0638132人目の素数さん
垢版 |
2024/03/11(月) 11:12:25.15ID:SfpYq/3Q
>>637
スレ主です
ちがうんじゃない?

・一般の確率論は、ツボの中の数を問題としていて
(なにか当てられる方法があれば、それを使うが)
 当てられる方法がない状態を前提として、当たる確率を計算する
・例えば、ある人はナンバー3がラッキーナンバーと思っていて
 常に”3”を唱えるとする
 繰り返すと、普通のサイコロの確率1/6になるだろう
・逆に、二つのサイコロの目の和を当てることにしょう
 そのとき、サイコロの目の和は分布を持つ
 2〜12 で、2や12は頻度が少ない、平均値の7が頻度最大だろう
 この場合、2〜12を等確率で唱えるのは不利で
 常に7を唱えるべきだ
0639132人目の素数さん
垢版 |
2024/03/11(月) 11:26:23.96ID:YoCGShW/
>>638
>ちがうんじゃない?
ちがわないんじゃない?

>例えば、ある人はナンバー3がラッキーナンバーと思っていて常に”3”を唱えるとする
別のある人はナンバー4がラッキーナンバーだよ
それぞれの番号をラッキーナンバーと思ってる人が同じくらいいる勘定

箱入り無数目の確率計算はそういうもの
いいがかりつけるのは勝手だが
そういう君は記事誤読してるってこと
0640132人目の素数さん
垢版 |
2024/03/11(月) 12:12:03.61ID:SfpYq/3Q
>>639
スレ主です
ちがうんじゃない?

 >>637より
「例えばツボの中のサイコロの目が4だったとしよう
 この場合、サイコロの目は実は確率変数ではない
 かける人がどの目を選択するかが確率変数」

面白いけど、面白すぎ
・普通の確率論は、ツボの中のサイコロの目がどういう確率分布になっているかを問題にしている
・もちろん、変則の賭け事で ある人Aさんが 超能力者でサイコロ1つの目の当てゲームをしていて、他の人より当てる確率が高いとする
 そのAさんの”当たり or 外れ”に対する賭けを考えることができる
 そのとき、掛け金は100円で、Aさんが目を当てれば600円貰えるとして、外れは0円
 これで、6回に1回当たれば掛け金は回収できる。1/6以上の確率で当たれば、プラスになる
・しかし、それはあまりに変則の議論だろう
 大学入試で、それ書いたらアウトでしょうね

(参考)
https://study-club.jp/news/matha-prob/
スタクラ情報局確率の計算ができないキミへ(数学A)
確率の計算の基礎
確率の計算ができない。
そう悩む人は多いのではないでしょうか?
数学A の「確率」の分野は、基本さえ理解すれば簡単ですが、それまでが大変。
確率がきっかけで数学が嫌いになってしまう人もいるはずです。
そこでこの記事では、数学A の山場の一つ「確率」の基本をお伝えしていきます。
以下の内容をゆっくり読めば、確率の計算ができるようになるでしょう。
「同様に確からしい」ということ
まずは、確率の重要概念である「同様に確からしい」ということについてお話しします。
略す
0641132人目の素数さん
垢版 |
2024/03/11(月) 12:29:38.85ID:CHDVCn9p
>>638 >一般の確率論は、ツボの中の数を問題としていて・・・
>>640 >普通の確率論は、ツボの中のサイコロの目がどういう確率分布になっているかを問題にしている

君のいう「一般の」とか「普通の」というのは、
君の中だけのことだと気づこう

もちろん
壺振り「あたしゃ4を出し続けるよ」
客  「おれは3にかけ続ける、長嶋ファンだから」
という場合、そりゃ永遠に当たらんわな

そんなこともある
0642132人目の素数さん
垢版 |
2024/03/11(月) 13:48:04.07ID:SfpYq/3Q
>>641
>君のいう「一般の」とか「普通の」というのは、
>君の中だけのことだと気づこう

・”石が流れて木の葉が沈む”か
 倒錯の強弁も、ここまでいけば狂気だろう
・私の「一般の」とか「普通の」とは、下記の九大 原「確率論I」通りです
 下記のサイコロの例”根元事象はE1,E2,E3,...,E6のどれか(ここでEjはサイコロのjの目が出ると言うこと)であり,標本空間は{E1,E2,...,E6}である”
 これを、百回”オンドク”してね
・逆に >>637より「サイコロの目は実は確率変数ではない かける人がどの目を選択するかが確率変数」
 を裏付ける 大学レベルの確率論のテキストがあれば示せ!w

(参考)
https://www2.math.kyushu-u.ac.jp/~hara/lectures/02/pr-grad-all.pdf
確率論I(原)九大
1.1確率論の舞台—事象と標本空間
「確率論」とはその名の通り,「確率」を扱う学問である.世の中には不確かなことが色々ある(例:天気予報).確率論の究極の目的はこの世の中の色々な現象を解き明かす(手助けになる)ことにあると僕は考えるが,初めから世の中の現象を扱うのはなかなか大変である.そのような場合には,まず,目的の現象を数学的に扱いやすい形に変形し(モデル化),そのモデルを考えるのが良い.モデルが理解できた後で,このモデルと現実の現象がどう対応しているのか(またはモデル化に失敗したために対応していないのか)などについて考えるのである.(ただし,数学としての確率論で扱うのは上で述べたプロセスの前半,数学的なモデルの解析が主である.)さて,確率論をやるには,まずその舞台を設定する必要がある.例として1個のサイコロを一回振る実験を考えよう.サイコロが端や角で立たないものとすると,サイコロの6つの面のどれかが出るであろう.そこで以下の定義を行う.
定義1.1.1(標本点と標本空間,有限バージョン)一回の実験の結果として起こりうるものを根元事象または標本点と呼ぶ.標本点の全体からなる集合を標本空間(samplespace)Ωと言う.
このサイコロの例では,根元事象はE1,E2,E3,...,E6のどれか(ここでEjはサイコロのjの目が出ると言うこと)であり,標本空間は{E1,E2,...,E6}である.

標本空間が有限でない場合はいろいろとややこしいことが起こるので,上の定義は根元事象が有限個しかない(つまり,標本空間が有限集合)の場合のものと理解されたい.(無限の場合は後述).
この講義では標本空間が有限の場合(および有限からのアナロジーで理解できる場合)から出発し,段々と深いところに入っていくつもりである.話が分かりにくくなったらいつでも有限の場合のアナロジーに戻って考えるのが良かろう.
さて,我々は根元事象のみに興味があるわけではない.そのために根元事象の集まりとして,「事象」を考える.
定義1.1.2(事象,有限バージョン)標本空間が有限集合の時,数学的には事象とは単に標本空間の部分集合,つまり「根元事象の集まり」のことである.
サイコロの例で言えば,事象の例としては「2と3の目がでること」「偶数の目が出ること」「6の目が出ないこと」などがある.

https://imidas.jp/proverb/detail/X-02-C-02-3-0003.html
imidas
日本語辞典 > 会話で使えることわざ辞典 > 石が流れて木の葉が沈む
0644132人目の素数さん
垢版 |
2024/03/11(月) 14:50:30.88ID:OUf/Z21V
>>642
>私の「一般の」とか「普通の」とは、・・・通りです
>サイコロの例
>”根元事象はE1,E2,E3,...,E6のどれかであり,
>(ここでEjはサイコロのjの目が出ると言うこと)
>標本空間は{E1,E2,...,E6}である”
これを、百回”オンドク”してね

違う問題をいくら読んでも意味がない

>逆に「サイコロの目は実は確率変数ではない かける人がどの目を選択するかが確率変数」
>を裏付ける 大学レベルの確率論のテキストがあれば示せ!

なに怒り狂ってんだこの●違い

そもそも君のテキストも、「箱入り無数目」の箱の中身が確率変数だと云ってるわけではない
「箱入り無数目」の箱の中身が確率変数だといいきる大学の確率論のテキストは存在しない
そもそも「箱入り無数目」の話なんかしてないのだから当然である
0645132人目の素数さん
垢版 |
2024/03/11(月) 14:53:57.58ID:Wi4x1z+m
>>643
>なんで後半で突然掛け算になってるん?

定数のところ(P(x_i<y_i))を、列の選択とは独立の”確率事象”としてあえて書いてるんでしょ
実際は99個が1で、1個が0であるので、”定数”

この程度のことも読み取れないって確率論分かってない証拠だな
0646132人目の素数さん
垢版 |
2024/03/11(月) 15:00:03.10ID:8cdYhrps
>>635
まとめると、君が書いた計算は適当に数式を並べただけで、勝率とは全く関係ないわけね
0647132人目の素数さん
垢版 |
2024/03/11(月) 15:15:55.82ID:CHDVCn9p
>>646
まとめると、ID:8cdYhrps はあの数式が読み取れないほど、確率論が分かってないわけね

そりゃ、数学の全分野、理解できんわ
0649132人目の素数さん
垢版 |
2024/03/11(月) 16:12:12.84ID:d0ha74te
>>642
> 下記のサイコロの例”根元事象はE1,E2,E3,...,E6のどれか(ここでEjはサイコロのjの目が出ると言うこと)であり,標本空間は{E1,E2,...,E6}である”
「Ejはサイコロのjの目が出ると言うこと」って書かれてるじゃんw
サイコロを振るという試行の結果としてサイコロの目が出るんだよ。
丁半博打の場合、既に振られて確定している目に対して客が張るので、試行はサイコロを振ることではなく客が張ること。
仮に客が張ってからサイコロを振るというルールに改変した場合はサイコロを振ることが試行となる。

> これを、百回”オンドク”してね
100万回オンドクしても君の一般・普通が世間の一般・普通になることはありません。
0650132人目の素数さん
垢版 |
2024/03/11(月) 16:16:23.48ID:Wi4x1z+m
>>648 あれ、完全に「答え」だけど、見てもなお意味分からない正真正銘の薄知がいるんだ・・・
0651132人目の素数さん
垢版 |
2024/03/11(月) 16:18:51.14ID:d0ha74te
>>648
あのさあ、自分で考える気が無く人になにからなにまでやらせておいてそれを言う?
0652132人目の素数さん
垢版 |
2024/03/11(月) 17:54:01.31ID:8cdYhrps
>>650
言ってることがわからんが、こういうことを言いたいわけ?

(Ω,P)を1/100 の一様な確率空間としたとき、
∀x∈ℝ^ℕ.∃y∈ようわからん. P(xとyは正解の組み合わである)=99/100
0653132人目の素数さん
垢版 |
2024/03/11(月) 20:00:35.22ID:d0ha74te
>>652
それ聞くってことは記事よんでねーだろおまえ
白状せい
0654132人目の素数さん
垢版 |
2024/03/11(月) 20:02:20.93ID:d0ha74te
自分で記事も読まずになにからなにまで教えてもらおうって魂胆が気に食わねー
そんあ教えて乞食に数学は無理なので諦めろ
0655132人目の素数さん
垢版 |
2024/03/11(月) 20:11:30.15ID:h46pBGwW
>>653
ようわからんのところをどうしたらいいの?
ちゃんとしたのいれたら攻略法があるって定理になると思ってるのこれ?
わしゃなにをどう入れても攻略法があるって定理にはならんと思うぞ
0656132人目の素数さん
垢版 |
2024/03/11(月) 20:46:08.64ID:dHWKTr/8
>>655
>ちゃんとしたのいれたら攻略法があるって定理になると思ってるのこれ?
>わしゃなにをどう入れても攻略法があるって定理にはならんと思うぞ

スレ主です
あなたが正しい

・いま 箱一つ、そこに「箱入り無数目」のように 実数を入れる
 完全任意実数r∈R でも良いが
 区間[0,1]で、r∈[0,1]としよう
・区間[0,1]にルベーグ測度が入る。Ω=[0,1]として
 1点 r∈[0,1]の測度は0
 よって、区間[0,1]の1点的中は、確率0です

(参考)時枝記事>>1
https://imgur.com/a/8bqlb08
数学セミナー201511月号「箱入り無数目」
0657132人目の素数さん
垢版 |
2024/03/11(月) 20:53:43.71ID:d0ha74te
>>656
それ箱入り無数目とは何の関係も無いから。
馬鹿なこと言ってないで>583 >597に答えてもらえません?
0658132人目の素数さん
垢版 |
2024/03/12(火) 00:54:50.68ID:G8Z10h33
結局、このスレの連中はこれが証明できたから攻略法があるって結論なのか?
ようわからんところがようわからんが

(Ω,P)を1/100 の一様な確率空間としたとき、
∀x∈ℝ^ℕ.∃y∈ようわからん. P(xとyは正解の組み合わせである)=99/100
0659132人目の素数さん
垢版 |
2024/03/12(火) 01:13:21.08ID:pMrLmsKB
>>658
トンチンカンなこと言ってないで記事を読みなさい
読んでどこがどう理解できないか言ってみなさい さすれば教えて進ぜよう 
読みもしない者に手取り足取り教えはせぬ
0661132人目の素数さん
垢版 |
2024/03/12(火) 05:42:31.39ID:MuuApGTu
>>652
>言ってることがわからんが、
それは考えてないから

>こういうことを言いたいわけ?
伺おうか

>(Ω,P)を1/100 の一様な確率空間としたとき、
>∀x∈ℝ^ℕ.∃y∈ようわからん. P(xとyは正解の組み合わである)=99/100

なんでx1列?なんでyが何か分からん?

∀x_1,…,x_100∈R^N.∃y_1,…,y_100∈N.P(回答者が選んだ列xと対応するyに対してx[y]=r(x)[y])=99/100

以下を前提する
∀i∈{1,…,100}.P(回答者がiを選ぶ)=1/100

以下は証明できる
∀i∈{1,…,100}. P(x_i[y_i]=r(x_i)[y_i])=1 は少なくとも99個で、P(x_i[y_i]=r(x_i)[y_i])=0 はたかだか1個

回答者の列選択は、出題と独立とする
P(回答者が選んだiのx_i[y_i]=r(x_i)[y_i])
=P(回答者が1を選ぶ)*P(x_1[y_1]=r(x_1)[y_1])
+P(回答者が2を選ぶ)*P(x_2[y_2]=r(x_2)[y_2])

+P(回答者が100を選ぶ)*P(x_100[y_100]=r(x_100)[y_100])
0662132人目の素数さん
垢版 |
2024/03/12(火) 05:48:07.48ID:MuuApGTu
>>655
>ちゃんとしたのいれたら攻略法があるって定理になると思ってるのこれ?
>>652で確率99/100で勝てる定理になっている 「思っている」は要らない

>わしゃなにをどう入れても攻略法があるって定理にはならんと思うぞ
それはキミが箱入り無数目の記事を理解できないだけ
決定番号も理解できず、順序の初歩も理解できないだけ

>>656
>あなた(ID:h46pBGwW)が正しい
大学1年の微分積分も線形代数も理解できずに挫折した人には
「正しい」と裁く資格がない 落ちこぼれは黙れ

>いま 箱一つ、そこに「箱入り無数目」のように 実数を入れる
>完全任意実数r∈R でも良いが
>区間[0,1]で、r∈[0,1]としよう
>区間[0,1]にルベーグ測度が入る。
>Ω=[0,1]として1点 r∈[0,1]の測度は0
>よって、区間[0,1]の1点的中は、確率0です

そもそも箱の中身は確率変数ではない
したがって上記は全く無意味
下手な考え 休むに似たり
縁なき衆生は度し難し
0663132人目の素数さん
垢版 |
2024/03/12(火) 05:50:59.00ID:MuuApGTu
>>658
>ようわからんところがようわからんが
>>661でようわかるようにかいてやったぞ ホレ!

∀x_1,…,x_100∈R^N.∃y_1,…,y_100∈N.P(回答者が選んだ列xと対応するyに対してx[y]=r(x)[y])=99/100

以下を前提する
∀i∈{1,…,100}.P(回答者がiを選ぶ)=1/100

以下は証明できる
∀i∈{1,…,100}. P(x_i[y_i]=r(x_i)[y_i])=1 は少なくとも99個で、P(x_i[y_i]=r(x_i)[y_i])=0 はたかだか1個

回答者の列選択は、出題と独立とする
P(回答者が選んだiのx_i[y_i]=r(x_i)[y_i])
=P(回答者が1を選ぶ)*P(x_1[y_1]=r(x_1)[y_1])
+P(回答者が2を選ぶ)*P(x_2[y_2]=r(x_2)[y_2])

+P(回答者が100を選ぶ)*P(x_100[y_100]=r(x_100)[y_100])
0664132人目の素数さん
垢版 |
2024/03/12(火) 05:53:49.95ID:MuuApGTu
>>661 >>663
「以下は証明できる」の式を修正した

∀x_1,…,x_100∈R^N.∃y_1,…,y_100∈N.P(回答者が選んだ列xと対応するyに対してx[y]=r(x)[y])=99/100

以下を前提する
∀i∈{1,…,100}.P(回答者がiを選ぶ)=1/100

以下は証明できる
∀x_1,…,x_100∈R^N.∃y_1,…,y_100∈N.P(x_i[y_i]=r(x_i)[y_i])=1 は少なくとも99個で、P(x_i[y_i]=r(x_i)[y_i])=0 はたかだか1個

回答者の列選択は、出題と独立とする
P(回答者が選んだiのx_i[y_i]=r(x_i)[y_i])
=P(回答者が1を選ぶ)*P(x_1[y_1]=r(x_1)[y_1])
+P(回答者が2を選ぶ)*P(x_2[y_2]=r(x_2)[y_2])

+P(回答者が100を選ぶ)*P(x_100[y_100]=r(x_100)[y_100])
0665132人目の素数さん
垢版 |
2024/03/12(火) 06:21:57.12ID:MuuApGTu
結局のところ、ID:G8Z10h33は以下の2点がわかってない

1.いかなる無限列でも決定番号から先の尻尾から代表を得れば
決定番号~尻尾の先頭の1つ手前までの情報を”漏洩”させられる
2.自然数n個に対して、他のn−1個よりも大きな自然数はたかだか1個である
これを利用して、列n個に対して、他のn−1個の決定番号の最大値を得れば
それが自列の決定番号よりも大きいような列は、たかだか1個になる

上記2点から100列中99列について情報漏洩が可能と証明できる
逆に言えば情報漏洩の仕組みが理解できないのは上記2点が分かってないから
0666132人目の素数さん
垢版 |
2024/03/12(火) 07:36:34.32ID:PJm9SO46
>>665
>上記2点から100列中99列について情報漏洩が可能と証明できる
>逆に言えば情報漏洩の仕組みが理解できないのは上記2点が分かってないから

スレ主です
笑える

・独立同分布(iid)の箱の中の数
・他の箱を開けて、残る一つの箱を見たところで、無関係
 残る一つの箱の数の情報が得られるはずない
・それを指して、ID:G8Z10h33氏は「情報漏洩」=なんかズルしてる
 と表現したと思うんだよね

それに乗せられて「情報漏洩」だってw
笑えるww
それって数学か?www
0667132人目の素数さん
垢版 |
2024/03/12(火) 07:56:06.28ID:pMrLmsKB
>>666
それ箱入り無数目とは何の関係も無いから。
馬鹿なこと言ってないで>583 >597に答えてもらえません?
0668132人目の素数さん
垢版 |
2024/03/12(火) 09:21:27.22ID:mpcn3wKD
>>666
>笑える
獣の数字を踏んでその言い草が笑止


>・独立同分布(iid)の箱の中の数
>・他の箱を開けて、残る一つの箱を見たところで、無関係
> 残る一つの箱の数の情報が得られるはずない
もし当てる箱が固定なら、ね

しかし、当てる箱がそもそも一定してない
仮に選択した列だけ見たとしよう
そのとき、君のやり方では
1列の決定番号の分布と
99列の決定番号の最大値の分布を
比較することになる

なぜなら、当てる箱の場所は99列の決定番号の最大値だから

2列の場合も1列の決定番号の分布同士
決して、1列の決定番号と固定した一か所の比較ではない
0669132人目の素数さん
垢版 |
2024/03/12(火) 09:44:22.07ID:wpIVsM5P
>>667-668
笑える
”独立同分布(iid)”が分かっていない妄言
アホか
0670132人目の素数さん
垢版 |
2024/03/12(火) 09:47:15.38ID:UzkxeLxM
>>669
君こそ、尻尾同値類とその代表が分かってない

いかなる無限列もその尻尾同値類の代表と
ほとんどすべての項で(つまり有限個の項を除き)一致する
0671132人目の素数さん
垢版 |
2024/03/12(火) 09:52:39.86ID:pMrLmsKB
>>669
>583 >597を黙殺するのはなぜですか?
0672132人目の素数さん
垢版 |
2024/03/12(火) 11:07:09.32ID:pMrLmsKB
>>669
>”独立同分布(iid)”が分かっていない妄言
出題列=0,0,0,0,・・・でした。これはiidですか?
出題列=0,1,2,3,・・・でした。これはiidですか?
出題列=3,1,4,1,5,9,2,6,5,3,・・・でした。これはiidですか?
出題列=π,π,π,π,・・・でした。これはiidですか?
どのような出題列ならiidですか?
どのような出題列なら非iidですか?
0673132人目の素数さん
垢版 |
2024/03/12(火) 12:12:28.01ID:pMrLmsKB
壷の中でサイコロを振って1の目が出ました
100人の客が全員1と賭けたとき何人が正解しますか?
100人の客が全員2と賭けたとき何人が正解しますか?
100人の客が全員ランダムに賭けたとき何人が正解しますか?
100人の客が全員ランダムに賭けたとき確率変数は何ですか?
「見えないものは確率変数でなければならない」は正しいですか?
「箱入り無数目における出題列は確率変数でなければならない」は正しいですか?
0674132人目の素数さん
垢版 |
2024/03/12(火) 13:32:26.33ID:wpIVsM5P
アホが何を書こうが
ダメなものはダメ
じゃないの? ;p)
0675132人目の素数さん
垢版 |
2024/03/12(火) 13:53:35.12ID:pMrLmsKB
>>674
答えに窮して発狂しましたか?
0676132人目の素数さん
垢版 |
2024/03/12(火) 15:10:21.06ID:ipHRQNQh
>>664
だからこれだとだめなんだって
この命題は正しいけど、この命題が成り立つから攻略法があるって主張はできない

>∀x_1,…,x_100∈R^N.∃y_1,…,y_100∈N.P(回答者が選んだ列xと対応するyに対してx[y]=r(x)[y])=99/100

そもそも
∀x_1,…,x_100∈R^N.∃y_1,…,y_100∈N.P(回答者が選んだ列xと対応するyに対してx[y]=r(x)[y])=1
だって証明できるんだから、上ので攻略法があるって主張できるんなら、こっちの命題からは必勝法があるって主張ができる
0677132人目の素数さん
垢版 |
2024/03/12(火) 15:23:56.96ID:pMrLmsKB
>>676
>この命題は正しいけど、この命題が成り立つから攻略法があるって主張はできない
君が主張できないとする理由はx_iが分からないとy_iも分からないからでは?
しかしそれは間違い
x_iのある項以降が分かればx_iが属す同値類が分かり従って代表列が分かり従って決定番号y_iが分かる
箱入り無数目ではひとつの箱を除いて開封してよいルールだから上記は成立する

君の得意の問題における知見を無理やり箱入り無数目に適用しようとしても、問題が違うのだから適用できる保証が無いし、実際できない
未だ理解できないようだね
0678132人目の素数さん
垢版 |
2024/03/12(火) 15:31:09.99ID:pMrLmsKB
君が理解できない最大の理由は記事を読んでないから
だから言ってるよね?
記事を読んで理解しなさいと
記事を読んだ上で理解できない部分があるならここへ書きなさい 教えてあげるから
0679132人目の素数さん
垢版 |
2024/03/12(火) 15:34:26.25ID:pMrLmsKB
ていうか君、同値関係、同値類、選択公理を理解してる?
まずそこだよw
0680132人目の素数さん
垢版 |
2024/03/12(火) 15:36:50.48ID:ipHRQNQh
>>677
違う
∃になってるyたちを具体的な式に展開して命題に書かないとだめって言ってるの

∀x_1,…,x_100∈R^N.P(回答者が選んだ列iに対してxi[f(x1,...,x100,i)]=r(xi)[f(x1,...,x100,i)])=99/100

みたいになってりゃ別に文句言わねーよ
0681132人目の素数さん
垢版 |
2024/03/12(火) 16:09:03.30ID:pMrLmsKB
>>680
>yたちを具体的な式に展開して命題に書かないとだめ

選択公理を仮定すれば
任意の類 ∀[s]∈R^N/〜 に対して代表列 r=f([s])∈[s] を与える選択関数 f:R^N/〜 → R^N の存在が保証される。
関数 g:R^N → R^N/〜 を g(s)=[s] で定義すれば、合成関数 f・g:R^N → R^N は、任意の実数列 ∀s∈R^N に対しその代表列 r=f・g(s) を与える。
ある実数列sの第D+1項から先すべてが分かっているなら、D+1より前の項を0で埋めた実数列s'はs〜s'を満たすから、f・g(s')=f・g(s)=r はsの代表rを与える。

y_iを次で定義する:∀n≧y_i ⇒ x_i[n]=f・g(x_i')[n]
ここでx_i'はx_iのある項より前を0で置換した実数列

以上で、x_i→x_i'→f・g(x_i')→y_i の対応関係が定義されるので、関数d:R^N→N が定義できて、y_i=d(x_i)=d(x_i') と書ける。

∀x_1,…,x_100∈R^N.∃d(x_1)=d(x_1'),…,d(x_100)=d(x_100')∈N.P(回答者が選んだ列xと対応するd(x)に対してx[d(x)]=r(x)[d(x)])=99/100

はい、yたちを具体的な式に展開して命題に書きました。
0682132人目の素数さん
垢版 |
2024/03/12(火) 16:18:07.54ID:wpIVsM5P
>>680
>∃になってるyたちを具体的な式に展開して命題に書かないとだめって言ってるの
>∀x_1,…,x_100∈R^N.P(回答者が選んだ列iに対してxi[f(x1,...,x100,i)]=r(xi)[f(x1,...,x100,i)])=99/100

ありがとうございます
スレ主です
当てられないに賛成なのです
お気に召すかどうかは不明だが、ご一読ください

・例えば、「箱入り無数目」出題(下記)で
 列a=(a1,a2,・・・)なる実数の無限列を作った(a1,a2,・・・たちはすべて箱の中)
・その隣に、回答者が
 列b=(b1,b2,・・・)なる無限列を作った
(当然列bは、列aとは何の関係もない)
・回答者は、列b=(b1,b2,・・・)を見て、決定番号dbを得る(決定番号は下記「箱入り無数目」に従う)
 回答者は、列aでdb+1番以降の箱を開けて、列aの代表raと決定番号daを得る
(ra=(ra1,ra2,・・・)とする)
 回答者は、代表raのdb番目 すなわちradbが、出題列のdb番目adbと等しい すなわち ”adb=radbだ!”と叫ぶ
 2列なので、da<dbの確率は1/2なので、確率1/2の的中が得られる

さて、これでおかしなところは下記です
・列bは、回答者が出題と無関係に作った列なのに これはどうしたことか?
・その列を使って、確率1/2の的中とはこれいかに?
・同様に、99列作れば 確率99/100の的中ですし
・同様に、確率1-εの的中もあり

やっぱり、デタラメさんでしょ 「箱入り無数目」
なので、”当てられない”!ですよね

(参考)時枝記事>>1
https://imgur.com/a/8bqlb08
数学セミナー201511月号「箱入り無数目」
0683132人目の素数さん
垢版 |
2024/03/12(火) 16:18:14.56ID:pMrLmsKB
まあこんな小難しい書き方しなくても
記事をちゃんと理解していれば、任意の実数列sに対してその決定番号を与える関数d(s)が存在することは理解できるはず
君が決定番号を式で書かないとダメと難癖つけたということは君は記事を理解していない証拠
0684132人目の素数さん
垢版 |
2024/03/12(火) 16:36:43.29ID:pMrLmsKB
>>682
>2列なので、da<dbの確率は1/2
はい、大間違い
正しくは 確率1でda≦db または 確率1でda≧db

一方
da,dbのいずれかをランダム選択した方をx、他方をyと書くと、x≦yの確率は1/2
は正しい。

君全然分かってないね 何度も教えたはずなのに

>確率1/2の的中が得られる
これも大間違い
なぜなら、的中するためには、2列のいずれかのランダム選択で列aが選択され、且つ、da≦db である必要があるが、
da≦dbの確率は上記の通り不明だから結局的中確率も不明。

君ズタボロなんだけど 頭悪いにも限度ってものがあるよ
0685132人目の素数さん
垢版 |
2024/03/12(火) 16:51:35.36ID:pMrLmsKB
>da,dbのいずれかをランダム選択した方をx、他方をyと書くと、x≦yの確率は1/2
正確には
da,dbのいずれかをランダム選択した方をx、他方をyと書くと、x≦yの確率は1/2以上
(da=dbの場合確率1でx≦yだから)
0688132人目の素数さん
垢版 |
2024/03/12(火) 17:26:43.30ID:pMrLmsKB
>>687
どう違うと?
0689132人目の素数さん
垢版 |
2024/03/12(火) 17:31:32.60ID:pMrLmsKB
>>687
関数の定義から∀x∈(fの定義域)に対してf(x)は必ず存在するけどその時
∀x.p(x,f(x))

∀x.∃f(x).p(x,f(x))
とでどう違うと?
0691132人目の素数さん
垢版 |
2024/03/12(火) 17:44:07.57ID:upjnOnB4
>>689
∀x∈ℕ.∃y∈ℕ.x<y

∀x∈ℕ.x<x+1
だと後者の方が強い主張をしてるでしょ
箱入り無数目の定式化では関数の形も主張に必要なんだから後者の形で命題を書かないと
0692132人目の素数さん
垢版 |
2024/03/12(火) 18:14:42.16ID:pMrLmsKB
>>691
そんなことは聞いてない
∀x∈ℕ.x<x+1

∀x∈ℕ.∃(x+1).x<x+1
の違いを聞いている
おまえは∃の後ろに変数以外を書くなと言ったが、x+1は変数ではないと?じゃ何?
0693132人目の素数さん
垢版 |
2024/03/12(火) 19:37:50.92ID:upjnOnB4
>>692
∃の後ろに変数じゃないものを書いてるのは君だろ、∃(x+1)ってなんだよ
ふざけて書いてるだろ
0694132人目の素数さん
垢版 |
2024/03/12(火) 19:38:47.26ID:MuuApGTu
>>676
>>∀x_1,…,x_100∈R^N.∃y_1,…,y_100∈N.P(回答者が選んだ列xと対応するyに対してx[y]=r(x)[y])=99/100
>だからこれだとだめなんだって
>この命題は正しいけど、
>この命題が成り立つから攻略法があるって主張はできない
>∀x_1,…,x_100∈R^N.∃y_1,…,y_100∈N.P(回答者が選んだ列xと対応するyに対してx[y]=r(x)[y])=1
>だって証明できるんだから、
>上ので攻略法があるって主張できるんなら、
>こっちの命題からは必勝法があるって主張ができる

なるほど

>>680
>∃になってるyたちを具体的な式に展開して命題に書かないとだめって言ってるの
>∀x_1,…,x_100∈R^N.P(回答者が選んだ列iに対してxi[f(x1,...,x100,i)]=r(xi)[f(x1,...,x100,i)])=99/100
>みたいになってりゃ別に文句言わねーよ

なんだ、それでいいんなら書けるよ

∀x_1,…,x_100∈R^N.
P(回答者が選んだ列xiに対して
 x_i[max(d(x_1),…,d(x_(i-1)),d(x_(i+i)),…,d(x_100))]
=r(x_i)[max(d(x_1),…,d(x_(i-1)),d(x_(i+i)),…,d(x_100))])=99/100

なぜなら
∀x_1,…,x_100∈R^N.(d(x_i)<=max(d(x_1),…,d(x_(i-1)),d(x_(i+i)),…,d(x_100))でないx_iはたかだか1つ)
だから
0696132人目の素数さん
垢版 |
2024/03/12(火) 19:54:36.74ID:MuuApGTu
>>682
>・例えば、「箱入り無数目」出題で
>列a=(a1,a2,・・・)なる実数の無限列を作った
>(a1,a2,・・・たちはすべて箱の中)
うむ

>その隣に、回答者が
>列b=(b1,b2,・・・)なる無限列を作った
>(当然列bは、列aとは何の関係もない)
ダウト1!
出題者が列を二つ作る
回答者は二つの列から一つ選ぶだけ
なんでそれがわからない?
🐎🦌なのか?🌲違いなのか?

ということで、出題者がa,b二列をつくり
回答者はa,bの中からaを選んだ、と言い換える
1もアタマ切り替えろ、🐎🦌

>回答者は、列b=(b1,b2,・・・)を見て、決定番号dbを得る
>(決定番号は下記「箱入り無数目」に従う)
うむ

>回答者は、列aでdb+1番以降の箱を開けて、列aの代表raと決定番号daを得る
>(ra=(ra1,ra2,・・・)とする)
ダウト2!
回答者は代表raは得られるが、
この段階では決定番号daは得られない
(代表だけ分かればいいので決定番号は知る必要もないが)

>回答者は、代表raのdb番目 すなわちradbが、出題列のdb番目adbと等しい すなわち ”adb=radbだ!”と叫ぶ
うむ

>2列なので、da<dbの確率は1/2なので、確率1/2の的中が得られる
然り

> さて、これでおかしなところは下記です
> ・列bは、回答者が出題と無関係に作った列なのに これはどうしたことか?
はいダメこれダメ全然ダメ
出題者が2列つくる 回答者が別の列をつくるのではない
なんでそんなことがわからない? 🌲違いなのか?

>その列を使って、確率1/2の的中とはこれいかに?
出題者が2列つくり、回答者がどちらか選ぶ 
はずれはどちらか一方 だから確率1/2

>同様に、99列作れば 確率99/100の的中ですし
>同様に、確率1-εの的中もあり
何度でも繰り返すが、回答者が99列作るのではない
出題者が100列作って、回答者が1列選ぶ
はずれは1列のみ、だから当たる確率は1-1/100=99/100

出題者が何列作っても、はずれは1列しかない
だから回答者が1列選んで当たる確率は1-1/n
1/nはいくらでも0に近づけられる
0697132人目の素数さん
垢版 |
2024/03/12(火) 19:55:49.07ID:MuuApGTu
>>695
>そうそうそんな風に書けば
おまえが書けよ この中卒ド素人
0699132人目の素数さん
垢版 |
2024/03/12(火) 20:18:08.85ID:MuuApGTu
ところで、100列の決定番号のうち最大の列は唯一、として
モンティ・ホールもどきをやってみよう
つまり、回答者が1列選んだ段階で司会者が残り99列のうち
決定番号が最大でない98列を箱も開けずに片付ける

さて、回答者は残り1列と交換したほうが得か損か(ニヤニヤ)
0700132人目の素数さん
垢版 |
2024/03/12(火) 20:28:34.77ID:PJm9SO46
>>695
スレ主です
教育的ご指導
ご苦労さまです

ようやくスタート地点ですか?
論理式で書いて終わりならば
確率論不要です

例えば、リーマン予想を論理式で書いたとて
それは、リーマン予想の証明ではありませんよね
0701132人目の素数さん
垢版 |
2024/03/12(火) 20:32:55.96ID:MuuApGTu
>>700
>ようやくスタート地点ですか?
1はスタートラインに立ててないけどね

>論理式で書いて終わりならば確率論不要です
確率論学んでも、違う問題解いちゃ無意味

1は「スマリヤンの錯覚」に陥ってますな

http://the-apon.com/coffeedonuts/illusions-have-same-root.html
0702132人目の素数さん
垢版 |
2024/03/12(火) 20:33:10.80ID:pMrLmsKB
>>700
馬鹿なこと言ってないで>583 >597に答えてもらえませんか?
なんで黙殺しようとするんですか?
0703132人目の素数さん
垢版 |
2024/03/12(火) 20:36:47.14ID:MuuApGTu
1は「回答者が100列から1列選ぶ」という問題文が理解できず
「出題者が99列作ってシミュレーションする」と誤読する
独善的な🌲違いですからね
0704132人目の素数さん
垢版 |
2024/03/12(火) 20:39:35.94ID:MuuApGTu
「出題者が99列作ってシミュレーション」という誤読の背景には
「出題列は確率変数でなければならない!」という独善的な思いこみがある
完全に狂っている 正常でない 異常そのもの
0705132人目の素数さん
垢版 |
2024/03/12(火) 20:40:33.37ID:MuuApGTu
>>703-704
誤 「出題者が99列作ってシミュレーションする」
正 「回答者が99列作ってシミュレーションする」
0706132人目の素数さん
垢版 |
2024/03/12(火) 21:12:44.94ID:PJm9SO46
>>682 補足

さて
1)前記の決定番号の大小比較について
 試験の点数の場合と比較してみよう
 多数の答案から 2枚選び その点数をTa,Tbとして比較する
 試験の成績は、正規分布で平均点50点、標準偏差10点とする
 試験結果の最低0点、最高100点
2)2枚の答案は裏向けで、点数は不明とする
 Ta>Tbの確率1/2
 逆に Ta<Tbの確率1/2 (同じ値の場合は頻度が小さいとして無視するとする)

つづく
0707132人目の素数さん
垢版 |
2024/03/12(火) 21:19:27.55ID:PJm9SO46
つづき

3)いま、試験の点数の場合には 点数分布で全体の位置が分かる(下記)

4)ところが、決定番号は >>575に書いたが 上限がなく発散しているので 非正則分布を成す(下記)
 このような場合、ある列の決定番号dx=mを得て 開けていない dyとmとの比較をすると
 dyは N(自然数)で全体を渡るので、dy<mは有限だが m<dyは無限
 強いて形式的に書けばP(m<dy)=1 (∵m<dyの領域は無限)
 つまり、P(m<dy)≠1/2。「箱入り無数目」不成立!
5)さらに、全体が発散しているので、P(dx=dy)=1/2 が疑問になる
 つまり、全体が∞に発散しているとき、∞/∞ の不定形になり
 ”P(dx=dy)=1/2”は 単純には言えない!

つづく
0708132人目の素数さん
垢版 |
2024/03/12(火) 21:20:49.40ID:PJm9SO46
つづき

(参考)
https://ja.wikipedia.org/wiki/%E5%81%8F%E5%B7%AE%E5%80%A4
偏差値

(参考)>>7より
https://ai-trend.jp/basic-study/bayes/improper_prior/
AVILEN Inc. 2020
2020/04/14
非正則事前分布とは?〜完全なる無情報事前分布〜
ライター:古澤嘉啓

https://en.wikipedia.org/wiki/Indeterminate_form
Indeterminate form(不定形)
例 ∞/∞

(参考)時枝記事>>1より
https://imgur.com/a/8bqlb08
数学セミナー201511月号「箱入り無数目」
(引用終り)
以上
0709132人目の素数さん
垢版 |
2024/03/12(火) 22:13:41.38ID:pMrLmsKB
>>693
答えになってない
なぜxは変数でx+1は変数でないのか、変数でないなら何なのか?
0710132人目の素数さん
垢版 |
2024/03/12(火) 22:15:56.18ID:pMrLmsKB
>>706
馬鹿なこと言ってないで>583 >597に答えてもらえませんか?
なぜ黙殺しようとするのですか?
0711132人目の素数さん
垢版 |
2024/03/12(火) 23:19:43.91ID:G8Z10h33
>>709
お前がどうしてもx+1を変数だと言うなら

>そんなことは聞いてない
>∀x∈ℕ.x<x+1
>と
>∀x∈ℕ.∃(x+1).x<x+1
>の違いを聞いている
>おまえは∃の後ろに変数以外を書くなと言ったが、x+1は変数ではないと?じゃ何?

これの後者はα変換したら
∀x∈ℕ.∃y.x<y
と同じだろ
∀x∈ℕ.x<x+1
とは明らかに違うだろ

あと、お前はd/d(x+1)みたいに(x+1)で微分とか普段からしてんのかよ
0712132人目の素数さん
垢版 |
2024/03/12(火) 23:22:14.52ID:G8Z10h33
>∀x∈ℕ.∃(x+1).x<x+1
ていうか、こいつ相手にする必要ある?
ふざけてやってるとしか思えない
0713132人目の素数さん
垢版 |
2024/03/12(火) 23:46:29.35ID:pMrLmsKB
>>711
答えになってない

おまえはxが変数でなぜx+1が変数でないのか答えてない

おまえはx+1が何か答えてない

おまえは
∀x∈ℕ.x<x+1

∀x∈ℕ.∃(x+1).x<x+1
が明らかに違うとしか言っておらずどう違うか答えてない

答えないくせになぜかまったく関係無い微分の話を持ち出している

ていうか、こいつ相手にする必要ある?
ふざけてやってるとしか思えない
0715132人目の素数さん
垢版 |
2024/03/13(水) 00:04:11.91ID:5iS9phMp
>>714
また逃げたw
おまえ答えられなくなるといつも逃げるね
0717132人目の素数さん
垢版 |
2024/03/13(水) 00:25:17.61ID:5iS9phMp
>>716
答えられないってことは君の独善持論ってことだよね?
そんなの聞いてもしかたないので無理に出てこなくていいよ
0719132人目の素数さん
垢版 |
2024/03/13(水) 00:37:03.59ID:5iS9phMp
>>718
なぜ変数でないのか
変数じゃなきゃ何なのか
答えられないってことは君の独善持論ってことじゃん
0721132人目の素数さん
垢版 |
2024/03/13(水) 00:58:49.14ID:5iS9phMp
ちなみに
https://web.sfc.keio.ac.jp/~hagino/logic16/07.pdf
のP4には

• 「もの」の集まり
 • 整数
 • 人間
• 「もの」の集まりを動く変数
 • 対象変数(object variable)
 • 𝑥, 𝑦, 𝑧, . . .

と書かれてる

xが「もの」の集まりである自然数を動く変数であるなら
xの後者であるx+1もやはり自然数を動くので変数の定義を満たす

頑なに変数でないと言い張る人もいるようだけどどうやら独善持論のようですね
0723132人目の素数さん
垢版 |
2024/03/13(水) 01:21:16.03ID:5iS9phMp
ものの集まりとはつまり集合のことだし
ものの集まりを動く変数とはつまり集合の不定元のことだね

∀x∈N.(xは不定) ⇒ x+1∈N ∧ (x+1は不定)
であるから変数の定義に従い
xはNを動く変数 ⇒ x+1はNを動く変数
が成立

>>720
君の中では非変数なんでしょ
それで首尾一貫してれば好きにすればいいじゃん
0725132人目の素数さん
垢版 |
2024/03/13(水) 01:34:59.38ID:5iS9phMp
>>724
あれ?認めちゃったんだw

じゃあ
∀x∈ℕ.∃(x+1).x<x+1
はOKってことね?∃の後ろは変数なんでしょ?

すると
∀x∈ℕ.x<x+1

∀x∈ℕ.∃(x+1).x<x+1
の違いは何だと言ってるの?
0727132人目の素数さん
垢版 |
2024/03/13(水) 01:43:02.61ID:5iS9phMp
>>726
変とは?
また独善持論ですか?
0728132人目の素数さん
垢版 |
2024/03/13(水) 01:51:52.05ID:ascKCvNK
>>727
xが変数だからx+1も変数とかいいだす人間に記号論理学ができるわけないだろ
家庭教師でも雇って教えてもらえ
0729132人目の素数さん
垢版 |
2024/03/13(水) 02:07:50.38ID:5iS9phMp
>>728
あれ?認めたんじゃなかったの?w
じゃあ>>723のどこに欠陥があるのか具体的にどうぞ
0730132人目の素数さん
垢版 |
2024/03/13(水) 02:12:43.16ID:ascKCvNK
>>729
君が問題ないと思ってるならそれでいいじゃん
全く住んでる星が違うんだから好きにやっていいよ
0731132人目の素数さん
垢版 |
2024/03/13(水) 02:41:05.48ID:5iS9phMp
>>730
はい、また逃げたw
0733132人目の素数さん
垢版 |
2024/03/13(水) 05:45:56.61ID:in9dXeLi
>>707
> dyは N(自然数)で全体を渡るので、dy<mは有限だが m<dyは無限
> 強いて形式的に書けばP(m<dy)=1 (∵m<dyの領域は無限)

はい、誤り
P(m<dy)=1 とはいえません

>つまり、P(m<dy)≠1/2。「箱入り無数目」不成立!

dy∈[1,n]の場合も 一般にP(m<dy)は1/2ではない
問題はdx,dy∈[1,∞)に対して、P(dx<dy)=1/2か、ということ
そして、キミのいう非正則分布では、それは導けない
(注:そうならない、という意味ではない)

1は「スマリヤンの錯覚」に陥ってますな

http://the-apon.com/coffeedonuts/illusions-have-same-root.html
0734132人目の素数さん
垢版 |
2024/03/13(水) 05:52:37.60ID:in9dXeLi
>>733
一般に d_1 , … , d_n ∈ [1,∞) に対して
P( max( d_1 , … , d_(n-1) ) < d_n )=1/n か?

d_1 , … , d_n ∈ [1,m) で、みな一様分布かつ相互に独立ならそうなるが
0735132人目の素数さん
垢版 |
2024/03/13(水) 06:48:56.74ID:in9dXeLi
>∀x∈ℕ.∃(x+1).x<x+1
>はOKってことね?

わざわざ∃(x+1).とつける必要はないけどね
0736132人目の素数さん
垢版 |
2024/03/13(水) 08:19:23.89ID:5iS9phMp
>>735
はい、ペアノの公理を前提とするなら ∀x∈N ⇒ x+1:=s(x)∈N なので、∃(x+1).と付ける必要無しは同意です。
但し付けても間違いではなく、付けない式 ∀x∈ℕ.x<x+1 と比較して意味が変わる訳でもないと思ってますが如何でしょう。
0737132人目の素数さん
垢版 |
2024/03/13(水) 12:23:46.26ID:NwNjK2/r
>>736
>∃(x+1).と付ける必要無しは同意です。
>但し付けても間違いではなく、付けない式 ∀x∈ℕ.x<x+1 と比較して意味が変わる訳でもないと思ってますが如何でしょう。

・論理式なんだから
 不要ならつけないのが本当と思うよ
・例えば、数式で x^2+1 と書くとき
 x^2 -x+x +1 と書いて、意味は x^2+1 というがごとし
 ”-x+x”の部分は、簡約できるなら書かないのが普通だろう?(中学数学ならバツではないが、減点される。大人ならアホかと言われる)
0738132人目の素数さん
垢版 |
2024/03/13(水) 14:12:36.74ID:NwNjK2/r
>>733
>1は「スマリヤンの錯覚」に陥ってますな

それ面白いね
「スマリヤンの錯覚」は、下記の”この二つを区別できない心理を私は スマリヤンの錯覚 と呼んでいます”
で、下記著者の造語ですね

http://the-apon.com/coffeedonuts/smulyan-two-envelope.html
モンティ・ホール問題好きのホームページ2015/01/11
スマリヤンの二つの文のパラドックス
スマリヤンのパズル本に出てくる二つの封筒問題を題材とした二つの文 (あるいは二つの命題) とその証明が新たなパラドックスを醸し出しています。

スマリヤンの二つの文
Smullyan, Raymond (1992). の翻訳本や Smullyan, R.: 1997, の翻訳本に書かれているスマリヤンの二つの文とは、次のようなものです。
文1と文2
・文1 封筒を交換して増額する場合の増額は封筒を交換して半減する場合の減額を上回る。
・文2 それらの金額(増額と減額)は等しい。
文1の証明
交換前の金額を x とすると、封筒を交換して増額する場合の増額は x で、 封筒を交換して半減する場合の減額は x/2 なので、文1 が成り立つ。
文2の証明
二つの封筒の金額の差を d とすると、封筒を交換して増額する場合の増額は d で、 封筒を交換して半減する場合の減額も d なので、文2 が成り立つ。
命題2の証明で封筒の金額の差に着目しているのは、うまいトリックです。差が決まれば金額の組み合わせも決まってしまうことをうまく隠しています。

スマリヤンの二つの文のパラドックス
次のような矛盾を感じる錯覚現象がスマリヤンの二つの文のパラドックスです。
・スマリヤンの二つの文はどちらも正しい。
・スマリヤンの二つの文は両立しない。

スマリヤンのパラドックスの解明
スマリヤンの二つの文が両立することがあることは具体例を考えると一発でわかります。

千円札1枚の封筒と千円札2枚の封筒の組み合わせと、 千円札2枚の封筒と千円札4枚の封筒の組み合わせがあるときに、 封筒を一つ選んだときのことを考える。 ← 2015/01/11 に訂正

選んだ封筒の金額を特定した場合
選んだ封筒が 2千円だったとする。
封筒を交換して半減したら千円の損で得したら 2千円の得で得の方が大きい。

二つの封筒の金額の組み合わせを特定した場合
選んだ封筒の一方は千円で他方が 2千円だったとする。
選んだ封筒が 2千円だったら交換して千円損し、選んだ封筒が千円だったら交換して千円得するので損と得は等しい。

次の点がポイントです。
・文1 の場合、選んだ封筒の金額を特定して、その範囲に絞って場合分けを考えている。
・文2 の場合、二つの封筒の金額の組み合わせを特定して、その範囲に絞って場合分けを考えている。
この二つを区別できない心理を私は スマリヤンの錯覚 と呼んでいます。
0739132人目の素数さん
垢版 |
2024/03/13(水) 17:26:19.99ID:ascKCvNK
∃x+1.は普通は構文エラーだろ、なくてもいいとかそういう次元じゃない
x+1が変数だというなら、他の人が読めるようにα変換して普通の変数で書き直せよ
0740132人目の素数さん
垢版 |
2024/03/13(水) 17:36:44.52ID:5iS9phMp
君x+1が変数でないことを示せなかったじゃん
0741132人目の素数さん
垢版 |
2024/03/13(水) 17:54:49.02ID:ascKCvNK
お前が変数だと主張するなら勝手にすればいいよ
x+1が変数ではないってのは一般常識の話なんだから
数学では普通は変数はアルファベット一文字が常識で、たまに長い単語を使うことがあるし、君はその延長でx+1も変数に入れたかったんだろ勝手にしろよ
0742132人目の素数さん
垢版 |
2024/03/13(水) 18:10:43.89ID:5iS9phMp
じゃ黙ってろよw
0743132人目の素数さん
垢版 |
2024/03/13(水) 18:13:15.13ID:ascKCvNK
x+1が仮に変数だとして
>∀x∈ℕ.∃(x+1).x<x+1
これをα変換したら
∀x∈ℕ.∃y.x<x+1
なのか
∀x∈ℕ.∃y.x<y
なのかすら不明瞭な状態で

>すると
>∀x∈ℕ.x<x+1
>と
>∀x∈ℕ.∃(x+1).x<x+1
>の違いは何だと言ってるの?
こんなのに答えろっていうのが馬鹿げてる
0744132人目の素数さん
垢版 |
2024/03/13(水) 18:50:41.96ID:5iS9phMp
>x+1が仮に変数だとして
仮とは? 変数か変数でないかどちらか 君は変数であるとの主張に反論できなかった

>>∀x∈ℕ.∃(x+1).x<x+1
>これをα変換したら
>∀x∈ℕ.∃y.x<x+1
>なのか
>∀x∈ℕ.∃y.x<y
>なのかすら不明瞭な状態で
じゃα変換しなきゃいいじゃん
0745132人目の素数さん
垢版 |
2024/03/13(水) 19:09:26.68ID:ascKCvNK
>>744
α変換しないと最後のx+1が変数なのかxに1を足したのかどっちなのか曖昧だろ
どっちとも取れる状態で質問しないでくれますか?
0746132人目の素数さん
垢版 |
2024/03/13(水) 19:22:12.59ID:in9dXeLi
>>736
何か付けるなら
∃+:N✕N→N.∀x∈ℕ.x<x+1
だろうな
0747132人目の素数さん
垢版 |
2024/03/13(水) 19:26:22.14ID:in9dXeLi
>>738
>> 1は「スマリヤンの錯覚」に陥ってますな
>それ面白いね
わかりもせずに「面白いね」と脊髄反射で言う奴、いるよね

>「スマリヤンの錯覚」は、下記著者の造語ですね
だから?
なんかリコウぶってつまんないこという奴、いるよね
0748132人目の素数さん
垢版 |
2024/03/14(木) 00:16:18.46ID:Wqp8i7yx
>>747
アホがしゃしゃり出るねw

1)「スマリヤンの錯覚」の定義は?
 それを明確にしたのが>>738だぞ
2)もう一人のアホが『「スマリヤンの錯覚」に陥ってますな』
 と宣うから、「スマリヤンの錯覚」が一般化されていると思ったのだが
 あに図らんや 単にブログの筆者の造語じゃないかよww

リコウぶってるのは、おまえだ!

・定義の確認しないで、議論するやつが数学科出身だって? わらかすな
・用語を勝手に使って議論する? どこかの大学の教授が造語するならともかく
 チンピラがうれしがってワケワカ用語で議論して何になるんだ?w
0749132人目の素数さん
垢版 |
2024/03/14(木) 03:06:10.51ID:Wz/uoV5i
結局これは書いてる本人でもどっちなのか分からない状態で書いてたってことと理解したんでいいんかね?

x+1が仮に変数だとして
>∀x∈ℕ.∃(x+1).x<x+1
これをα変換したら
∀x∈ℕ.∃y.x<x+1
なのか
∀x∈ℕ.∃y.x<y
なのかすら不明瞭
0750132人目の素数さん
垢版 |
2024/03/14(木) 05:37:52.63ID:bNpw3CSv
>>748
🐎🦌、怒り●う

>・・・の定義は?
🐎🦌は知らん言葉を聞くと脊髄反射で「定義は?」と絶叫
そもそもリンク貼ってあったんだから、そこに書いてあるって悟れよ

>『・・・に陥ってますな』と宣うから、
>・・・が一般化されていると思ったのだが
🐎🦌はなんでも勝手に思い込む 完全な🌲違いですな

>定義の確認しないで、議論するやつが数学科出身だって? わらかすな
>用語を勝手に使って議論する? どこかの大学の教授が造語するならともかく
>チンピラがうれしがってワケワカ用語で議論して何になるんだ?
🐎🦌は中身がなにか分かってないものが確率変数、とか身勝手定義をでっち上げる
ま、大学入試に四度落ちて諦めた数学ド素人じゃしゃあない
関数のリーマン積分可能条件も、線形写像の正則性の条件も知らん
数学界ではまったくの”土人”だな ど・じ・ん
0751132人目の素数さん
垢版 |
2024/03/14(木) 10:31:39.93ID:IoTgOBI5
>>750
>>・・・の定義は?
>知らん言葉を聞くと脊髄反射で「定義は?」と絶叫
>そもそもリンク貼ってあったんだから、そこに書いてあるって悟れよ

影山 利郎氏の著書「素人と玄人」(下記)に
”プロ(玄人)は基本が身についている”みたいな教えが書いてあった
『知らん言葉を聞くと脊髄反射で「定義は?」と絶叫』は
数学やるなら普通だろ?w

>>『・・・に陥ってますな』と宣うから、
>>・・・が一般化されていると思ったのだが
>なんでも勝手に思い込む 完全な🌲違いですな

定義なしで、「スマリヤンの錯覚」と出してきた
普通は一般化された学術用語(例えば「数学辞典」に載っているなど)
と思うだろうよ
なんのために学術用語が決められているのか、分かってないんか
数学の基本が身についていない
ど素人だな

(参考)
https://ja.wikipedia.org/wiki/%E5%BD%B1%E5%B1%B1%E5%88%A9%E9%83%8E
影山 利郎(かじやま としろう、1926年6月21日 - 1990年7月31日)は、日本の囲碁棋士である。静岡県出身[1][2]。
師匠 安永一
概要
1926年(大正15年)6月21日、静岡県に生まれる。16歳になった1942年(昭和17年)から囲碁を習い始め、1948年(昭和23年)の全日本素人本因坊戦で優勝し、翌年の秋にプロ入り(初段)を果たした[1][2]。
執筆家としての側面も持ち、多数の著書がある[2]。
主な書籍
影山利郎 (2013年). 素人と玄人: 徹底分析、これだけ違う両者の視点. 日本棋院. ISBN 9784818206113 1971年初版の本の再刊
0752132人目の素数さん
垢版 |
2024/03/14(木) 10:39:44.18ID:mL8LcQVb
>>751
馬鹿なこと言ってないで>583 >597に答えてもらえませんか?
なぜ黙殺しようとするのですか?
0753132人目の素数さん
垢版 |
2024/03/14(木) 19:11:58.15ID:bNpw3CSv
>>751
なんか🌲違いがぐだぐだ喚いてるが肝心のページの文章読んだか?
http://the-apon.com/coffeedonuts/illusions-have-same-root.html

金額だけ見た場合、それは少額のほうかもしれんし高額のほうかもしれん
しかしそれぞれの確率が1/2ずつだと決めつける理由はなにもない
2つの封筒から1つを選んでそれが少額が高額か、の確率とは全く違う

違うものを同じと妄想するのが「スマリヤンの錯覚」
0754132人目の素数さん
垢版 |
2024/03/14(木) 23:40:23.51ID:mKpj542N
結局これはどっちなんや?

x+1が仮に変数だとして
>∀x∈ℕ.∃(x+1).x<x+1
これをα変換したら
∀x∈ℕ.∃y.x<x+1
なのか
∀x∈ℕ.∃y.x<y
なのかすら不明瞭
0755132人目の素数さん
垢版 |
2024/03/15(金) 05:51:24.65ID:ATM0vb6x
>>754
自分でα変換いうてるやん
定義しってんなら
∀x∈ℕ.∃(x+1).x<x+1 が
∀x∈ℕ.∃y.x<y になるしかないやん
0756132人目の素数さん
垢版 |
2024/03/15(金) 05:55:10.72ID:ATM0vb6x
>>753
(2つの封筒で)
>金額だけ見た場合、それは少額のほうかもしれんし高額のほうかもしれん
>しかしそれぞれの確率が1/2ずつだと決めつける理由はなにもない
>2つの封筒から1つを選んでそれが少額が高額か、の確率とは全く違う

モンティ・ホールも同じ
3つのドアのうち1つしかない賞品のドアを選ぶ確率と
1つ1つのドアの後ろに賞品がある確率は同じではない

違うものを同じと妄想するのが「スマリヤンの錯覚」
0757132人目の素数さん
垢版 |
2024/03/15(金) 07:42:24.54ID:xEd6gXjp
箱入り無数目も同じ
100個の箱のうち1つしかないハズレの箱を選ぶ確率と
1つ1つの箱がハズレである確率は同じではない

違うものを同じと妄想するのが「スマリヤンの錯覚」
0758132人目の素数さん
垢版 |
2024/03/15(金) 07:57:44.82ID:sYXmV0f/
>>756
>違うものを同じと妄想するのが「スマリヤンの錯覚」

君のは、全然説明になってないと思うよ ;p)
下記の 彼の”私の造語”「スマリヤンの錯覚」の説明
『封筒を交換したらどうなるかを考えるときに、選んだ封筒の金額を条件として考えるやり方と、二つの封筒の金額の組み合わせを条件として考えるやり方が、同じ問題を考えているという錯覚です』
を読んで意味分かるか?

他人に分かる用語解説になってないと思うよ
錯覚だから、何かの”思い違い 勘違い”なのは当然だが(下記)

たぶん、これは 人の推論の一般のやり方と、二つの封筒問題やモンティ・ホール問題での正解とが、ちょっと違うんだろう
”人の推論の一般のやり方”を、まず解説しないと、「錯覚」の説明にならないと思うよ

(参考)
http://the-apon.com/coffeedonuts/illusions-have-same-root.html
モンティ・ホール問題好きのホームページ
二つの封筒問題の錯覚とスマリヤンの錯覚の源は一つか 2014/03/23
用語解説
スマリヤンの錯覚
私の造語です。
封筒を交換したらどうなるかを考えるときに、選んだ封筒の金額を条件として考えるやり方と、二つの封筒の金額の組み合わせを条件として考えるやり方が、同じ問題を考えているという錯覚です。
この錯覚に罹った人は、二封筒問題のおまじないの王様を唱えたり、スマリヤンの二つの文のパラドックスに罹ったりします。

https://ja.wiktionary.org/wiki/%E9%8C%AF%E8%A6%9A#:~:text=%E5%90%8D%E8%A9%9E&text=(%E5%BF%83%E7%90%86%E5%AD%A6)%20%E3%81%82%E3%82%8B%E7%89%A9%E3%81%AB%E5%AF%BE%E3%81%99%E3%82%8B,%E5%8B%98%E9%81%95%E3%81%84%E3%80%82
錯 覚(さっかく)
1.(心理学) ある物に対する知覚が実際の物と異なること。
2.思い違い。勘違い。

https://ja.wikipedia.org/wiki/%E9%8C%AF%E8%A6%9A
錯覚
心理学でいう錯覚とは、間違いや誤りの類いでは無い。注意深く観察しても、予備知識があっても生じてしまう、人間の感覚・知覚特性によって作り出される現象を指す[1]。
錯覚の種類
錯覚はその原因により大きく4つに分けることができる。
・不注意性錯覚
 対象物への注意が不十分のために起こる錯覚。見間違い、聞き違い、人違いなど、われわれが日常経験する多くの間違いを含んでいる。
・感動錯覚
 暗くて怖い場所を歩いていると、物の影が人影に見えたり、何でもない物音を人の気配に感じることがある。恐怖や期待などの心理状態が知覚に影響を与えるものである。
・パレイドリア
 雲の形が顔に見えたり、しみの形が動物や虫に見えたりと、不定形の対象物が違ったものに見える現象に代表される。対象物が雲やしみであることは理解しており、顔や動物ではないという批判力も保っているが、一度そう感じるとなかなかその知覚から逃れられない。熱性疾患の時にも現れやすい。
・生理的錯覚
 数多く知られている幾何学的錯視や、音階が無限に上昇・下降を続けるように聞こえるシェパード・トーンなどのように、対象がある一定の配置や状態にあると起こる錯覚。誰にでもほぼ等しく起こる。
0759132人目の素数さん
垢版 |
2024/03/15(金) 08:10:17.15ID:sYXmV0f/
時枝の錯覚も、同様に説明できる
1)人は、二つの決定番号 daとdb で、確率 P(da>db)=1/2 と思ってしまう
2)ところが、決定番号 daとdbは、自然数N全体を渡り、自然数N全体は無限集合だから
 ∞/∞ の不定形になり、確率計算 1/2 は正当化できない

こういうことでしょうね ;p)
0760132人目の素数さん
垢版 |
2024/03/15(金) 08:55:09.63ID:8QDMDRfQ
日本棋院アーカイブの続刊が楽しみ
0761132人目の素数さん
垢版 |
2024/03/15(金) 09:01:01.85ID:xEd6gXjp
>>759
馬鹿なこと言ってないで>583 >597に答えてもらえませんか?
なぜ黙殺しようとするのですか?
0762132人目の素数さん
垢版 |
2024/03/15(金) 10:02:56.06ID:tFlszaLY
>>755
>>>754
>自分でα変換いうてるやん
>定義しってんなら
>∀x∈ℕ.∃(x+1).x<x+1 が
>∀x∈ℕ.∃y.x<y になるしかないやん

またまた ずさんなことを言うw

∀x∈ℕ.∃(x+1).x<x+1
 ↓
∀x∈ℕ.∃y.x<y
にするためには、y=x+1と定義しないと。その定義が必要でしょ?

一般に
・∀x∈ℕ.∃(x+1).x<x+1
・∀x∈ℕ.∃y.x<y

この二つの式は意味違うし、そもそも”∃y∈ℕ”とかも要りそうに思うけど
まあ、ともかくあんたの思考は、ずさんそのものだね ;p)

(参考)(α-変換)
https://ja.wikipedia.org/wiki/%E3%83%A9%E3%83%A0%E3%83%80%E8%A8%88%E7%AE%97
ラムダ計算
ラムダ計算(ラムダけいさん、英語: lambda calculus)は、計算模型のひとつで、計算の実行を関数への引数の評価(英語: evaluation)と適用(英語: application)としてモデル化・抽象化した計算体系である。ラムダ算法とも言う。関数を表現する式に文字ラムダ (λ) を使うという慣習からその名がある。アロンゾ・チャーチとスティーヴン・コール・クリーネによって1930年代に考案された。

歴史
元々チャーチは、数学の基礎となり得るような完全な形式体系を構築しようとしていた。彼の体系がラッセルのパラドックスの類型に影響を受けやすい(例えば論理記号として含意 → を含むなら、λx.(x→α) にYコンビネータを適用してカリーのパラドックスを再現できる)ということが判明した際に、彼はそこからラムダ計算を分離し、計算可能性理論の研究のために用い始めた。この研究からチャーチは一階述語論理の決定可能性問題を否定的に解くことに成功した。

α-変換
アルファ変換の基本的なアイデアは、束縛変数の名前は重要ではない、ということにある。例えば、 λx. x と λy. y は同じ関数を表している。
しかし、ことはそう単純ではない。
ある束縛変数の名前を置換してもよいかどうかには、いくつかの規則が絡んでくる。例えば、ラムダ式 λx. λy. x 中の変数 x を y に置き換えると、 λy. λy. y となるが、これは最初の式とはまったく異なるものを表すことになる。
0763132人目の素数さん
垢版 |
2024/03/15(金) 10:40:46.74ID:tFlszaLY
ご参考
α-変換 ラムダ計算

https://scrapbox.io/mrsekut-p/%CE%B1%E5%A4%89%E6%8F%9B
α変換 scrapbox mrsekut-p
[/ alpha-conversion]
[$ \lambda x.x]と[$ \lambda z.z]は同じだよね、
このような変換のことを[$ \alpha]変換という
つまり、[束縛変数]を別のものに入れ替えて全く同じ意味のラムダ抽象を作成する操作
名前の衝突を回避するときに使う
[$ \lambda x.(y\lambda y.yx)]のように、同じ`y`でも、[自由変数]と[束縛変数]が混在していて読みづらい
α変換を施し、別の文字を使った同値の式にする
例えば[$ \lambda x.(y\lambda z.zx)]
このとき[$ \lambda x.(z\lambda y.yx)]のように自由変数の方を置き換えない。
α変換では[* 束縛変数の方を置き換える]
この状態のことを「[変数条件]を満たす」と呼ぶ
定義
略す

https://web.sfc.keio.ac.jp/~hagino/mi15/07-ppt.pdf
情報数学第7回ラムダ計算
萩野達也
慶應義塾大学環境情報学部2015/11/10
P7
α変換と代入
略す
0764132人目の素数さん
垢版 |
2024/03/15(金) 18:13:58.84ID:ATM0vb6x
>>762
>・∀x∈ℕ.∃(x+1).x<x+1
>・∀x∈ℕ.∃y.x<y
>この二つの式は意味違うし

意味違うとわめく根拠は、後者のyは例えばx+2でもいいとか、そういうことかい?

じゃ、以下は同じ意味かい

∀x ∈ ℕ. x<x+1∧∀z ∈ ℕ. x<z ⇒ x+1<=z
∀x ∈ ℕ. ∃y ∈ ℕ. x<y∧∀z ∈ ℕ. x<z ⇒ y<=z
0765132人目の素数さん
垢版 |
2024/03/15(金) 18:23:14.68ID:Elhq9FcO
>>755
x+1が変数だと主張する異常な論理式相手してるんだから、どっちともとれるやろ
完全に常識の範囲外のことやってるんだから
0766132人目の素数さん
垢版 |
2024/03/15(金) 19:52:49.15ID:ATM0vb6x
α変換と言い切った瞬間に∀x∈ℕ.∃y.x<yにきまる
それ以外はα変換でない
0767132人目の素数さん
垢版 |
2024/03/15(金) 19:59:53.62ID:Elhq9FcO
>>766
なんでだよ
後ろのx+1が変数なのかxに1を足した式なのか書いた本人にしか分からんだろうが
0768132人目の素数さん
垢版 |
2024/03/15(金) 20:18:42.34ID:Elhq9FcO
変数記号 := { x, y, x+1 }
定数記号 := { 1 }
関数記号 := { + }
の設定で、x+1って書いてあったら変数なのか関数を適用した項なのか、誰にもわからんやろ
0769132人目の素数さん
垢版 |
2024/03/15(金) 22:56:36.63ID:sYXmV0f/
>>764
>>>762
>>・∀x∈ℕ.∃(x+1).x<x+1
>>・∀x∈ℕ.∃y.x<y
>>この二つの式は意味違うし
>意味違うとわめく根拠は、後者のyは例えばx+2でもいいとか、そういうことかい?

・例えば、yを有理数にとって
 ∀x∈ℕ.∃y∈Q.x<y とか
・例えば、yを実数にとって
 ∀x∈ℕ.∃y∈R.x<y とか
・だから、∃yで放り出すと、yは自然数に限らないから 二つの式の意味は違う
 yを自然数に限っても、冒頭の二つの式の意味は違う
0770132人目の素数さん
垢版 |
2024/03/15(金) 23:09:49.73ID:xEd6gXjp
>>769
馬鹿なこと言ってないで>583 >597に答えてもらえませんか?
なぜ黙殺しようとするのですか?
0771132人目の素数さん
垢版 |
2024/03/16(土) 02:16:26.20ID:mUoFzVnS
>∀x∈ℕ.∃(x+1).x<x+1
はそもそも論理式としてぶっ壊れてるんだからハナから無視しとけよ

やるなら
∀x∈ℕ.x<x+1
∀x∈ℕ.∃y.x<y
の2つを比較しろよ
0772132人目の素数さん
垢版 |
2024/03/16(土) 08:15:19.83ID:/v13gW+O
>>771
比較しろよってw
やりたきゃ自分でやりな
そうでなきゃ黙ってな

で、>>260が間違いであることは理解したの?
0773132人目の素数さん
垢版 |
2024/03/16(土) 10:56:50.80ID:vIT5CHVL
>>769
>例えば、yを有理数にとって ∀x∈ℕ.∃y∈Q.x<y とか
>例えば、yを実数にとって  ∀x∈ℕ.∃y∈R.x<y とか
>∃yで放り出すと、yは自然数に限らないから 二つの式の意味は違う
∀x∈ℕ.∃y∈ℕ.x<y でないと考えねばならない理由があるか?全くない
したがって上記は却下

>yを自然数に限っても、冒頭の二つの式の意味は違う
だから何がどう違うのか?
以下の>>764に全く答えられないのは
君が大学に入れなかった高卒素人だからか?

後者のyは例えばx+2でもいいとか、そういうことかい?
じゃ、以下は同じ意味かい
∀x ∈ ℕ. x<x+1∧∀z ∈ ℕ. x<z ⇒ x+1<=z
∀x ∈ ℕ. ∃y ∈ ℕ. x<y∧∀z ∈ ℕ. x<z ⇒ y<=z
0774132人目の素数さん
垢版 |
2024/03/16(土) 11:03:56.20ID:vIT5CHVL
>>758
>『封筒を交換したらどうなるかを考えるときに、
>選んだ封筒の金額を条件として考えるやり方と、
>二つの封筒の金額の組み合わせを条件として考えるやり方が、
>同じ問題を考えているという錯覚です』
>を読んで意味分かるか?

日本人だからわかるよ 日本語わかるから

上は「もう一方の封筒が自分の封筒の金額の2倍か2分の一か」
下は「二つの封筒のうち低い方を選んだか高い方を選んだか」

下の確率はどちらも1/2だが
このことから
上の確率もどちらも1/2だ
といえるというのは
数学でもなんでもない

>他人に分かる用語解説になってないと思うよ

いや、君は日本語分からんニホンザルだから
君はヒトではないよ 「思う」も要らない
君はヒトではなくサル サルは数学板に書いちゃダメ
サルは数学に興味もつな 無意味だから
0775132人目の素数さん
垢版 |
2024/03/16(土) 11:07:38.18ID:vIT5CHVL
>>758
ニホンザル語
>たぶん、・・・んだろう
>と思うよ

ヒトの言葉が理解できないので
「たぶん」「だろう」「と思うよ」
ということばで誤魔化す

当然ながら国立大学どころか私立のFランク大学も受からん
0776132人目の素数さん
垢版 |
2024/03/16(土) 11:15:44.88ID:vIT5CHVL
>>758
>これは
>人の推論の一般のやり方と、
>二つの封筒問題やモンティ・ホール問題での正解
>とが、ちょっと違う
>”人の推論の一般のやり方”
>を、まず解説しないと、
>「錯覚」の説明にならない

誤 人の推論
正 サルの脊髄反射

「サルの脊髄反射」とは、ズバリ
「分からんものは、脊髄反射で確率変数と考える」
ということ

「二つの封筒」の封筒中身、然り
「モンティ・ホール問題」のドアの後ろ側、然り

脊髄反射でそう考えるサルは必ず間違えるw

たしかに自分の封筒の中身が10000円だったとき
相手の封筒は20000円かもしれんし、5000円かもしれん

しかし、それは確率変数ではない つまり
20000円の確率P、5000円の確率1-P
と考えるのは人間失格のサルってこと

モンティ・ホール問題もそう
3つのドアA、B、Cについて
Aのドアの後ろに賞品がある確率 P
Bのドアの後ろに賞品がある確率 Q
Cのドアの後ろに賞品がある確率 1-P-Q
と考えるのは人間失格のサルってこと
0777132人目の素数さん
垢版 |
2024/03/16(土) 11:20:00.21ID:vIT5CHVL
モンティ・ホール問題のシミュレーションで
3つのドアの後ろの賞品の配置を等確率1/3で変え
回答者のドアの選択は(例えばAに)固定する
というのは最大の誤りである

そうではなく
3つのドアの後ろの賞品の配置は(例えばAに)固定する
回答者のドアの選択はA,B,Cそれぞれ確率1/3ずつに割り振る
という形でのみシミュレーションすべきである

問題は固定である
回答者は3つのドアのどれでも選択できる
確率1/3ずつなのは、ドアの選択であって
ドアの後ろに賞品がある確率ではない!
0778132人目の素数さん
垢版 |
2024/03/16(土) 11:26:45.61ID:+LjGwmYz
>>773
>>>769
>>例えば、yを有理数にとって ∀x∈ℕ.∃y∈Q.x<y とか
>>例えば、yを実数にとって  ∀x∈ℕ.∃y∈R.x<y とか
>>∃yで放り出すと、yは自然数に限らないから 二つの式の意味は違う
>∀x∈ℕ.∃y∈ℕ.x<y でないと考えねばならない理由があるか?全くない
>したがって上記は却下

・そうかな?
 例えば、∃y∈C(複素数)としよう
 そうすれば、∀x∈ℕ.∃y∈C.x<y となるけど
 そもそも、∃y∈C(複素数)は 一般には 不等号 < は適用できないぞ(下記)
・だから、∃yがどの範囲の数なのかを、論理式を書いた人が明示しないと
 意味ある論理式にならないと思うけどね ;p)

(参考)
https://math-fun.net/20210522/14166/
趣味の大学数学
複素数で普通の順序・不等号・大小関係を考えないのはなぜか
2021年5月22日 木村

高校数学以降では、複素数の扱いを学びます。

実数では
0<1といったように大小比較ができますが、複素数ではそのような比較を考えません。それはなぜでしょうか。

簡単に言えば、仮に
0<iというような順序関係があったとすると、両辺に
iをかけると
0<−1となってしまうからです。

今回は、複素数では「普通の」順序・不等号を定義できないことを紹介します。

目次
順序とは何か
順序関係とは
複素数に「普通の」順序が定まらないこと
複素数でも順序を考えること自体はできる

複素数でも、全順序という順序を考えること自体はできるのです。しかし、普通の順序=和と積と両立するような順序を考えることはできない、そういうものがあったとすると矛盾するというのが今回の話でした。

ちなみに、ベクトルに対しても辞書式順序を考えることはできます。しかし、演算と両立するような順序……と議論しようとすると、ベクトルではそもそも良い「積」を考えられません。ベクトルの内積は実数を返し、ベクトルを返さないです。外積はベクトルを返しますが、交換法則や結合法則を満たしません。

以上、複素数では普通の順序・不等号・大小関係を考えないのはなぜか、紹介してきました。

整数や実数で考えていたような、和と積と両立させる性質を持った順序を考えることはできない、というのが理由です。
0779132人目の素数さん
垢版 |
2024/03/16(土) 11:34:27.13ID:vIT5CHVL
>>778
どうでもいいことばかりだらだか書いて
肝心なことは一字も書かない

さすがニホンザル
0780132人目の素数さん
垢版 |
2024/03/16(土) 11:58:34.85ID:/v13gW+O
>>778
馬鹿なこと言ってないで>583 >597に答えてもらえませんか?
なぜ黙殺しようとするのですか?
0781132人目の素数さん
垢版 |
2024/03/16(土) 12:42:52.79ID:+LjGwmYz
>>777
>確率1/3ずつなのは、ドアの選択であって
>ドアの後ろに賞品がある確率ではない!

違うよ
・下記の”素事象を格子状に配置すれば標本空間がよくわかる”の通り
 事象を2次元の格子図(デカルト積)に書けば良いんだよ
・簡単に説明すると、”挑戦者が選ぶ扉を扉1に限定して”
 扉1が外れの場合のみ説明する
・司会者は残る二つで当りが分かっていて、外れの扉を開ける
 だから、開けない扉が当りだ
・もし、司会者が当りが分かっていなくて当りの扉を開けたら、ノーカウントでやり直し
 しかし この場合にモンティ・ホール問題のルールが理解できれば、正解の選択ができて当てられるので当りの確率アップになるってこと

残念だった ;p)

(参考)
http://the-apon.com/coffeedonuts/matrix.html
モンティ・ホール問題好きのホームページ
素事象を格子状に配置すれば標本空間がよくわかる
2013/05/28
数学では確率を次のように定義している。
何かの集合を標本空間として、そのべき集合の一部で加算加法族になっているものを事象の集合とし、事象の集合に確率測度を与え、・・・・・・、(難しいので以下省略)
もっと直感的な定義もある。
確率変数の値域のデカルト積のべき集合の・・・・・、(難しいので以下省略)

2次元の格子図を使えば、このような数学の確率の定義に添いながら、条件付確率もしくは事後確率の説明ができるかも知れないという期待がある。格子図の交点が最も細かい事象、すなわち素事象 (prime events) に対応するので、仮説事象と証拠事象の関係がよくわかるのではないか、という期待もある。

モンティ・ホール問題に応用してみる
例によって、挑戦者が選ぶ扉を扉1に限定して考える。

ステップ2 確率変数の値域の直積として2次元のテーブルを書く
    当扉1      当扉2     当扉3
開扉1 当扉1, 開扉1 当扉2, 開扉1 当扉3, 開扉1
開扉2 当扉1, 開扉2 当扉2, 開扉2 当扉3, 開扉2
開扉3 当扉1, 開扉3 当扉2, 開扉3 当扉3, 開扉3

結論
上記のように素事象を2次元に配列した図表を使う方法には、次のような利点がある。
・同一の図を使って、証拠事象が特定的な場合と、不特定な場合の両方の確率が計算できる。
・決定木などの階層的標本空間分析手法や、ベイズ推定などの条件付確率の公式を使わなくても確率を計算できるので、 より基本的な理解が可能になる

つづく
0782132人目の素数さん
垢版 |
2024/03/16(土) 12:43:09.20ID:+LjGwmYz
つづき

https://ja.wikipedia.org/wiki/%E3%83%A2%E3%83%B3%E3%83%86%E3%82%A3%E3%83%BB%E3%83%9B%E3%83%BC%E3%83%AB%E5%95%8F%E9%A1%8C
モンティ・ホール問題
一種の心理トリックになっており、確率論から導かれる結果を説明されても、なお納得しない者が少なくないことから、モンティ・ホール・ジレンマ、モンティ・ホール・パラドックスとも称される。「直感で正しいと思える解答と、論理的に正しい解答が異なる問題」の適例とされる。
概要
「<投稿された相談>
プレーヤーの前に閉じた3つのドアがあって、1つのドアの後ろには景品の新車が、2つのドアの後ろには、はずれを意味するヤギがいる。プレーヤーは新車のドアを当てると新車がもらえる。プレーヤーが1つのドアを選択した後、司会のモンティが残りのドアのうちヤギがいるドアを開けてヤギを見せる。
ここでプレーヤーは、最初に選んだドアを、残っている開けられていないドアに変更してもよいと言われる。
ここでプレーヤーはドアを変更すべきだろうか?」
1990年9月9日発行、ニュース雑誌「Parade」にてマリリン・ヴォス・サヴァントが連載するコラム「マリリンにおまかせ」で、上記の読者投稿による質問に「正解は『ドアを変更する』である。なぜなら、ドアを変更した場合には景品を当てる確率が2倍になるからだ」と回答。すると直後から、読者からの「彼女の解答は間違っている」との約1万通の投書が殺到し、本問題は大議論に発展した
(引用終り)
以上
0783132人目の素数さん
垢版 |
2024/03/16(土) 13:11:37.58ID:+LjGwmYz
>>781
>>>777
>>確率1/3ずつなのは、ドアの選択であって
>>ドアの後ろに賞品がある確率ではない!
>・下記の”素事象を格子状に配置すれば標本空間がよくわかる”の通り
> 事象を2次元の格子図(デカルト積)に書けば良いんだよ
>・もし、司会者が当りが分かっていなくて当りの扉を開けたら、ノーカウントでやり直し
> しかし この場合にモンティ・ホール問題のルールが理解できれば、正解の選択ができて当てられるので当りの確率アップになるってこと

・この教訓は、"事象を2次元の格子図(デカルト積)に書けば良い"!ってことだ
・「ドアの後ろに賞品がある」場合を、キチンと数え上げれば良い!

なお、「直感で正しいと思える解答と、論理的に正しい解答が異なる」のは
ルールの「司会のモンティが残りのドアのうちヤギがいるドアを開けてヤギを見せる」
の意味が直感では、把握が難しい
だから、ルールを正しく生かした選択に至らない人が 少なくないってことだね

時枝の二つの決定番号 da,db ∈N(可算無限集合)で
確率P(da>db)=1/2 と錯覚するが如し
(∵ N(可算無限集合)だから、∞/∞ で不定形で、1/2はいえない)
けれども、人はついつい 確率P(da>db)=1/2 と錯覚するのです
0784132人目の素数さん
垢版 |
2024/03/16(土) 14:25:12.12ID:/v13gW+O
モンティホール問題、二つの封筒問題、箱入り無数目
いずれも1回の出題でも確率が定義できなければならないので出題は試行にできない。

例えば二つの封筒問題で最初に開封した封筒の中身が1万円だった場合、他方は5千円か2万円であるが、1回の出題では必ずどちらかであって、確率P(0<P<1)で5千円・確率1-Pで2万円なんてことにはならない。
従って期待値計算:5千円×(1/2)+2万円×(1/2)は誤り。

このように錯覚してしまうのは、無意識に出題を試行としている為であろう。
しかし繰り返すが、出題を試行としてしまうと1回の出題での確率は定義できない。

確率を考えるときは試行が何かを明確にする必要がある。
0785132人目の素数さん
垢版 |
2024/03/16(土) 14:33:59.63ID:/v13gW+O
>>783
>時枝の二つの決定番号 da,db ∈N(可算無限集合)で
>確率P(da>db)=1/2 と錯覚するが如し
>(∵ N(可算無限集合)だから、∞/∞ で不定形で、1/2はいえない)
1/2は言えないは正しいが、その理由は大間違い
そして時枝証明はそもそも1/2を論拠にしていないからまったくナンセンス

馬鹿なこと言ってないで>583 >597に答えてもらえませんか?
なぜ黙殺しようとするのですか?
0786132人目の素数さん
垢版 |
2024/03/16(土) 15:05:34.46ID:+LjGwmYz
>>784
>例えば二つの封筒問題で最初に開封した封筒の中身が1万円だった場合、他方は5千円か2万円であるが、1回の出>題では必ずどちらかであって、確率P(0<P<1)で5千円・確率1-Pで2万円なんてことにはならない。
>従って期待値計算:5千円×(1/2)+2万円×(1/2)は誤り。

違うよ
・下記の>>575より 再録の通りです
 期待値計算 1万2千500円が正しい場合があるよ
・設定を変えよう
 下記 組合わせ 二つ、{5千円、1万円}と{1万円、2万円}は同じだが
 予算がないので、{1万円、2万円}は5回に1回に減らし、{5千円、1万円}を5回に4回とする
・この場合、封筒を開けて1万円として もし封筒を取り替えたときの期待値は
 2万円x(1/5)+5千円x(4/5)=8千円 となる
・つまりは、組合わせ{5千円、1万円}と{1万円、2万円}で
 組合わせの出現頻度が変わると
 当然、期待値は変わる

 >>575より 再録
(引用開始)
1)ある大学において 学生の奨励として、学長賞で賞金を出すことにした
 1年に10回(夏休み8月とクリスマス休暇の12月を除く)、学年のトップ(1番の人)に
 封筒は二つ使う。そして組合わせが二つ、{5千円、1万円}と{1万円、2万円}と
 どの組合わせを使うかは、ランダムで等確率として、学長のみが知る
 授賞式の事務員は知らない
2)事務員がルールを説明する
「封筒二つで、片方の倍か半分かで。一つの封筒を開けて見て良い。別の封筒に取り替える権利がある。
 もちろん、取り替えないのも可」と
 但し、具体的金額は教えない(説明する事務員も知らない)
3)この場合
 開けた封筒が、1万円ならば
 {5千円、1万円}と{1万円、2万円}が等確率で考えられる
 従って、取り替えると 5千円と2万円が等確率で出現するので
 期待値は、1万2千500円です
(引用終り)
0787132人目の素数さん
垢版 |
2024/03/16(土) 15:16:48.03ID:/v13gW+O
>>786
何の反論にもなってない

馬鹿なこと言ってないで>583 >597に答えてもらえませんか?
なぜ黙殺しようとするのですか?
0788132人目の素数さん
垢版 |
2024/03/16(土) 15:23:41.98ID:vIT5CHVL
>>781
>>確率1/3ずつなのは、ドアの選択であって
>>ドアの後ろに賞品がある確率ではない!
>違うよ
いちいち口答えすんな、ニホンザル

>”素事象を格子状に配置すれば標本空間がよくわかる”の通り
>事象を2次元の格子図(デカルト積)に書けば良いんだよ
それ、当選扉を扉1に限定して
回答者が選ぶ扉とホストが開ける扉を事象として
2次元の格子図(デカルト積)に書かなくちゃウソだけどw

    選扉1     選扉2     選扉3
開扉1 選扉1、開扉1 選扉2、開扉1 選扉3、開扉1
開扉2 選扉1、開扉2 選扉2、開扉2 選扉3、開扉2
開扉3 選扉1、開扉3 選扉2、開扉3 選扉3、開扉3

回答者が選ぶ扉こそ等確率であるので、
p(選扉1, 開扉1) + p(選扉1, 開扉2) + p(選扉1, 開扉3) = 1/3
p(選扉2, 開扉1) + p(選扉2, 開扉2) + p(選扉2, 開扉3) = 1/3
p(選扉3, 開扉1) + p(選扉3, 開扉2) + p(選扉3, 開扉3) = 1/3

ホストは挑戦者が選んだ扉を開けないので、
p(選扉1, 開扉1) = 0
p(選扉2, 開扉2) = 0
p(選扉3, 開扉3) = 0

ホストは当たり扉を開けないので
p(選扉1, 開扉1) = 0
p(選扉2, 開扉1) = 0
p(選扉3, 開扉1) = 0

ホストが開ける扉に偏りがないので、
p(選扉1, 開扉2) = p(選扉1, 開扉3)

確率の総和は1なので、
p(選扉1, 開扉1) + p(選扉1, 開扉2) + p(選扉1, 開扉3) +
p(選扉2, 開扉1) + p(選扉2, 開扉2) + p(選扉2, 開扉3) +
p(選扉3, 開扉1) + p(選扉3, 開扉2) + p(選扉3, 開扉3) = 1

上記の手がかりからテーブルの各マス(素事象)の確率が求まる
0  0  0
1/6 0  1/3
1/6 1/3 0

ホストが例えば扉3を開いたという証拠事象の範囲だけ切り出せば
1/6 1/3 0

2倍すれば総和は1になる
1/3 2/3 0

したがって
選んだ扉が当たりの確率1/3
残った扉が当たりの確率2/3

まったく同じ形で説明できる
だからニホンザルの主張をまったく補強しない
残念だったな ギャハハハハハハ!!!
0789132人目の素数さん
垢版 |
2024/03/16(土) 15:30:43.42ID:vIT5CHVL
>>783
>・この教訓は、"事象を2次元の格子図(デカルト積)に書けば良い"!ってことだ
「格子図に書けば良い」のは確かだが
当たりの配置は事象ではないw

>・「ドアの後ろに賞品がある」場合を、キチンと数え上げれば良い!
キチンと数えるのは「どのドアを選ぶか」

>「直感で正しいと思える解答と、論理的に正しい解答が異なる」のは
>ルールの「司会のモンティが残りのドアのうちヤギがいるドアを開けてヤギを見せる」
>の意味が直感では、把握が難しいから
思考力ゼロのサルには理解できんかw

>ルールを正しく生かした選択に至らない人が 少なくないってことだね
ヒトはわかる サルにはわからん

この世のホモ・サピエンスの9割はヒトではなくサルってこった
まあ、サルのほうが自然に適応できてるかもな
野生動物は字が読めんでも数が数えられんでも生きてるからな

どうだ、嬉しいか?考える能力がないサル ギャハハハハハハ!!!
0790132人目の素数さん
垢版 |
2024/03/16(土) 15:34:42.46ID:vIT5CHVL
>>786
>>例えば二つの封筒問題で最初に開封した封筒の中身が1万円だった場合、
>>他方は5千円か2万円であるが、1回の出題では必ずどちらかであって、
>>確率P(0<P<1)で5千円・確率1-Pで2万円なんてことにはならない。
>>従って期待値計算:5千円×(1/2)+2万円×(1/2)は誤り。
>違うよ
いちいち口答えすんな、ニホンザル

>期待値計算 1万2千500円が正しい場合があるよ
>設定を変えよう

設定を変えるのはNG
だから正しい「場合がある」はNG

迷わず地獄に堕ちるが良い ニホンザル
ギャハハハハハハ!!!
0791132人目の素数さん
垢版 |
2024/03/16(土) 15:42:51.64ID:vIT5CHVL
>>783
>時枝の二つの決定番号 da,db ∈N(可算無限集合)で
>確率P(da>db)=1/2 と錯覚するが如し
>(∵ N(可算無限集合)だから、∞/∞ で不定形で、1/2はいえない)
>けれども、人はついつい 確率P(da>db)=1/2 と錯覚するのです

そもそも確率P(da>db)を考えるのがニホンザルw
da>db かつ da<db ということはない
つまり、成り立つのは2つの式たかだか1つである

ここでda>dbとする(固定!)

daを選ぶかdbを選ぶかは1/2ずつ
これが真の確率事象である

決して無作為にda,dbを選んで
da>dbとなる確率を求めるのではない!

何が確率事象かを間違えるのはニホンザル
正しく理解するのが日本人!

悔しいか?大学入れん数学ド素人ニホンザル

ギャハハハハハハ!!!
0792132人目の素数さん
垢版 |
2024/03/16(土) 15:44:52.64ID:/v13gW+O
>>783
>・「ドアの後ろに賞品がある」場合を、キチンと数え上げれば良い!
1回の出題では1通りしかないから数え上げようが無い
君が言ってるのは出題が試行の場合 君は無意識に出題を試行としている
だが出題を試行としてしまうと1回の出題における確率は定義されない

分かる?

wikipediaより引用
「確率論において、試行(しこう、英: trial, experiment)とは、起こりうる結果がいくつかあり、そのどれか1つだけが偶然で起こる流れのことである」
0793132人目の素数さん
垢版 |
2024/03/16(土) 15:47:59.31ID:vIT5CHVL
【結論】
サルは脊髄反射で「問題が確率変数だ!」と考えるがそれが間違い
0794132人目の素数さん
垢版 |
2024/03/16(土) 15:50:14.96ID:vIT5CHVL
【結論】
サルは無意識に「俺の選択は決定事項!確率変数じゃない」と考えるがそれも間違い
0795132人目の素数さん
垢版 |
2024/03/16(土) 15:51:21.32ID:vIT5CHVL
【結論】
サルは回答者の視点でしか考えられない 出題者の視点で考えることができないから間違う
0797132人目の素数さん
垢版 |
2024/03/16(土) 17:29:45.75ID:vIT5CHVL
>>778
>>>∃yで放り出すと、yは自然数に限らないから 二つの式の意味は違う
>>∀x∈ℕ.∃y∈ℕ.x<y でないと考えねばならない理由があるか?全くない
>>したがって却下
>そうかな?
そうだな
(完)

ところで、サルは全順序と整列順序の違いが
未だに分かってないと思われる

整列順序は全順序だが、
全順序なら整列順序、とはいえない

整列順序は以下が成立する順序
∀x.((∃y.x<y)⇒(∃z.x<z∧∀w.x<w⇒z<=w))

日本語で書けば
任意のxについて、
xより大きい元yが存在するなら
その中でもっとも小さい元zが存在する

これxの後者という
0798132人目の素数さん
垢版 |
2024/03/16(土) 17:35:52.89ID:vIT5CHVL
さて有限整列順序の場合、最大元が存在する すなわち
∃x∀y.x>=y
しかし、無限整列順序の場合、最大元が存在しない場合がある すなわち
∀x.(∃z.x<z∧∀w.x<w⇒z<=w)
0799132人目の素数さん
垢版 |
2024/03/16(土) 19:03:19.17ID:+LjGwmYz
>>792
>>・「ドアの後ろに賞品がある」場合を、キチンと数え上げれば良い!
>1回の出題では1通りしかないから数え上げようが無い
>君が言ってるのは出題が試行の場合 君は無意識に出題を試行としている
>だが出題を試行としてしまうと1回の出題における確率は定義されない

・”1回の出題では1通りしかないから数え上げようが無い”?
 か、なんか笑えるな www
・下記の”【中学数学】さいころ2個の確率問題をパターン別に解説!”を
 ”百回”繰返し見てね
・全部 試行は 1回前提で、確率計算するよw
 まあ、君には理解が難しいんだな? なんか笑えるな www

やれやれだな ;p)

https://study-line.com/kakuritsu-saikoro/
数スタ
【中学数学】さいころ2個の確率問題をパターン別に解説!
0800132人目の素数さん
垢版 |
2024/03/16(土) 20:01:11.64ID:/v13gW+O
>>796
>∀が先頭についてるんだから相手側に全公開してるだろ
0801132人目の素数さん
垢版 |
2024/03/16(土) 20:07:05.86ID:/v13gW+O
>>799
”1回の出題”の意味を誤解したんだね?
未来に定まる予定の出題ではなく過去に定まった出題のことだよ
文脈で読み取ろうよ 小学生じゃないんだから
0802132人目の素数さん
垢版 |
2024/03/16(土) 20:08:59.89ID:+LjGwmYz
>>797-798
整列順序ねw

・院試で、あたま良すぎて落ちるやつ
 自分で定義を作るやつ
・整列順序についてなら、決まった定義があるのだから
 それをきっちり覚えておかないとね(下記)
 ”S 上の全順序関係 "≤" であって、S の空でない任意の部分集合が必ず ≤ に関する最小元をもつものをいう”

おっと、あたま悪すぎで
自分で定義を作ったのかな?w

(参考)
https://ja.wikipedia.org/wiki/%E6%95%B4%E5%88%97%E9%9B%86%E5%90%88
整列集合
整列集合(せいれつしゅうごう、英: well­ordered set)、または整列順序付けられた集合(せいれつじゅんじょづけられたしゅうごう)とは、数学における概念の1つで、整列順序を備えた集合のことをいう。ここで、集合 S 上の整列順序関係 (well­order) とは、S 上の全順序関係 "≤" であって、S の空でない任意の部分集合が必ず ≤ に関する最小元をもつものをいう。あるいは同じことだが、整列順序とは整礎な全順序関係のことである。整列集合 (S, ≤) を慣例に従ってしばしば単純に S で表す。

導入
整列集合 X の任意の元 s は、それが X の最大元でない限り、ただ一つの後者(successor; 後継、次の元、直後の元)を持つ。これはつまり、s よりも大きな X の元全体の成す部分集合における最小元として s の後者が決まるということである。また、整列集合 X の中で上に有界な任意の部分集合は(その上界全体の成す X の部分集合に最小元がとれるから)必ず上限を持つ。あるいは整列集合 X には、前者(predecessor; 直前の元)を持たない元が必ず存在する(それはもちろん、X 全体における最小元である)。

集合に整列順序が与えられれば、そこでは集合の全ての元に対する命題の超限帰納法を用いた証明を考えることができる。

自然数全体の成す集合 N が通常の大小関係 "<" に関して整列集合となるという事実は、一般に整列原理と呼ばれる。

(選択公理に同値な)整列可能定理は、任意の集合が整列順序付け可能であることを主張するものである。整列可能定理はまたツォルンの補題とも同値である。

https://www.math.is.tohoku.ac.jp/~obata/student/subject/file/2018-13_WellOrdered.pdf
GAIRON-book : 2018/6/21(19:23)
第13章 整列集合 東北大 尾畑研
0803132人目の素数さん
垢版 |
2024/03/16(土) 20:15:49.64ID:/v13gW+O
>>799
>・全部 試行は 1回前提で、確率計算するよw
「サイコロを一回投げる試行」は1回の試行ではない。
試行毎に結果は変化し、その集合が標本空間。そのひとつの元が1回の試行の結果。

君、根本的に分かってないようだね
0804132人目の素数さん
垢版 |
2024/03/16(土) 20:21:58.49ID:+LjGwmYz
>>801
>”1回の出題”の意味を誤解したんだね?
>未来に定まる予定の出題ではなく過去に定まった出題のことだよ
>文脈で読み取ろうよ 小学生じゃないんだから

中学 確率
・サイコロ二つを振って、箱の中
 目は決まっている
・二つの和が12になる確率は?
 二つとも6の場合で、1/36

さて、まだサイコロは振っていない
・サイコロ二つを振るが
 目はまだ決まっていない
・二つの和が12になる確率は?
 二つとも6の場合で、1/36

幼稚園レベルだな
0805132人目の素数さん
垢版 |
2024/03/16(土) 20:22:24.29ID:/v13gW+O
>>799
ひとつのサイコロを一回投げる試行の結果は1〜6の6通り
1回の試行の結果はそのいずれかひとつの1通り

小学生かい?君は
0806132人目の素数さん
垢版 |
2024/03/16(土) 20:26:56.64ID:/v13gW+O
>>804
>中学 確率
>・サイコロ二つを振って、箱の中
> 目は決まっている
>・二つの和が12になる確率は?
> 二つとも6の場合で、1/36
それは箱の中の目を決めることを試行とした場合の確率ね

君、試行が全然分かってないね 定義を読み直してごらん
0808132人目の素数さん
垢版 |
2024/03/16(土) 20:34:01.42ID:/v13gW+O
>>804
なんかおかしいと思って
https://study-line.com/kakuritsu-saikoro/
を見てみたが、

>中学 確率
>・サイコロ二つを振って、箱の中
> 目は決まっている
>・二つの和が12になる確率は?
> 二つとも6の場合で、1/36

なんて書かれてないじゃんw
捏造すんなよサル
0809132人目の素数さん
垢版 |
2024/03/16(土) 20:34:53.30ID:/v13gW+O
>>807
それが君の感想
0810132人目の素数さん
垢版 |
2024/03/16(土) 20:39:32.28ID:/v13gW+O
サルはサイコパスか?
平気で捏造するのはサイコパスだからなんだろうな

違うというならどこに
>・サイコロ二つを振って、箱の中
> 目は決まっている
>・二つの和が12になる確率は?
> 二つとも6の場合で、1/36
と書かれてるか示してごらん
0811132人目の素数さん
垢版 |
2024/03/16(土) 20:43:19.00ID:/v13gW+O
馬鹿は仕方ないが捏造は人間として終わってるだろ

まあサルは最初から人間じゃないがw
0812132人目の素数さん
垢版 |
2024/03/16(土) 20:45:08.05ID:+LjGwmYz
>>783 補足
>時枝の二つの決定番号 da,db ∈N(可算無限集合)で
>確率P(da>db)=1/2 と錯覚するが如し
>(∵ N(可算無限集合)だから、∞/∞ で不定形で、1/2はいえない)
>けれども、人はついつい 確率P(da>db)=1/2 と錯覚するのです

二つ例を挙げよう
・ケースA
 n∈N(可算無限集合) で、nは奇数か偶数だ
 奇数か偶数かは、半々だが
 確率P(nは奇数)=1/2 ?
 しかし、Ω=Nだと 全体が発散しているので、確率計算1/2は正当化できない
・ケースB
 冒頭の場合だが、補足しよう
 Ω={1,2,・・n}の有限集合で、一様分布とする
 この場合、nが十分大きいと、上記 確率P(da>db)=1/2は正しい
 補足すると
 daを横軸(x軸)、dbを縦軸(y軸)にとると、
 一辺nの正方形の格子が描ける
 da=db は 対角線(直線y=x)上にある
 da>db の領域は、対角線 より下側の直角三角形を成す
 だから、全体の正方形に対し 下側の直角三角形の部分は1/2だ
 ところが、これは有限の場合だ
 Ω=Nで 全体が発散していると 有限の場合の全体の正方形が無限大に発散し
 下側の直角三角形の部分も無限大に発散する
 よって、∞/∞ で不定形で、1/2はいえない
0813132人目の素数さん
垢版 |
2024/03/16(土) 20:50:36.40ID:/v13gW+O
>>812
>確率P(da>db)=1/2 と錯覚
それ君だけね
時枝先生はそんな阿呆な錯覚していない
0814132人目の素数さん
垢版 |
2024/03/16(土) 20:55:28.45ID:+LjGwmYz
>>808 >>810
>https://study-line.com/kakuritsu-saikoro/
>を見てみたが、
>>・サイコロ二つを振って、箱の中
>> 目は決まっている
>>・二つの和が12になる確率は?
>> 二つとも6の場合で、1/36
>と書かれてるか示してごらん

本気で聞いているのかな?w
上記のサイト中で

冒頭に
”今回の内容をサクッと理解したい方はこちらの動画がおススメです”
とあって、動画のリンク貼ってあるよ。そこにあるよ

本気で聞いていたのか!w
やれやれ ;p)
0815132人目の素数さん
垢版 |
2024/03/16(土) 21:01:11.07ID:/v13gW+O
>>814
動画の何分何秒から?
0816132人目の素数さん
垢版 |
2024/03/16(土) 21:04:19.38ID:/v13gW+O
>>814
>そこにあるよ
嘘でないなら答えられるよな? 何分何秒から?
0817132人目の素数さん
垢版 |
2024/03/16(土) 21:05:05.53ID:/v13gW+O
嘘に嘘を重ねて破綻する
完全にサイコパスだわこいつ
0818132人目の素数さん
垢版 |
2024/03/16(土) 21:16:45.98ID:/v13gW+O
>>814
おかしいなあ、俺が見た限り
「〇〇のサイコロを投げる。〇〇になる確率を求めなさい。」
という出題パターンしか無いんだが

どこにも
>・サイコロ二つを振って、箱の中
> 目は決まっている
なんて無いんだが

もしかして君の捏造? いったいどういう了見なんだ? 何のために捏造するんだ?
数学どうこう以前の人間性がぶっ壊れてるよ君
0819132人目の素数さん
垢版 |
2024/03/16(土) 21:28:21.91ID:/v13gW+O
他人にマウント取るためなら捏造なんて屁とも思わない
動画を特定したらすぐバレることは火を見るより明らかなのにそれでもやらずにいられない

狂ってる 完全に狂ってる ここまでの異常人格者見たこと無い
0820132人目の素数さん
垢版 |
2024/03/16(土) 21:38:04.47ID:/v13gW+O
サイコパスの特徴
・窃盗などの違法行為を繰り返すことがある
・自信にあふれ、魅力的に見える
・他人に対して共感する気持ちが持てない
・人を支配する
・平然とうそをつく
・衝動的に行動する
・他人を責める
・無責任である

魅力的に見える以外全部当てはまってて草
0821132人目の素数さん
垢版 |
2024/03/16(土) 21:42:01.00ID:REZvRVWI
>>809
開けてない箱の∀を後ろに持っていけないのが非公開の情報を使ってるなによりの証拠じゃん
0822132人目の素数さん
垢版 |
2024/03/16(土) 21:59:09.03ID:/v13gW+O
>>821
非公開の情報とは?
0824132人目の素数さん
垢版 |
2024/03/16(土) 22:04:02.72ID:/v13gW+O
>>823
証拠になる理由は?
0825132人目の素数さん
垢版 |
2024/03/16(土) 22:05:26.27ID:/v13gW+O
言っておくが、君の好きな問題において証拠になるは理由にならないぞ?
問題が違えば証拠になるならないも違ってくる
0827132人目の素数さん
垢版 |
2024/03/16(土) 22:13:42.24ID:/v13gW+O
>>824
それ待遇を言ってるに過ぎないじゃんw
君の持論の正しさを示せと言ってるのに待遇を持ち出すって、君、もしかして馬鹿?
0828132人目の素数さん
垢版 |
2024/03/16(土) 22:14:22.55ID:/v13gW+O
アンカミス >>826
0829132人目の素数さん
垢版 |
2024/03/16(土) 22:17:14.66ID:/v13gW+O
命題Aが真であることとその対偶が真であることは同値
よって対偶を持ち出してもAが真である理由には1_もならない
高校で習わんかった?
0830132人目の素数さん
垢版 |
2024/03/16(土) 22:26:15.94ID:/v13gW+O
正しい理由を聞かれてつい対偶を持ち出してしまったってことは、正しいと妄想してるだけってことさ
違うと言うなら箱入り無数目においても正しいことを示してみて
0832132人目の素数さん
垢版 |
2024/03/16(土) 22:46:13.79ID:/v13gW+O
>>831
じゃ証明して
0834132人目の素数さん
垢版 |
2024/03/16(土) 22:55:50.48ID:/v13gW+O
>>833
はい、また逃げました〜
その逃亡芸もう秋田
0835132人目の素数さん
垢版 |
2024/03/16(土) 22:58:16.85ID:/v13gW+O
できもしないことを簡単にできるとか言っちゃう君もサイコパスザルと同類やね
不成立派はこんなんばっかやなw
0836132人目の素数さん
垢版 |
2024/03/16(土) 22:59:48.78ID:/v13gW+O
動画に出てる←嘘
簡単にできる←嘘

サイコパスしかおらんのかい、不成立派はw
0837132人目の素数さん
垢版 |
2024/03/16(土) 23:02:13.92ID:REZvRVWI
>>834
やっぱり∃(x+1).と同一人物だったのか
こんな頭のおかしい論理式書くやつの相手なんて誰がするかよ
0838132人目の素数さん
垢版 |
2024/03/16(土) 23:03:57.43ID:+LjGwmYz
>>818
サイコパスのおサル>>8
詭弁のデパートだな
次のスレでテンプレに入れておくぜ!w

>>814より再録)
>>808 >>810
>https://study-line.com/kakuritsu-saikoro/
>を見てみたが、
>>・サイコロ二つを振って、箱の中
>> 目は決まっている
>>・二つの和が12になる確率は?
>> 二つとも6の場合で、1/36
>と書かれてるか示してごらん

本気で聞いているのかな?w
上記のサイト中で
冒頭に
”今回の内容をサクッと理解したい方はこちらの動画がおススメです”
とあって、動画のリンク貼ってあるよ。そこにあるよ
(引用終り)

>>818より再録)
>>814
おかしいなあ、俺が見た限り
「〇〇のサイコロを投げる。〇〇になる確率を求めなさい。」
という出題パターンしか無いんだが
どこにも
>・サイコロ二つを振って、箱の中
> 目は決まっている
なんて無いんだが
(引用終り)

1)サイコロ二つを振って 二つの和が12になる確率は? 二つとも6の場合で、1/36
 これが分からないと聞いてきた
2)動画にあると示したら、「サイコロ二つを振って、箱の中 目は決まっている なんて無いんだが」
 ときたもんだ。笑える
0839132人目の素数さん
垢版 |
2024/03/16(土) 23:06:45.95ID:+LjGwmYz
中学レベルの確率論でつまずいているんだ
アホのきわみだね
0840132人目の素数さん
垢版 |
2024/03/16(土) 23:07:39.62ID:/v13gW+O
>>837
言い訳になってないよw
0841132人目の素数さん
垢版 |
2024/03/16(土) 23:12:43.81ID:REZvRVWI
∀x.∃y.P(x,y)の証明が、λx.ex t f(x,t)の形をしてて、xがtに自由に現れないとき、これこれが証明可能であるなんて、∃(x
÷1)野郎に分かるわけないじゃん
0842132人目の素数さん
垢版 |
2024/03/16(土) 23:18:18.29ID:/v13gW+O
>>838
>1)サイコロ二つを振って 二つの和が12になる確率は? 二つとも6の場合で、1/36
どこにも
>・サイコロ二つを振って、箱の中
> 目は決まっている
なんて書かれてないじゃんw

>2)動画にあると示したら、「サイコロ二つを振って、箱の中 目は決まっている なんて無いんだが」
> ときたもんだ。笑える
いや、動画のどこにも
>・サイコロ二つを振って、箱の中
> 目は決まっている
なんて無いよね
君、嘘ついてるよね
笑える? 君、病院行った方がいいよ
0843132人目の素数さん
垢版 |
2024/03/16(土) 23:23:34.82ID:/v13gW+O
>>841
>これこれが証明可能である
ってなに?w

>∃(x÷1)野郎に分かるわけないじゃん
じゃあ数学者に分かる証明を書いて
知り合いの数学者に見てもらうから
0844132人目の素数さん
垢版 |
2024/03/16(土) 23:32:51.63ID:/v13gW+O
>>838
君さあ
これからサイコロを振ることと、サイコロを振った結果は違うことは理解できる?
前者の標本空間={1,2,3,4,5,6}
後者の標本空間={出た目} ←1元しかないので確率を考えてもナンセンス
0845132人目の素数さん
垢版 |
2024/03/16(土) 23:33:50.33ID:REZvRVWI
>>843
∃y.∀x.P(x,y)に決まってるだろ
お前はどうせわかんないんだろ、伝言ゲームするぐらいのらそいつここによんでくれぱいいじゃん
0846132人目の素数さん
垢版 |
2024/03/16(土) 23:34:44.04ID:/v13gW+O
動画に出てる←嘘
簡単にできる←嘘

サイコパスしかおらんのかい、不成立派はw
0847132人目の素数さん
垢版 |
2024/03/16(土) 23:39:17.12ID:/v13gW+O
>>845
書かれてないのに呼ぶ意味は?
君、乱心してんの?落ち着けよ
0848132人目の素数さん
垢版 |
2024/03/16(土) 23:42:00.72ID:/v13gW+O
>>845
簡単なんでしょ?
じゃさっさと書いてよ
書かれてもないのに呼ばないよw なんで呼んだ?ってなっちゃうだろーがw
0850132人目の素数さん
垢版 |
2024/03/16(土) 23:45:16.11ID:/v13gW+O
>>849
だーかーらー
呼ばれた人はどうすんだよw なんで呼ばれたんだ?ってなるだろ? 分からんのか? 阿呆なの?馬鹿なの?
0851132人目の素数さん
垢版 |
2024/03/16(土) 23:46:57.63ID:/v13gW+O
てかつべこべ言わずさっさと書けや
またいつものハッタリか?
0852132人目の素数さん
垢版 |
2024/03/16(土) 23:48:56.82ID:REZvRVWI
∀x.∃y.P(x,y)の証明が、λx.ex t f(x,t)の形をしてて、xがtに自由に現れないとき、ex t (λx.f(x,t))は∃y.∀x.P(x,t)の証明である。終わり
0853132人目の素数さん
垢版 |
2024/03/16(土) 23:49:46.07ID:/v13gW+O
君さ、会社でもそんななん?
レビューアを呼び出してから資料書くん?
資料書くのが先じゃないん?
だいじょうぶか?君
0854132人目の素数さん
垢版 |
2024/03/16(土) 23:50:39.70ID:REZvRVWI
で、これは結局放置するわけ?

x+1が仮に変数だとして
>∀x∈ℕ.∃(x+1).x<x+1
これをα変換したら
∀x∈ℕ.∃y.x<x+1
なのか
∀x∈ℕ.∃y.x<y
なのかすら不明瞭
0857132人目の素数さん
垢版 |
2024/03/16(土) 23:55:14.40ID:/v13gW+O
>>852
箱入り無数目におけるx、y、P(x,y)がそれぞれ何か述べよ
0858132人目の素数さん
垢版 |
2024/03/16(土) 23:58:01.45ID:/v13gW+O
>>852
∀x.∃y.P(x,y)なる命題が箱入り無数目のそれと一致してなければ>>852は完全にナンセンスなので心して答えるように
0860132人目の素数さん
垢版 |
2024/03/17(日) 00:16:23.47ID:VAa6dkvQ
>>859
違うけど、
x=先手の手、y=後手の手、P(x,y)=後手の勝率は99/100以上
という理解で合ってる?

まずはここまで
0861132人目の素数さん
垢版 |
2024/03/17(日) 00:27:17.53ID:egixwGA8
>>860
じゃあこっちは上のアンカーみたいな定式化に対して∀∃の形にすんなって言ってるんだから、関係ないなら来ないでくれる
0862132人目の素数さん
垢版 |
2024/03/17(日) 00:40:01.46ID:VAa6dkvQ
>>859
>∀x.∃y.P(x,y)の証明が、λx.ex t f(x,t)の形をしてて、xがtに自由に現れないとき、ex t (λx.f(x,t))は∃y.∀x.P(x,t)の証明である。

ex t (λx.f(x,t)) が ∃y.∀x.P(x,t) の証明であるとなぜ君の主張「∀が先頭についてるんだから相手側に全公開してるだろ」が正当化されるの?
0864132人目の素数さん
垢版 |
2024/03/17(日) 00:53:55.64ID:egixwGA8
そもそも、ゲームの攻略法を∀と∃の組み合わせで定式化したら前についてる量化子はすべて公開されてる状態で次の量化子の手番に進むに決まってんじゃん
これになんの疑問の余地があるのかさっぱりわからんのだが
0865132人目の素数さん
垢版 |
2024/03/17(日) 00:54:26.72ID:VAa6dkvQ
>>863
>∀x.∃y.P(x,y)の証明が、λx.ex t f(x,t)の形をしてて、xがtに自由に現れないとき、ex t (λx.f(x,t))は∃y.∀x.P(x,t)の証明である。

>∀が先頭についてるんだから相手側に全公開してるだろ
の証明になってないって言ってるんだけど

的確に反論しないとw
0866132人目の素数さん
垢版 |
2024/03/17(日) 00:55:56.21ID:VAa6dkvQ
>>864
>決まってんじゃん
はい、決まってないです
あなたの妄想です
0868132人目の素数さん
垢版 |
2024/03/17(日) 00:57:24.92ID:egixwGA8
>>866
そう思うなら無視すりゃいいじゃん
こっちは∃(x+1)とか書き始める人間に何言われても気にしないから
0869132人目の素数さん
垢版 |
2024/03/17(日) 00:58:15.47ID:VAa6dkvQ
>>867
じゃ何の証明だよw
こっちは最初から>>260が間違いであることは理解したの?って言ってるんだけどw
それに対する反論じゃなかったんか? 馬鹿かよw
0870132人目の素数さん
垢版 |
2024/03/17(日) 00:59:52.45ID:VAa6dkvQ
>>867
おまえ頭オカシイのか?
さっさと>>260の正当性を示せよw
0871132人目の素数さん
垢版 |
2024/03/17(日) 01:02:08.61ID:VAa6dkvQ
>>868
おまえ都合悪くなるといつも逃げるのな
0872132人目の素数さん
垢版 |
2024/03/17(日) 01:02:45.83ID:egixwGA8
>>869
∀x.∃y.P(x,y)の証明が、λx.ex t f(x,t)の形をしてて、xがtに自由に現れないとき、∃y.∀x.P(x,t)が証明できることの証明だろ
お前が>>832でこれを証明しろっていったんじゃねーかよ
記憶力ダチョウ並かよ
0873132人目の素数さん
垢版 |
2024/03/17(日) 01:05:59.86ID:egixwGA8
>>870
ゲームの定式化をそうやるって話だよ
これが嫌いなら勝手にすればいいじゃん
定式化が変われば結果が変わるのは当たり前なんだからさ
0874132人目の素数さん
垢版 |
2024/03/17(日) 01:08:42.91ID:egixwGA8
そもそも箱の中身を確率変数にして定式化すれば、∀の順序とか悩まずに定式化できるのに、こんな入口で躓いててこの先どーすんの
0875132人目の素数さん
垢版 |
2024/03/17(日) 01:09:04.96ID:VAa6dkvQ
>>872
>∀x.∃y.P(x,y)の証明が、λx.ex t f(x,t)の形をしてて、xがtに自由に現れないとき、∃y.∀x.P(x,t)が証明できることの証明だろ
え???
λx.ex t f(x,t) ⇒ ex t (λx.f(x,t)) の証明は?
それ無いと∃y.∀x.P(x,t)が証明できることの証明になってないぞ
0876132人目の素数さん
垢版 |
2024/03/17(日) 01:10:22.07ID:VAa6dkvQ
>>874
そもそも箱の中身を確率変数にできないのにこんな入口で躓いててこの先どーすんの
0878132人目の素数さん
垢版 |
2024/03/17(日) 01:16:14.72ID:VAa6dkvQ
>>877
そうだよ
P⇒Q は Pが真ならQが真という命題
0879132人目の素数さん
垢版 |
2024/03/17(日) 01:16:22.80ID:egixwGA8
>>876
できないってなんだよ
定式化なんだから好きにやりゃいいんだよ
そもそも記事でやってる説明だって確率変数で書いたときの特別な場合だろ
0880132人目の素数さん
垢版 |
2024/03/17(日) 01:17:37.88ID:egixwGA8
>>878
なんでそこにならばをつけたの?
お前はπ⇒eとかみたいな感じでならばを使うんか?
0881132人目の素数さん
垢版 |
2024/03/17(日) 01:22:54.32ID:VAa6dkvQ
>>880
「∀x.∃y.P(x,y)の証明が、λx.ex t f(x,t)の形をしてて、xがtに自由に現れない」から「ex t (λx.f(x,t))は∃y.∀x.P(x,t)の証明である」へギャップがある
ギャップを埋めよ
0882132人目の素数さん
垢版 |
2024/03/17(日) 01:23:20.56ID:egixwGA8
1ならば2とか
sin 60°ならばtan 30°とか
お前はそういう風にならばを使うの?
0885132人目の素数さん
垢版 |
2024/03/17(日) 01:30:15.41ID:VAa6dkvQ
>>879
>定式化なんだから好きにやりゃいいんだよ
大間違い
問題設定に沿ってなければダメ

ある一つの固定された出題に対し回答者の勝率が定義できるためには出題が試行であってはダメ
箱の中身を確率変数にするということは出題が試行ということ
なぜなら箱の中身が変化するのは出題毎だから

>そもそも記事でやってる説明だって確率変数で書いたときの特別な場合だろ
意味不明
0886132人目の素数さん
垢版 |
2024/03/17(日) 01:31:06.42ID:VAa6dkvQ
>>884
ねえ、ギャップは?
いつ埋まるの?
0887132人目の素数さん
垢版 |
2024/03/17(日) 01:32:59.45ID:egixwGA8
>>885
じゃあ君がコロナに感染してるかどうかも確率変数にしちゃだめだね
医者どもはそれを確率変数だと思って診断してるから、お前はもうPCR検査行くなよ
0888132人目の素数さん
垢版 |
2024/03/17(日) 01:33:33.37ID:VAa6dkvQ
>>882
おまえ馬鹿? そういう風にも使えるよ
命題「x=π ⇒ x=e」は偽
はい、使った
0891132人目の素数さん
垢版 |
2024/03/17(日) 01:37:08.43ID:VAa6dkvQ
>>887
意味不明
確率空間を書いてみて
0892132人目の素数さん
垢版 |
2024/03/17(日) 01:38:04.38ID:VAa6dkvQ
>>890
いいから早くギャップを埋めてよ
また逃げる気?
0895132人目の素数さん
垢版 |
2024/03/17(日) 01:39:57.90ID:VAa6dkvQ
こいつまた逃げる気か
都合が悪くなるといつも>>889 >>890みたいにごまかして逃げるな
0896132人目の素数さん
垢版 |
2024/03/17(日) 01:40:31.93ID:VAa6dkvQ
>>894
ごまかすな
ギャップを示せ
0897132人目の素数さん
垢版 |
2024/03/17(日) 01:40:35.03ID:egixwGA8
>>891
なんの確率空間を書くの?
前から何度も言ってるように、普通は確率空間は任意だよ
0898132人目の素数さん
垢版 |
2024/03/17(日) 01:41:53.07ID:egixwGA8
>>896
ならばのところにギャップがあるんでしょ?
だから1⇒2ってなんなんだよって聞いてんだよ
0899132人目の素数さん
垢版 |
2024/03/17(日) 01:43:11.58ID:VAa6dkvQ
>>898
>>881読めないの?なら小学校の国語からやり直し
0900132人目の素数さん
垢版 |
2024/03/17(日) 01:43:41.06ID:VAa6dkvQ
>>897
>>887
0902132人目の素数さん
垢版 |
2024/03/17(日) 01:46:12.09ID:egixwGA8
>>899
どこにギャップがあるの?
⇒はどこに行ったの?
まず⇒の話を解決しないと意味不明なんですけど
0903132人目の素数さん
垢版 |
2024/03/17(日) 01:47:08.12ID:VAa6dkvQ
>>897
>前から何度も言ってるように、普通は確率空間は任意だよ
事象空間が任意てw じゃ何の確率だよw 完全に狂ってるw
0904132人目の素数さん
垢版 |
2024/03/17(日) 01:48:24.79ID:VAa6dkvQ
>>902
>どこにギャップがあるの?
「∀x.∃y.P(x,y)の証明が、λx.ex t f(x,t)の形をしてて、xがtに自由に現れない」から「ex t (λx.f(x,t))は∃y.∀x.P(x,t)の証明である」へギャップがある
って日本語が読めないようだね。小学校の国語からやり直し
0907132人目の素数さん
垢版 |
2024/03/17(日) 01:51:55.31ID:VAa6dkvQ
>>897
>前から何度も言ってるように、普通は確率空間は任意だよ
サイコロひとつを一回振る試行の標本空間は任意なので{}としました
この標本空間{}を用いて1の目がでる確率を計算してください
0909132人目の素数さん
垢版 |
2024/03/17(日) 01:53:10.53ID:VAa6dkvQ
>>905
>>904が具体的なギャップの指摘
ギャップを埋めるのは君
0910132人目の素数さん
垢版 |
2024/03/17(日) 01:53:50.14ID:VAa6dkvQ
>>906
屁理屈はよいので>>907に答えて
0913132人目の素数さん
垢版 |
2024/03/17(日) 01:55:41.74ID:VAa6dkvQ
>>908
あれ?任意でいいって言ってなかったっけ?
任意でいいってことは俺が決めてもいいんだろ?ダメなの?じゃ誰が決めんの?それ任意って言うの?馬鹿?
0914132人目の素数さん
垢版 |
2024/03/17(日) 01:58:53.06ID:egixwGA8
>>913
任意の確率空間(Ω,F,P)について何々から書き始めるから任意なんだよ、お前がひとつに決めるなよ
0915132人目の素数さん
垢版 |
2024/03/17(日) 02:05:48.90ID:VAa6dkvQ
>>911
>>852って単に
「先手の情報が全公開されてないなら ∀x.∃y.P(x,y) を ∃y.∀x.P(x,t) に書き換えられる」
と言ってるに過ぎず、それって君の持論を言い直しただけじゃんw
何の証明にもなってないw 馬鹿過ぎw
0916132人目の素数さん
垢版 |
2024/03/17(日) 02:07:55.95ID:VAa6dkvQ
>>914
>任意の確率空間(Ω,F,P)について何々から書き始めるから任意なんだよ
日本語でお願いします

>お前がひとつに決めるなよ
じゃあおまえがひとつに決めろよ
0917132人目の素数さん
垢版 |
2024/03/17(日) 02:09:56.03ID:VAa6dkvQ
>>911
なぜ書き換えられるかの理由が無いと証明にならんぞw
てかおまえ証明って何か分かってる?w
0919132人目の素数さん
垢版 |
2024/03/17(日) 02:12:00.92ID:VAa6dkvQ
>>918
不便でいいから決めろ
0920132人目の素数さん
垢版 |
2024/03/17(日) 02:12:31.38ID:egixwGA8
>>915
全然違う

>「先手の情報が全公開されてないなら ∀x.∃y.P(x,y) を ∃y.∀x.P(x,t) に書き換えられる」
勝手に捏造しないでくれます?
0922132人目の素数さん
垢版 |
2024/03/17(日) 02:13:11.84ID:VAa6dkvQ
>>918
おまえいっつも逃げるのな
逃げ口上考える頭を数学に使えば?
0923132人目の素数さん
垢版 |
2024/03/17(日) 02:13:49.01ID:VAa6dkvQ
>>920
何がどう違うと?
0924132人目の素数さん
垢版 |
2024/03/17(日) 02:14:15.95ID:VAa6dkvQ
>>921
だから逃げ口上はいいって
0925132人目の素数さん
垢版 |
2024/03/17(日) 02:15:00.99ID:egixwGA8
>>922
不便なものに不便だと言って何が悪い
確率論の標準的な手法に文句があるならコルモゴロフにでも言ってくれ
0927132人目の素数さん
垢版 |
2024/03/17(日) 02:16:11.43ID:VAa6dkvQ
>>921
ひとつに決めないのも定式化の一部ならひとつに決めるのも定式化の一部だろ
じゃ決めろよ
これ以上逃げ口上は勘弁な
0928132人目の素数さん
垢版 |
2024/03/17(日) 02:17:35.09ID:VAa6dkvQ
>>925
悪いなんて言ってないじゃんw 幻聴が聞こえるのか?
0929132人目の素数さん
垢版 |
2024/03/17(日) 02:18:02.57ID:VAa6dkvQ
>>926
何をどう捏造してると?
0930132人目の素数さん
垢版 |
2024/03/17(日) 02:18:16.92ID:egixwGA8
>>927
何言ってんの
定式化は各人好きなようにやっていいよ
君がやりたいならそうしろよ
こっちは確率論を具体的に書かない標準的なやり方でやるから
0931132人目の素数さん
垢版 |
2024/03/17(日) 02:19:43.84ID:VAa6dkvQ
>>930
おまえさっき勝手に決めるなって言ったのもう忘れたの?w
ダメだこいつw 完全にイカレてやがるw
0932132人目の素数さん
垢版 |
2024/03/17(日) 02:20:11.32ID:egixwGA8
>>929
>「先手の情報が全公開されてないなら ∀x.∃y.P(x,y) を ∃y.∀x.P(x,t) に書き換えられる」
このような主張はしてない
0934132人目の素数さん
垢版 |
2024/03/17(日) 02:22:58.10ID:VAa6dkvQ
0908132人目の素数さん
2024/03/17(日) 01:52:50.48ID:egixwGA8
>>907
お前が決めるんじゃねーよ

0930132人目の素数さん
2024/03/17(日) 02:18:16.92ID:egixwGA8
>>927
定式化は各人好きなようにやっていいよ
君がやりたいならそうしろよ


錯乱してるw
0935132人目の素数さん
垢版 |
2024/03/17(日) 02:23:33.91ID:VAa6dkvQ
>>933
0908132人目の素数さん
2024/03/17(日) 01:52:50.48ID:egixwGA8
>>907
お前が決めるんじゃねーよ
0936132人目の素数さん
垢版 |
2024/03/17(日) 02:24:43.95ID:VAa6dkvQ
>>932
じゃあどう主張してんの?
0937132人目の素数さん
垢版 |
2024/03/17(日) 02:28:27.68ID:VAa6dkvQ
>>932
>∀x.∃y.P(x,y)の証明が、λx.ex t f(x,t)の形をしてて、xがtに自由に現れないとき、ex t (λx.f(x,t))は∃y.∀x.P(x,t)の証明である
証明である理由が何も書かれてなくて勝手に宣言してるだけじゃんw
宣言したもん勝ちかよw
0938132人目の素数さん
垢版 |
2024/03/17(日) 02:28:43.68ID:HNHCaIr5
>>935
こっちの定式化では任意にしてるところをお前が決めんなって言ってんの
お前が定式化してるところは固定で好きなようにやっていいから
0939132人目の素数さん
垢版 |
2024/03/17(日) 02:29:49.82ID:HNHCaIr5
>>937
何?証明であるのところが解らなかったの?
なんで今まで具体的に書かなかったの?
0940132人目の素数さん
垢版 |
2024/03/17(日) 02:31:41.64ID:VAa6dkvQ
>>938
なんでおまえが持ち出したPCR検査の確率空間をそんなに書きたくないの?
書けないなら持ち出さなきゃいいじゃん
持ち出したからには書けよ
不便とか言い訳してんなよカス
0941132人目の素数さん
垢版 |
2024/03/17(日) 02:33:16.72ID:VAa6dkvQ
>>939
うん分からなかった
証明になってないから分かり様が無い
0943132人目の素数さん
垢版 |
2024/03/17(日) 02:34:26.62ID:VAa6dkvQ
>>939
もう一回聞くけど
ex t (λx.f(x,t)) が ∃y.∀x.P(x,t) の証明である理由は何?
0946132人目の素数さん
垢版 |
2024/03/17(日) 02:35:52.22ID:VAa6dkvQ
>>942
任意ってことは{}でもいいんだろ?
じゃあ>>907に答えろや 何逃げてんだよ
0947132人目の素数さん
垢版 |
2024/03/17(日) 02:36:44.79ID:VAa6dkvQ
>>945
証明の定義を書いてみて
0948132人目の素数さん
垢版 |
2024/03/17(日) 02:38:38.65ID:VAa6dkvQ
こいつ書けって言うと全部逃げるのなw
口から出まかせだから書けないんだろう そりゃ逃げるしかないわな
0949132人目の素数さん
垢版 |
2024/03/17(日) 02:40:40.00ID:VAa6dkvQ
>>939
もうひとつ聞くけど
xがtに自由に現れるとき ex t (λx.f(x,t)) が ∃y.∀x.P(x,t) の証明でない理由は何?
0950132人目の素数さん
垢版 |
2024/03/17(日) 02:41:38.99ID:HNHCaIr5
>>947
それ知らないのになんでギャップがあるって主張すんの?
お前が知ってる定義を書いてみろよ
0951132人目の素数さん
垢版 |
2024/03/17(日) 02:42:44.70ID:VAa6dkvQ
>>950
自分が書けないからって何言いだすんだおまえw
0952132人目の素数さん
垢版 |
2024/03/17(日) 02:43:37.40ID:VAa6dkvQ
証明の定義の言い出しっぺはおまえだろ?
言い出しっぺが書けやクズ
0953132人目の素数さん
垢版 |
2024/03/17(日) 02:47:50.71ID:VAa6dkvQ
>>943>>949は宿題な
忘れずやれよ
0956132人目の素数さん
垢版 |
2024/03/17(日) 02:54:07.97ID:VAa6dkvQ
>>954
結局理由をひとつも言えないのね?
だけど証明になってると思ってるのね?
ダメだこりゃ
0958132人目の素数さん
垢版 |
2024/03/17(日) 02:58:01.19ID:VAa6dkvQ
聞いていいか?
箱入り無数目は∃y.∀x.P(x,t)とは書けない
一方、xを全公開してなくても∀x.∃y.P(x,y)であることが証明されている
それはおまえの持論に反してるんだろ?
てことは上記証明が間違ってるはずなんだろ?
どこが間違ってるの?
0959132人目の素数さん
垢版 |
2024/03/17(日) 03:00:50.13ID:VAa6dkvQ
>>958宿題な
忘れずやれよ
0960132人目の素数さん
垢版 |
2024/03/17(日) 03:07:31.30ID:HNHCaIr5
>>958
>箱入り無数目は∃y.∀x.P(x,t)とは書けない
書ける

>一方、xを全公開してなくても∀x.∃y.P(x,y)であることが証明されている
後者はPが具体的に書かれていないと証明されているのか不明。そもそも証明にはxが公開されているとかいう要素は関係ない
0961132人目の素数さん
垢版 |
2024/03/17(日) 03:09:28.49ID:VAa6dkvQ
時枝証明とおまえの持論は相容れない
おまえの持論の正しさを証明するには時枝証明の間違い箇所を具体的に指摘する必要がある
がんばれよw
0962132人目の素数さん
垢版 |
2024/03/17(日) 03:13:14.35ID:HNHCaIr5
>>961
記事の証明に間違いなんてねーぞ
そこは全員の共通認識だろ
今さら何言ってんの?
0963132人目の素数さん
垢版 |
2024/03/17(日) 03:13:50.93ID:VAa6dkvQ
>>960
>書ける
じゃyを書いてみて

>そもそも証明にはxが公開されているとかいう要素は関係ない
0260132人目の素数さん
2024/03/07(木) 16:55:42.19ID:C3Ro7iPT
>>259
∀が先頭についてるんだから相手側に全公開してるだろ
0964132人目の素数さん
垢版 |
2024/03/17(日) 03:15:50.26ID:VAa6dkvQ
>>962
>記事の証明に間違いなんてねーぞ
え???
じゃおまえは箱入り無数目成立派なの?
じゃ何に対し文句垂れてんの?
0965132人目の素数さん
垢版 |
2024/03/17(日) 03:16:26.41ID:HNHCaIr5
>>963
yは1でいいよ

>>そもそも証明にはxが公開されているとかいう要素は関係ない
>0260132人目の素数さん
>2024/03/07(木) 16:55:42.19ID:C3Ro7iPT
>>259
>∀が先頭についてるんだから相手側に全公開してるだろ
これの何が関係あるの?
0966132人目の素数さん
垢版 |
2024/03/17(日) 03:17:53.85ID:HNHCaIr5
>>964
お前らが適当に∀と∃を並べて曖昧なろんりを書いたり、確率変数を使った定式化を否定してるのに文句言ってんだよ
0968132人目の素数さん
垢版 |
2024/03/17(日) 03:19:42.66ID:VAa6dkvQ
>>965
>yは1でいいよ
ダメだこいつw yがどんな空間に属すかすら分かってないw

>これの何が関係あるの?
そもそもおまえは何に対して文句垂れてんの?
0969132人目の素数さん
垢版 |
2024/03/17(日) 03:21:48.03ID:VAa6dkvQ
>>966
確率変数を使った定式化を否定してる訳ねーだろw 何をどう勘違いしたらそうなるんだ?w
100列のいずれを選択するかが確率変数だと言ってんだよw 馬鹿かおまえは
0970132人目の素数さん
垢版 |
2024/03/17(日) 03:24:08.39ID:VAa6dkvQ
>>967
あいまいな論理式の前に
確率変数だから∀の位置がどうたらこうたらっておまえの持論の方がよっぽどデタラメだぞw
0971132人目の素数さん
垢版 |
2024/03/17(日) 03:25:50.44ID:HNHCaIr5
>>968
お前が∃y∀xの形で書けっていったんだろ、∃y∈{1}.∀x∈{1}.でいいじゃん残りは勝手に埋めるから
0972132人目の素数さん
垢版 |
2024/03/17(日) 03:26:17.62ID:VAa6dkvQ
もう一回聞くけどおまえは箱入り無数目記事は正しい、つまり、任意の出題列に対して回答者が確率99/100以上で勝てる戦略が存在すると思ってるのね?
0974132人目の素数さん
垢版 |
2024/03/17(日) 03:27:24.38ID:VAa6dkvQ
>>971
>∃y∈{1}.∀x∈{1}.でいいじゃん
それ箱入り無数目じゃないからダメ
0975132人目の素数さん
垢版 |
2024/03/17(日) 03:28:03.57ID:VAa6dkvQ
>>973
いいとは?
0976132人目の素数さん
垢版 |
2024/03/17(日) 03:28:05.55ID:HNHCaIr5
>>972
その確率をどういう情報をもとに計算したかによるから定式化によって答が変わるって言ってるの
0977132人目の素数さん
垢版 |
2024/03/17(日) 03:29:55.37ID:HNHCaIr5
>>974
なんで?
∃y∈{1}.∀x∈{1}.の後ろにお前が思う
正しい論理式をxとyの名前だけ変えて書けばいいじゃん
0978132人目の素数さん
垢版 |
2024/03/17(日) 03:30:01.31ID:VAa6dkvQ
いいとはどういう意味のいいなの?
そういう問題を考えてもいいなのか箱入り無数目をそう定式化してもいいなのか
後者なら完全に間違い
0981132人目の素数さん
垢版 |
2024/03/17(日) 03:32:33.77ID:VAa6dkvQ
>>976
大間違い
問題に曖昧さが無いから答えは唯一
定式化によって答えが変わるなら定式化の仕方が間違ってるだけ
0982132人目の素数さん
垢版 |
2024/03/17(日) 03:33:06.23ID:HNHCaIr5
何が完全に間違いだよ
記事の定式化の拡張になってるのに間違いになる要素なんてあるわけないだろ
0983132人目の素数さん
垢版 |
2024/03/17(日) 03:34:31.35ID:VAa6dkvQ
>>979
箱入り無数目にはそんな定式化は無い
問題自体が変わる
0984132人目の素数さん
垢版 |
2024/03/17(日) 03:35:46.11ID:VAa6dkvQ
>>982
>記事の定式化の拡張になってるのに間違いになる要素なんてあるわけないだろ
0985132人目の素数さん
垢版 |
2024/03/17(日) 03:37:11.72ID:VAa6dkvQ
>>980
それはおまえが箱入り無数目を理解していないだけのこと
0988132人目の素数さん
垢版 |
2024/03/17(日) 03:48:31.14ID:VAa6dkvQ
箱入り無数目の前にまずおまえは「見えないもの=確率変数」と言ったよな?
それ間違いな

壷の中でサイコロを振って1の目が出た
客は1に賭ける
客が勝つ確率は?

壷の中でサイコロを振って1の目が出た
客はランダムに賭ける
客が勝つ確率は?

客は1に賭けた
壷の中でサイコロを振る
客が勝つ確率は?

この問題に正答できるなら「見えないもの=確率変数」が間違いであることも分かるだろう
0989132人目の素数さん
垢版 |
2024/03/17(日) 03:51:42.62ID:VAa6dkvQ
>>986
箱入り無数目では箱の中身を確率変数とする定式化は間違いだって言ってるの
0992132人目の素数さん
垢版 |
2024/03/17(日) 03:57:36.02ID:VAa6dkvQ
>>986
(選択公理を除けば)問題文に曖昧さが無いから
回答者が勝率99/100以上で勝つ戦略があるか?という問いにはあるという答えしかない
記事の戦略で勝つ確率は?という問いには1-1/n以上という答えしかない
そういうことを言ってるんだよ
で、それには箱の中身を確率変数とする定式化はダメだと言ってるんだよ
0993132人目の素数さん
垢版 |
2024/03/17(日) 04:00:16.54ID:VAa6dkvQ
>>990
客は壷の中でサイコロを振ったとかこれから振るとかしか知らんよ 壷の中身は知らんよ 当然w
0994132人目の素数さん
垢版 |
2024/03/17(日) 04:02:29.63ID:VAa6dkvQ
>>991
そういう大口は>>988に正答してからたたこうな
0995132人目の素数さん
垢版 |
2024/03/17(日) 04:03:45.41ID:HNHCaIr5
>>992
中身を確率変数にしても記事と同じ結果になる確率空間は存在するだろ
ちゃんと拡張になってんだよ
0997132人目の素数さん
垢版 |
2024/03/17(日) 04:06:26.02ID:VAa6dkvQ
>>995
存在するなら書けやクズ
書きもせずに存在するする詐欺かおまえはw
0998132人目の素数さん
垢版 |
2024/03/17(日) 04:06:50.48ID:HNHCaIr5
>>994
お前確率論の本読み切れてないじゃん
正解かどうか以前に自分がどういう問題を出してるかすら分かってないだろ
0999132人目の素数さん
垢版 |
2024/03/17(日) 04:06:57.20ID:VAa6dkvQ
>>996
当たり前だろw
1000132人目の素数さん
垢版 |
2024/03/17(日) 04:07:49.86ID:HNHCaIr5
>>997
そんなん箱の中身がデルタ分布になってる場合に決まってるだろ
いい加減頭使えよ
10011001
垢版 |
Over 1000Thread
このスレッドは1000を超えました。
新しいスレッドを立ててください。
life time: 11日 20時間 3分 10秒
10021002
垢版 |
Over 1000Thread
5ちゃんねるの運営はUPLIFT会員の皆さまに支えられています。
運営にご協力お願いいたします。


───────────────────
《UPLIFT会員の主な特典》
★ 5ちゃんねる専用ブラウザからの広告除去
★ 5ちゃんねるの過去ログを取得
★ 書き込み規制の緩和
───────────────────

会員登録には個人情報は一切必要ありません。
4 USD/mon. から匿名でご購入いただけます。

▼ UPLIFT会員登録はこちら ▼
https://uplift.5ch.net/

▼ UPLIFTログインはこちら ▼
https://uplift.5ch.net/login
レス数が1000を超えています。これ以上書き込みはできません。

ニューススポーツなんでも実況